Evaluating Exam Review and Guide Q&A


260 50 8MB

English Pages 437 Year 2016

Report DMCA / Copyright

DOWNLOAD PDF FILE

Table of contents :
001 Canadian Pharmacy Review Q&A Content ver1
Misbah Biabani, Ph.D
Abbreviations
Part I: Biomedical sciences
Part II: Pharmaceutical Sciences
Part III: Social / Behavioural /Administrative Sciences
01 Chapter Human Anatomy Q&A
02 Gastrointestinal System Q&A
1. A person with Vitamin B12 deficiency due to intrinsic factors. Should take?
C) Patient is using NSAIDs
03 Chapter Nervous System Q&A
04 Chapter Cardiovascular System Q&A
05 Chapter Endocrine System Q&A ver1
ENDOCRINE SYSTEM
A. Sensitivity to cold
D. Constipation
B) refer to doctor to decrease dose of Synthroid
6. Which of the following stimulates secretion of calcitonin hormone from
C) Hypokalemia
06 Chapter Renal System Q&A ver1
RENAL DISORDERS
07 Chapter Liver functions and Chronic Liver Diseases Q&A ver1
E. None of the above
B. Cholecystitis
D. Hepatic encephalopathy
E) Liver cirrhosis
08 Chapter Respiratory System Q&A ver1
RESPIRATORY COMPLICATIONS
E) Bronchial cavity
09 Chapter Urinary System Q&A
Pharmacy Prep
Urinary System
10 Chapter The Eye and Ear Q&A
D. Optic nerve
Ans: A
E. None of the above
A) Cones
Ans. B
11 Chapter Blood and Anemia Q&A
Ans: A
Ans: B
12 Chapter Biochemistry Q&A ver1
13 Chapter Fluids Electrolytes and Nutrition Q&A ver1
PHARMACY PREP
NUTRITION
14 Chapter Microbiology Q&A ver1
PHARMACY PREP
MICROBIOLOGY
15 Chapter Cell and Molecular Biology Q&A ver1
CELL AND MOLECULAR BIOLOGY
C) Same time as transcription
A) factor 5 B) factor10 C) factor 8 D) factor 2 E) factor 9
X
X
xy
C. Oxidoreductases
16 Chapter Pharmacogenetics Q&A
D) The pharmacogenetics identifies the genetic variation that cause drug cause differences in drug response.
E) All of the above
17 Chapter Immunology and Immunization Q&A ver 1
IMMUNOLOGY and Immunizations
18 Chapter Biotechnology Q&A ver1
BIOTECHNOLOGY
B. Tumor necrosis factor alpha inhibitor
C) thrombocytopenia associated with cancer chemotherapy
E) chemotherapy induced anemia
Ans: A
Ans. A
9) Operlaveukin (IL-11) is indicated for?
Ans. A
C. Thrombocytopenia associated with cancer chemotherapy
D. Anemia associated with chronic renal diseases
E. None of the above
HAMA is?
B) Humanized chain linked to rat chain
D) One quarter the antibody is from one species origin where
19 Chapter Toxicology Q&A ver1
TOXICOLOGY
B. Myoglobin
Ans. A
BIBLIOGRAPHIC REFERENCE
20 Chapter Pharmacokinetic Q&A ver 1
PHARMACOKINETICS
K = 0.693/0.7 = 0.999
B.936 mg/hr
Cloxacillin 50% 50%
Ans-B
Enter 64
Multiply
Drug Dose
Form
Product Company
21 Chapter Rates and orders of reaction Q&A ver1
RATES AND ORDERS OF REACTIONS
22 Chapter Pharmacodynamics Q&A ver1
D. Concentration of AgE and Ag*E increase
23 Chapter Medicinal Chemistry Basics Q&A ver1
MEDICAL CHEMISTRY
D) Proline ring
E) Carbamate group
24 Chapter Medicinal Chemistry of Autonomic Nervous System Q&A ver 1
Pharmacy Prep
Medicinal Chemistry of Autonomic Nervous System
25 Chapter Medicinal chemistry of autocoids, NSAID and Opioid analgesics Q&A ver1
26 Chapter Medicinal Chemistry and Pharmacology of Cardiovascular Drugs Q&A ver1
MEDICINAL CHEMISTRY AND PHARMACOLOGY OF CARDIOVASCULAR SYSTEM
B. Warfarin
D) Proline ring
B. Digoxin
B. Ischemic stroke
27 Chapter Medicinal chemistry and pharmacology of psychiatric and neurological drugs
28 Chapter Medicinal Chemistry and Pharmacology of Endocrine System Q&A
29 Chapter Medicinal Chemistry and Pharmacology of Respiratory Drugs
30 Chapter Medicinal Chemistry and Pharmacology of Musculoskeletal Drugs Q&A
31 Chapter Medicinal Chemistry and Pharmacology of antimicrobials
32 Chapter Metabolism Q&A ver1
METABOLISM
33 Chapter Biopharmaceutics Q&A ver1
BIOPHARMACEUTICS
A-100% solute is ionized
Ans. C
14. A drug degradation at room temperature is determined by?
A. Noye-Whittney equation
B. Arrhenius equation
34 Chapter Pharmaceutics Q&A ver1
Physical Pharmacy
35 Chapter Pharmaceutical Exipients Q&A
36 Chapter Rheology Q&A ver1
RHEOLOGY
37 Chapter Pharmaceutical dosage forms Q&A ver 1
PHARMACEUTICAL DOSAGE FORMS
38 Chapter Drug Delivery Systems Q&A ver1
DRUG DELIVERY SYSTEMS
39 Pharmaceutical analysis Q&A ver1
PHARMACEUTICAL ANALYSIS
II- Addition of a specific buffer will increase the reaction
ANSWERS
BIBLIOGRAPHIC REFERENCE
40 Chapter Canadian Healthcare System Q&A
41 Chapter Pharmacy Regulations Q&A ver1
42 Chapter Social Behavioral aspects of pharmacy profession
43 Chapter Pharmacy Practice Management Q&A ver1
PHARMACY PREP
PHARMACY MANAGEMENT
B. Sole proprietorship
D. Oligopolistic competition
Beginning inventory $200,000
PHARMACY PRACTICE MANAGEMENT
Tips: [Sales - COGS/Sales] x 100
44 Chapter Pharmacoeconomics Q&A ver1
PHARMACOECONOMICS
A. Quality unit
6. Pharmacoeconomic studies are used to analyze cost of drugs to:
A. Healthcare system
A. Quality unit
11. Pharmacoeconomic studies are used to analyze cost of drugs to:
A. Healthcare system
PHARMACOECONOMICS
6. Ans: B
11. Ans: B
45 Chapter Drug discovery and new drug approval process Q&A
46 Chapter Epidemiology and clinical studies Q & A
EPIDEMIOLOGY and Clinical Studies
47 Chapter Biostatistics Calculations Q&A ver1
BIOSTATISTICS CALCULATIONS
48 Chapter Hospital Pharmacy
49 Chapter Basic Calculations ver1
Basic CALCULATIONS
3.A patient weight 180 lbs has admitted to emergence for congestive heart failure and severe edema. Patient was give furosemide iv infusion for the past 24 hours. After discharge the patient weight was 173 lbs. How many kg patient weight is lost?
50 Chapter Dose Calculations
Provera 5 mg tab QD
Ans: E
Ans: D
Ans: D
Ans-C
Ans. C
A. 300 mg
Total therapy 21 days
51 Chapter Dilutions and concentrations Q&A ver1
52 Chapter Brand and Generic Name Indexes Q&A
53 Chapter Prescription Processing and Dispensing Q&A ver1
PHARMACY PREP
54 Chapter Safety of drug use in special populations Q&A ver1
DRUGS IN SPECIAL POPULATIONS
55 Chapter Clinical Toxicology Q&A
A) S-adenosyl transferase
56 Chapter Professional Communication skills Q&A ver1
PHARMACY PREP
PHARMACY COMMUNICATION SKILLS
57 Chapter Bioethics and Professional Standards Q&A ver1
1. The ethical standard beneficence in pharmacy practice means:
A. Pharmacist act with justice
2. The ethical standard nonmaleficence in pharmacy practice means:
A. Pharmacist act with justice
3. The ethical standard veracity in pharmacy counselling means:
A. Pharmacist act with justice
4. The ethical standard justice in pharmacy practice means:
A. Pharmacist act with equality
E) Veracity
A. Discard the sterile preps and redo new batch
A. Beneficence
B. Nonmaleficence
C. veracity
D. Justice
E. Autonomy
C) Verify and give patient profile to EMS, it is in best interest of patient.
1. D
2. E
3. C
4. B
Tips: It is circle of care, you can share information after confirmation.
58 Chapter Drug Information Sources Q&A
B. A Canadian pharmacy journal
59 Chapter Medication Errors Q&a
B-Calm down patient
60 Chapter Health Promotion and Disease Prevention Q&A
61 Chapter Collaboration and Teamwork Q&A
62 Chapter Sterile Preparations Q&A
STERILE PREPARATIONS
A-Horizontal laminar airflow hood
B-Vertical Laminar airflow hood
63 Chapter Pharmaceutical Storage Conditions Q&A
64 Chapter Pharmaceutical Care and Drug Related Problems Q&A
65 Chapter Adverse Drug Reactions Q&A ver1
PHARMACY PREP
66 Chapter Drug Interactions Q&A ver1
DRUG & DRUG INTERACTIONS
E) Call the doctor, change to levofloxacin.
D. None of the above
DRUG & DRUG INTERACTIONS
67 Chapter Clinical Biochemistry Q&A
CLINICAL BIOCHEMISTRY
B. Sensitive TSH assay
A. Creatinine clearance
CLINICAL BIOCHEMISTRY
13. Ans: A
68 Chapter Therapeutic Drug Monitoring Q&A ver1
1-Which antipsychotic drugs monitored for WBC or CBC?
D-Haloperidol
3) Patient using statins should be monitored for?
A-LFT and CK-MM
1) Ans: B
3) Ans: A
69 Chapter OTC drugs for skin conditions Q&A
B-Calamine lotion
D-II and III only
10-What is incorrect in treatment of diaper rash?
B-clean diaper area with alcohol swabs
10- Ans: B
70 Chapter OTC and Prescription Drugs for Ophthalmic, Ear and Mouth Disorders Q&A
1. What is an optimal range of pH for ophthalmic products
5. What is correct statement about treatment in eruption cysts in infants
III-antibiotics
I- Increase of IOP
1.Ans: D
5. Ans: A
71 Chapter OTC antihistamines, decongestants, antitussives Q&A
OTC ANTIHISTAMINE, DECONGESTANTS AND ANTITUSSIVES
E-Mydriasis
A-recommend ASA
OTC ANTIHISTAMINE, DECONGESTANTS AND ANTITUSSIVES
72 Chapter OTC drugs for nausea, vomiting, constipation, diarrhea, hemorrhoids and helmenths Q&A
73 Chapter OTC drugs for headache, sports injuries, pressure ulcer, and low back pain Q&A
A-Irreversible inhibition of platelet aggregation
74 Chapter Asthma and COPD Q&A
75 Chapter Smoking cessation Q&A
PHARMACY PREP
SMOKING CESSATION
2) Nicotine patches are stored in?
A-Refrigerator B-Room temperature C-Cold temperature
D-Freezer E-Any temperature
A. Stored in room temperature
D-Avoid in sexual dysfunction
E. Nicotine gum
SMOKING CESSATION
2) Ans: B
76 Chapter Sleep Disorders Q&A
E-Chlorozepate
77 Chapter Eating Disorders Q&A
B)Weight in kg/height in meters
78 Chapter Gastrointestinal system Q&A
GASTROINTESTINAL DISORDERS
I- Villi
III- They decrease stomach acid secretion
III- Taken 2hrs before bedtime
II- Aluminum hydroxide
E) sucralfate
GASTROINTESTINAL DISORDERS
ANSWERS
79 Chapter Diabetes Q&A ver1
PHARMACY PREP
DIABETES
A-I only
E-May cause anorexia
13-What is correct about sulfonylureas?
E-Glyburide
E-Glyburide
DIABETES
13- Ans: E
80 Chapter Thyroid disorders Q&A ver1
E-Hypothyroidism(Graves disease
A-Tetraiodinase enzyme
III) Decreased TSH levels
C) Should not be cut
14-What is the most common side effects of propylthiouracil?
D-platelet aggregation
E-diarrhea
C-Iron supplements
D-Excessive Soya fiber
14- Ans: C
81 Chapter Contraception Q&A
E-Justice
E-Refer to doctor
82 Chapter Genitourinary and Gynecology Conditions Q&A ver1
GENITOURINARY CONDITIONS
A-Nocturia
III-vitamins, and estrogens deficiency may cause PMS
A-Hydrochlorthiazide
C-Call doctor to change other antibiotic
D-all of the above
D) None of the above
26. What is the drug of choice to treat patient with vaginal pruritus, curdy vaginal discharge, no odor and no color discharge?
83 Chapter Bones and joints complications Q&A ver1
84 Chapter Osteoporosis Q&A
85 Chapter Hypertension Q&A
C-200 mm Hg
D-120 mm Hg
86 Chapter Coronary artery Disease (CAD) Q&A
A-Amiodarone
10) All of the following are low molecular weight heparins (LMWH), except?
11) What is not true about low molecular weight heparins
(LMWH)?
A-It act by inhibiting factor Xa
10) E(was D before)
11) D
87 Chapter Stroke Q&A
88 Chapter Congestive Heart Failure (CHF) Q&A
PHARMACY PREP
89 Chapter Antiarrhythmic drugs Q&A
90 Chapter Peripheral Vascular Diseases Q&A
13) Which of the of the following can be used in a pregnant women with deep vein thrombosis ?
E)Reynaud phenomenon is may occur in old age
D) diabetes
13) Ans: D
91 Chapter Anticoagulants Q&A
92 Chapter Anti anxiety Disorder Q&A
93 Chapter Depression Q&A
D) Serotonin withdrawal syndrome
94 Chapter Psychosis Q&A
95 Chapter Dementia Q&A
13)What is incorrect statement?
96 Chapter Seizure and antiepileptic drugs Q&A
B-Complex partial
C-Simple partial
E) Absence seizure (petit mal)
E-Valproic acid
C-Tiagabine
E-Phenobarbital
25) All of the following are GABA analogs except?
25) Ans: E
97 Chapter Parkinsons Disease Q&A
E-Pseudo parkinsonism
B-MAO-B selective inhibitor
A-COMT inhibitor decrease metabolism of levodopa to 3-O-methyldopa
10-Tardive Dyskinesia is?
B-Involuntary movement of limbs, trunk, mouth and face
38) Parkinson’s disease is caused by deficiency of?
1- Ans: A
10- Ans: B
38) Ans: A
98 Chapter Anti-infective agents Q&A
2.What is empiric treatment of urinary tract infection, patient has allergy to penicillin?
A. cotrimoxazole 3 d
B. Fluroquinolone 3d
C. Amoxicillin 3 d
D. Cephalexin 7 d
E. Aminoglycoside 14 d
Ans. A
99 Chapter Anticancer drugs and Chemotherapy Q&A
A-Mesna
B-Doxarazoxane
C-Chlropropamide
D-Erythropoeitins
4) The drug of choice to treat delayed nausea and vomiting?
A-Ondansetron
B-Metoclopramide
C-Dexamethasone
D-Dimenhydrinate
E-Scopolamine
100 Chapter Natural Products and Pharmacognosy Q&A
Recommend Papers

Evaluating Exam Review and Guide Q&A

  • 0 0 0
  • Like this paper and download? You can publish your own PDF file online for free in a few minutes! Sign Up
File loading please wait...
Citation preview

www.pharmacyprep.com

Canadian Pharmacy Review

Pharmacy Prep

Evaluating Exam Review and Guide

Q&A MUST PASS July 2016 Misbah Biabani, Ph.D Director Toronto Institute of Pharmaceutical Sciences (TIPS) Inc. Toronto, ON M2N K7

Pharmacy Prep

Professional Exams Preparation Center 4789 Yonge St. Suites # 417 Toronto, ON, M2N 5M5 WWW.PHARMACYPREP.COM 416-223-PREP (7737) / 647-221-0457 Toronto Institute of Pharmaceutical Sciences Inc. © 2000 to 2016 TIPS Inc. All Rights Reserved.

Copyright © 2000 to 2016 Tips Inc

1

www.pharmacyprep.com

Canadian Pharmacy Review Disclaimer

Your use and review of this information constitutes acceptance of the following terms and conditions: The information contained in the notes intended as an educational aid only. It is not intended as medical advice for individual conditions or treatment. It is not a substitute for a medical exam, nor does it replace the need for services provided by medical professionals. Talk to your doctor or pharmacist before taking any prescription or over the counter drugs (including any herbal medicines or supplements) or following any treatment or regimen. Only your doctor or pharmacist can provide you with advice on what is safe and effective for you. Pharmacy prep make no representation or warranty as to the accuracy, reliability, timeliness, usefulness or completeness of any of the information contained in the products. Additionally, Pharmacy prep do not assume any responsibility or risk for your use of the pharmacy preparation manuals or review classes. In our teaching strategies, we utilize lecture-discussion, small group discussion, demonstrations, audiovisuals, case studies, written projects, role play, gaming techniques, study guides, selected reading assignments, computer assisted instruction (CAI), and interactive video discs (IVD). Our preparation classes and books does not intended as substitute for the advise of NABPLEX®. Every effort has been made to ensure that the information provided herein is not directly or indirectly obtained from PEBC® previous exams or copyright material. These references are not intended to serve as content of exam nor should it be assumed that they are the source of previous examination questions. ©2000 to 2016 TIPS Inc. All rights reserved. Foreword by Misbah Biabani, Ph.D Coordinator/Director, Pharmacy Prep Toronto Institute of Pharmaceutical Sciences (TIPS) Inc 4789 Yonge St. Suites 417 Toronto ON M2N 5M5, Canada

Copyright © 2000 to 2016 Tips Inc

2

www.pharmacyprep.com

Canadian Pharmacy Review

Content Abbreviations

Part I: Biomedical sciences 1. Human Anatomy 2. Gastrointestinal System 3. Nervous System 4. Cardiovascular System 5. Endocrine System 6. Renal System 7. Liver Function and Pathophysiology 8. Respiratory system 9. Urinary System 10. The Eye 11. Blood and anemia 12. Biochemistry 13. Clinical Biochemistry 14. Nutrition 15. Microbiology 16. Genetics and Molecular Biology 17. Pharmacogenomics 18. Immunology 19. Immunizations (vaccines) 20. Biotechnology 21. Toxicology Part II: Pharmaceutical Sciences 22. Basic Calculations 23. Pharmacy Calculations: Dosage 24. Pharmacy Calculations: Dilutions 25. Pharmacokinetics 26. Rates and Orders of Reactions 27. Pharmacodynamics Copyright © 2000 to 2016 Tips Inc

3

www.pharmacyprep.com

Canadian Pharmacy Review

28. Basics of Medicinal Chemistry 29. Medicinal chemistry and Pharmacology of Drugs that act on autonomic nervous system 30. Medicinal chemistry and Pharmacology of Histamines, Serotonin, Prostaglandin and NonSteroidal anti-inflammatory Drugs

31. Medicinal chemistry and pharmacology of cardiovascular drugs 32. Medicinal chemistry and pharmacology of psychiatric & neurological diseases 33. Medicinal chemistry and pharmacology Endocrine drugs 34. Medicinal Chemistry and Pharmacology of Respiratory Drugs 35. Medicinal Chemistry and Pharmacology of Musculoskeletal drugs 36. Medicinal Chemistry and Pharmacology of Antimicrobial 37. Drug Metabolism 38. Biopharmaceutics 39. Physical Pharmacy 40. Pharmaceutical Excipients 41. Rheology 42. Pharmaceutical Dosage Forms 43. Drug delivery Systems 44. Sterile Preparations 45. Extemporaneous Pharmaceutical Preparations 46. Pharmaceutical Analysis Part III: Social / Behavioural /Administrative Sciences 47. Bioethics and Professional ethics 48. Canadian Healthcare System 49. Canadian Pharmacy Regulations and administration 50. Social and Behavioural Aspects of Pharmacy Profession 51. Pharmacy Management 52. Pharmacoeconomics 53. Drug Information Resources 54. The new drug Approval Process 55. Basics of Clinical Research and Epidemiology 56. Biostatistics 57. Hospital Pharmacy Copyright © 2000 to 2016 Tips Inc

4

www.pharmacyprep.com

Canadian Pharmacy Review

Part IV: Pharmacy Practice/ Therapeutics and OTC Drugs 58. Generic and Brand Index 59. Prescription processing and Medication dispensing 60. Pharmaceutical Care and Drug Related Problems 61. Adverse Drug Reactions and Management 62. Drug Interactions 63. Therapeutic Drug Monitoring 64. Safety of medications in special populations 65. Identification and prevention of drug toxicity 66. Professional Pharmacy Communication Skills 67. Medication Errors 68. Pharmaceutical Preparation Storage Conditions 69. OTC and Prescription Drugs for Dermatological Disorders 70. OTC and Prescription Drugs for Ophthalmic, Ear and Mouth Disorders 71. OTC Drugs antihistamine, decongestants, antitussives, Expectorants 72. OTC Drugs for Nausea, Vomiting, Constipation, Diarrhea, and Hemorrhoids 73. OTC Drugs for Headache, Sports Injuries, Pressure Ulcers, Low Back Pain 74. Asthma and Chronic Obstructive Pulmonary Disease 75. Smoking cessation 76. Insomnia 77. Eating disorders 78. GERD, Ulcers, Inflammatory Bowel Disease and Irritable Bowel Syndrome 79. Diabetes Mellitus Type I and Type 2 80. Thyroid Disorders 81. Contraception 82. Reproductive, Gynaecologic and Genitourinary Disorders 83. Bone and Joint Complications 84. Osteoporosis 85. Hypertension 86. Coronary Artery Diseases 87. Stroke 88. Congestive Heart Failure 89. Cardiac Arrhythmias Copyright © 2000 to 2016 Tips Inc

5

www.pharmacyprep.com

Canadian Pharmacy Review

90. Peripheral vascular diseases 91. Anticoagulants 92. Anxiety Disorder 93. Depression 94. Psychosis and schizophrenia 95. Dementia 96. Seizures and epilepsy 97. Parkinson’s Disease 98. Anti-microbial agents 99. Anticancer drugs and chemotherapy 100. Natural Products and Pharmacognosy

Copyright © 2000 to 2016 Tips Inc

6

PharmacyPREP.com

Human anatomy

PHARMACY PREP HUMAN ANATOMY 1. What is an opposite of anterior view? A. Ventral view B. Posterior view C. Lateral view D. Medial view E. Supination Ans. B Tips. Ventral (anterior) is opposite to Dorsal (posterior) and the lateral is opposite to medial.

2. Epithelial tissue is present in all of the following except? A. sweat gland B. milk gland C. Endocrine gland D. Blood vessel Ans. D 2. All of the following have skeletal muscles, except? A. arm B. biceps C. triceps D. heart E. Foot Copyright © 2000-2015 TIPS Inc. Unauthorized reproduction of this manual is prohibited. This manual is being used during review sessions conducted by PharmacyPrep.

1-1

PharmacyPREP.com

Human anatomy

Ans. D

Copyright © 2000-2015 TIPS Inc. Unauthorized reproduction of this manual is prohibited. This manual is being used during review sessions conducted by PharmacyPrep.

1-2

PharmacyPrep.Com

Gastrointestinal System PHARMACY PREP GASTROINTESTINAL SYSTEM

1. A person with Vitamin B12 deficiency due to intrinsic factors. Should take? A. Vitamin B 12 supplements B. Vitamin B 12 IM inj. C. Vitamin B 12 oral D. Vitamin B 12 inj. and folic acid Ans. B 2. PPI & H 2 RAs act on? A. G cells B. Gastrin C. Parietal cell D. Hydrochloric acid Ans. C 3. Which of the following may increase excessive acid secretion in patient with? A. high carbohydrate diet B. high protein diet C. High liquid diet D. High salt diet E. All of the above Ans. b 4. What is the main cause of dyspepsia? A. GERD B. Peptic ulcers C. Reflux esophagitis D. Gastric cancer Ans. B

5. What is not symptoms associated with irritable bowel syndrome? A. chronic constipation B. abdominal pain C. Chronic diarrhea D. GI bleeding Ans. D 6. Epigastric pain is a symptom of Ulcer. (less likely GERD), GERD is heartburn, this could be regurgitation of acid into esophagus to throat. 2-1

PharmacyPrep.Com

Gastrointestinal System

A. Ulcer B. GERD C. Appendicitis D. Angina E. All of the above Ans. A Tips: Epigastric pain is a symptom of Ulcer. (less likely GERD), GERD is heartburn, this could be regurgitation of acid into esophagus to throat. 7. Fistula or granulomas or skip patches occurs in? A. Irritable bowel disease B. Ulcerative colitis C. Crohn's disease D. Gastritis E. GERD Ans. C 8. Hyper acid secretion caused by? A) Heart burn (reflux of gastric content into esophagus) B) Peptic ulcers (H. pylori) C) Ulcerative colitis (inflammatory) D) Zollinger Ellison syndrome E) GIT cancers Ans. D Tips. Zollinger-Ellison Syndrome occurs when gastrin secreted by non-beta cell tumors of pancreas. 9. A regular customer of your pharmacy, is taking treatment of osteoarthritis. Recently got gastritis? What is correct? A) Patient is using acetaminophen B) Patient is using corticosteroids C) Patient is using NSAIDs D) Patient is using methotrexate E) Patient has heartburn Ans. C 10. What is the most sensitive and specific means to diagnose the cause of dyspepsia? A. Endoscopy B. CT scan C. Magnetic resonance imaging D. Urea Breath Test E. Esophagogastroduodenoscopy Ans. A 2-2

PharmacyPrep.Com

Gastrointestinal System

11. Which of the following bacteria can cause chronic peptic ulcer disease? A. E. coli B. H. pylori C. S. aureus D. S. pneumonia E. Candida infections Ans. B 12. What is the laboratory test confirm the peptic ulcer disease caused by H. pylori? A. Endoscopy B. CT scan C. Magnetic resonance imaging D. Urea Breath Test E. Esophagogastroduodenoscopy Ans. D 13. All GI conditions are associated with GI bleeding symptoms? except A) GI cancer B) Pancreatitis C) Gastritis D) Crohn's disease E) Ulcerative colitis Ans. b Tips. Pancreatitis has no GI bleeding and pancreatitis symptoms are upper abdominal pain that radiates into back. It may aggravated by eating, especially foods high in fat. Tender abdomen, nausea and vomiting and increased heart rate. 14. What type of hernia is common in seniors or elderly? A) heital hernia B) sliding hernia C) Para esophageal hernia D) None of the above Ans.A 15. Polypeptides and caffeine stimulates secretion of? A. parietal cells B. vitamin B 12 C. gastrin D. chyme E. amylase Ans. C 16. When parietal cells are stimulated, they secrete? A. Intrinsic factor and gastrin 2-3

PharmacyPrep.Com

Gastrointestinal System

B. Intrinsic factor and HCl C. Intrinsic factor and HCO 3 D. Intrinsic factor and mucus Ans. B 17. A patient of your pharmacy, recently had gastrectomy and ileum was removed. Which of the following can be deficient? A. Iron deficient anemia B. Calcium deficient due to deficiency of vitamin D C. Vitamin B 12 deficiency due to deficiency of intrinsic factor D. Vitamin K deficiency due to deficiency if GI bacteria E. All of the above Ans. C

2-4

PharmacyPrep.Com

Nervous System PHARMACY PREP Nervous System

1. Which part of brain that controls important cognitive skills in humans, such as emotional expression, problem solving, memory, language? A. Frontal lobe B. Occipital lobe C. Temporal lobe D. Parietal lobe E. Brainstem Ans. A

2.Involuntary functions are controlled by? A. Cerebellum B. Cerebrum C. Cortex D. Hypothalamus Ans. B 3.Dopaminergic pathways are associated with? A. occipital lobe B. Middle lobe C. Frontal lobe D. Temporal lobe Ans. C

4. Which part of brain that controls important cognitive skills in humans, such as emotional expression, problem solving, memory, language? A. Frontal lobe B. Occipital lobe C. Temporal lobe D. Parietal lobe E. Brainstem Copyright © 2000-2015 TIPS Inc. Unauthorized reproduction of this manual is prohibited. This manual is being used during review sessions conducted by Pharmacy Prep.

3-1

PharmacyPrep.Com

Nervous System

5. Patient is diagnosed with Alzheimer's dementia. The Alzheimer's is associated with? A. Temporal lobe B. Frontal lobe C. Occipital lobe D. Brain stem Ans. b

Copyright © 2000-2015 TIPS Inc. Unauthorized reproduction of this manual is prohibited. This manual is being used during review sessions conducted by Pharmacy Prep.

3-2

PharmacyPrep.com

Cardiovascular System PHARMACY PREP CARDIOVASCULAR SYSTEM

1. Which of the following cardiovascular disease is the most commonly associated with atherosclerotic plaques? A. Angina B. Myocardial infarction C. Deep vein thrombosis D. Ischemic stroke E. All of the above Ans. e Tips: if stable plaque than commonly cause angina, if plaque is ruptured than cause inflammation can lead to MI. 2. What is the most common cause of arteriosclerosis is? A) high LDL B) Smoking C) Hypertension D) emotional stress E) all of the above Ans. A 3. Clarithromycin cause QT prolongation, it associated? A. Atrial depolarization B. ventricular depolarization C. Atrial tachycardia D. side effects of drugs Ans. B 4. A patient with atrial fibrillation, have characteristic effect on ? A. affect on QRS wave B. affect on P wave C. affect on T wave D. affect on Q-T prolongation Ans. B

Copyright © 2000-2016 TIPS Inc. Unauthorized reproduction of this manual is prohibited. This manual is being used during review sessions conducted by PharmacyPrep.

PharmacyPrep.com

Cardiovascular System

Copyright © 2000-2016 TIPS Inc. Unauthorized reproduction of this manual is prohibited. This manual is being used during review sessions conducted by PharmacyPrep.

www.Pharmacyprep.com

Endocrine System PHARMACY PREP. ENDOCRINE SYSTEM

1. Characteristic symptoms of hypothyroidism may include all of the following, EXCEPT: A. Sensitivity to cold B. Heat intolerance C. Lethargy D. Constipation E. Weight gain Ans. B 2. A diabetic patient mistakenly taken double dose of insulin. Which of the following is NOT effect of excessive insulin administration? A) Sweating B. palpitation C. Confusion D. Diarrhea E. Fatigue Ans. D 3. Which of the following is NOT effect of excessive dose of levothyroxine? A. diarrhea B. Tachycardia C. sensitive to heat D. oily skin E. hypertension Ans. D 4. KCR is 49 yo women is using treatment of Levothyroxine (Synthroid) 75 mcg daily for hypothyroidism. She complains palpitation, weight loss and sensitivity to heat. What is appropriate? A) refer doctor to increase dose of Synthroid 100 mcg B) refer to doctor to decrease dose of Synthroid C) hypothyroid symptoms so should refer to doctor D) She may have some new problem so refer to doctor E) None Ans. B 5. MK is 30 year old female taking insulin for type 1 DM. Which of the following conditions decrease requirement of insulin dose? A) Pregnancy B) Infection

Copyright protected by Tips Inc

Protect copyright and prevent piracy

5-1

www.Pharmacyprep.com

Endocrine System

C) Physical activities D) Heavy meals E) Acute illness Ans . C 6. Which of the following stimulates secretion of calcitonin hormone from thyroid gland? A) hypocalcemia B) hypercalcemia C) Hypokalemia D) Hyperkalemia E) hypertension Ans. B 7. Thyroid stimulating hormone (TSH) secretes from? A. Anterior pituitary gland B. Posterior pituitary gland C. Thyroid gland D. Parathyroid gland E. None of the above Ans. A

Copyright protected by Tips Inc

Protect copyright and prevent piracy

5-2

Pharmacyprep.com PHARMACY PREP RENAL DISORDERS 1. Reabsorption of drugs in kidney does NOT depend on: A. Flow rate B. Tonicity C. Tubular secretion D. pH E. Metabolism Ans. C 2. Prerenal acute renal failure (ARF) is characterized as? A) Inadequate blood circulation (perfusion) to the kidneys B) Excessive blood perfusion to the kidney C) Excessive drug elimination D. Bladder cancer E) All of the above Ans. A 3. All of the following drug cause metabolic alkalosis, except? A. ACIDzolamide B. Thiazides C. hydrochlorothiazide D. Loop diuretics E. Furosemide Ans. A 4. A man with history of severe diarrhea, the loss of HCO 3 - from GI tract causes a decrease in blood [HCO 3 -], and increase CO 2 his values are thus this man has? pH = 7.25, pCO 2 = 24 mmHg, [HCO 3 -] = 10 mEq/L A. Metabolic alkalosis B. Metabolic acidosis C. Respiratory acidosis D. Respiratory alkalosis E. Normal acid base status Ans. B

5. What is incorrect about potassium levels? A. Chronic renal disease may cause hyperkalemia B. Adrenal cancer may cause hypokalemia Copyright © 2000-2016 TIPS Inc. Unauthorized reproduction of this manual is prohibited. This manual is being used during review sessions conducted by PharmacyPrep.

6-1

Pharmacyprep.com C. ACE Inhibitors like captopril may cause hypokalemia D. Spironolactone may cause hyperkalemia E. Hydrochlorothiazide may cause hypokalemia Ans. C

6. A regular customer of your pharmacy, age 55 yo, medication profile include Ramipril 10 mg, valsartan 5 mg, and recently doctor added spironolactone. Patient diet include excessive intake of bananas. All of the following are pharmacist concern, except? A. Ramipril B. Valsartan C. Spironolactone D. Age 55 yo E. Bananas Ans. D 7) Which of the following is the least preferable combination antihypertensive in a patient whose K+ levels are 5.5 mEq/L? A) Ramipril/Hydrochlorothiazide B) Valsartan/Amlodipine C)Perindropril/spironolactone D) Losartan/Hydrochlorothiazide E) Amlodipine/Atorvastatin Ans. C 8.Characteristics of nephrotic symptoms, include all except? A) Proteinuria B) Hypoalbuminemia C) edema D) Hyperalbuminemia E) hyperlipidemia Ans: D 9.A patient eGFR declined significantly and doctor suspect chronic kidney diseases. All of the following are associated with chronic kidney disease, except? A. Intravenous radiographic contrast B. NSAIDs or COX-II inhibitors C. Volume depletion D. Strict blood pressure or blood glucose control E. Aminoglycosides Ans. D

Copyright © 2000-2016 TIPS Inc. Unauthorized reproduction of this manual is prohibited. This manual is being used during review sessions conducted by PharmacyPrep.

6-2

Pharmacyprep.com

Copyright © 2000-2016 TIPS Inc. Unauthorized reproduction of this manual is prohibited. This manual is being used during review sessions conducted by PharmacyPrep.

6-3

PharmacyPrep.Com

Liver Function and Diseases PHARMACY PREP

LIVER PHYSIOLOGY AND CHRONIC LIVER DISEASES 1. Which of the following is NOT a chronic hepatitis infection? A. Hepatitis A B. Hepatitis B C. Hepatitis C D. Hepatitis B and C E. None of the above Ans. A 2) Obstruction of bile secretion into duodenum from liver is referred as? A. Cholestasis or cholestatitis B. Cholecystitis C) Cholelithiasis D. Hepatic encephalopathy E. Ascites Ans. A Tips. cholestasis is a reduction or stoppage of bile flow. The cause of cholestasis is acute hepatitis, alcoholic liver disease, primary biliary cirrhosis, hormonal changes in pregnancy, stones in bile duct, cancer of bile duct, pancreatitis.

3. All of the following are complications of cirrhosis, except? A. ascites B. liver cancer C. hepatic encephalopathy's D. edema E. diabetes ans. E 4. What are the common cause of liver cirrhosis? A. chronic alcohol B. hepatitis A C. chronic alcohol and hepatitis B & C D. Hepatitis B and C E. hypertension Ans. C 5. Liver cirrhosis is a type of end stage chronic liver disease (liver cirrhosis)? What clinical laboratory tests is used to diagnose ? A) ALT B) AST Copyright © 2000-2016 TIPS Inc. Unauthorized reproduction of this manual is prohibited. This manual is being used during review sessions conducted by PharmacyPrep.

7-1

PharmacyPrep.Com

Liver Function and Diseases

C) Bilirubin D) Albumin E) LDH Ans. C Tips. The liver cirrhosis diagnosis can be confirmed by a liver biopsy (removal of a tissue sample for examination under microscope). CT scan and ultrasound may show that liver is shrunken or abnormalities. Alcoholic liver disease can result into liver cirrhosis. Trans aminases AST to ALT ratio >2:1 strongly suggest alcohol abuse. 6. MK is a 50 year old man diagnosed for ascites. Which of the following are causes of ascites, except A) Tuberculosis B) Abdominal surgeries C) Congestive heart failure D) Stroke E) Liver cirrhosis Ans. D Tips. Stroke is cerebrovascular conditions and not associated with chronic liver disease ascites. 7. Drugs that causes cholestatic jaundice side effects? A) Cotrimoxazole B) Erythromycin base C) Erythromycin lactobacillus D) Probiotics E) Erythromycin estolate Ans: E 8. Liver enzymes like cytochrome CYP450 cause drug metabolism in liver. What is NOT related to drugs that are metabolized in liver? A) metabolism effects by tissue binding B) metabolism changes with liver blood circulation C) metabolism changes with intrinsic activity D) metabolism effected by chronic liver diseases E) oxidative metabolism is catalyzed by cytochrome CYP450. Ans: C 9. Hepatitis B is transmitted by all, except? A) contact with blood and its products B) sexual transmission C) contact oral secretion D) sharing needle E) blood transfusion Copyright © 2000-2016 TIPS Inc. Unauthorized reproduction of this manual is prohibited. This manual is being used during review sessions conducted by PharmacyPrep.

7-2

PharmacyPrep.Com

Liver Function and Diseases

Ans. C Tips: oral secretion contact can cause hepatitis A. 10. Phase II metabolic reaction glucoronidation is effected by Enterohepatic recirculation. Which of the following drugs are effected? A) drugs that undergoes phase II metabolism B) estrogen/progestins oral contraceptive pills C) drugs with phase I metabolism D) Drug with first pass metabolism E) Lipid soluble drugs Ans: A 11. What is the most common cause of alcoholic liver diseases? A. Corticosteroids therapy B. Acetaminophen toxicity C. Alcohol abuse D. Ascites E. Wilson's disease Ans. c

Copyright © 2000-2016 TIPS Inc. Unauthorized reproduction of this manual is prohibited. This manual is being used during review sessions conducted by PharmacyPrep.

7-3

PharmacyPrep.Com

Respiratory System PHARMACY PREP. RESPIRATORY COMPLICATIONS

1) Emphysema is? A) COPD B) Fibrosis C) Bronchitis D) Airway disease E) None of the above Ans. A 2. Dyspnea means A. Painful muscle spasms B. Pain in the heart C. Pain in extremities D. Painful breathing E. Painful menstruation Ans. D 3. Condition characterized by a irreversible form of airflow obstruction is known as: A. Aneurism B. Emphysema C. Embolism D. Cirrhosis E. Jaundice Ans. D 4. Asthma is a condition of respiratory tract that may be aggravated by? A) Allergens B) Cold weather C) Exercise D) Emotional stress E) All are correct Ans. E 5. The MOST well known characteristic symptom of asthma include: A. Wheezing B. Mucosal edema C. Cough D. Chest tightness E. Tachycardia Copyright © 2000-2016 TIPS Inc. Unauthorized reproduction of this manual is prohibited. This manual is being used during review sessions conducted by PharmacyPrep.

8-1

PharmacyPrep.Com

Respiratory System

Ans. A 6. All are correct concerning the action of corticosteroids in asthma status, EXCEPT: A. Suppress the inflammatory response B. Decrease production of inflammatory mediators C. Decrease airway responsiveness to inflammation D. Relieve brochocontriction E. Increase β-agonist receptors response Ans. E 7. The capacity of inhalation is depends on the volume of? a) alveoli B) total lung capacity C) airways D) Bronchus E) Bronchial cavity Ans. B Tips: Total lung capacity is the volume in the lungs after a maximal inspiration. Total lung capacity includes tidal volume, inspiratory reserve volume, Expiratory volume and residual volume. 8. Peak flow meter is test to determine asthma severity for patient at home. This test measures? A) The highest forced expiratory flow B) Volume that has been exhaled at the end of the first second (FEV 1 ). C) Total lung capacity D) Expiration rate E) Inspiration rate Ans. A 9. Spirometer is device used in clinics to diagnose asthma? Spirometer measures? A) The highest forced expiratory flow B) Volume that has been exhaled at the end of the first second (FEV 1 ). C) Total lung capacity D) Expiration rate E) Inspiration rate Ans. b 10. Factors that decrease the respiration except: A. Mucosal edema B. Increase brachial secretion C. Increase ventilation D. Bronchospasm

Ans: C Copyright © 2000-2016 TIPS Inc. Unauthorized reproduction of this manual is prohibited. This manual is being used during review sessions conducted by PharmacyPrep.

8-2

PharmacyPrep.Com

Respiratory System

Copyright © 2000-2016 TIPS Inc. Unauthorized reproduction of this manual is prohibited. This manual is being used during review sessions conducted by PharmacyPrep.

8-3

www.pharmacyprep.com

Urinary System

Pharmacy Prep Urinary System 1. Which of the following is NOT a symptom of benign prostatic hyperplasia? A. Frequent urine B. Nocturia C. Irritation D. Jet urination E. All of the above Ans. D Tips: in BPH normal stream of urination is not possible. 2. Which of the following is NOT a symptom of urinary tract infection? A) Increase in urinary frequency B) Burning sensations C) Fever D) Bactiurea E) Weight loss Ans. E 3. What is the most common type of UTI? A. Complicated UTI B. Uncomplicated UTI C. Cystitis D. Pylonephritis E. Ureteritis Ans. C Tips. cystitis is bladder infection, and the most common. 4. What is the most common agent that cause UTI? A. S.aureus B. P. aeruginosa C. E. coli D. Syphilis E. Gonorrhea Ans. C 5. What is the drug of choice to treat cystitis? A. Amoxicillin B. Cefixime C. Cotrimoxazole D. Nitrofurontoin Copyright © 2000-2012 TIPS Inc. Unauthorized reproduction of this manual is prohibited. This manual is being used during review sessions conducted by PharmacyPrep.

9-1

www.pharmacyprep.com

Urinary System

E. Trimethoprim Ans.c 6. What is the distinguishing symptom of complicated UTI? A. High fever B. blood in urine C. dysurea D. frequent urination Ans. b 7. What is the drug of choice to treat benign prostatic hyperplasia? A. Doxazosin B. Tadalafil C. Finasteride D. Saw palmetto E. oxybutynin Ans. c 8. All of the following used to treat BPH? except A. Doxazosin B. Tadalafil C. Finasteride D. Saw palmetto E. oxybutynin Ans. E 9. Which of the following is stress incontinence? A) Relaxed pelvic floor B) Urethral blockade C) inability void urine D) Urgency of urine E) Urinary retention Ans: A 10. What is the most common pathogen associated with cystitis? A. S. aureus B. Gonorrhea C. Chlamydia D. E. coli E. Shigella sp Ans. D 11. Which of the following medical conditions cause difficulty to urination in stream? A. Urinary tract infections Copyright © 2000-2012 TIPS Inc. Unauthorized reproduction of this manual is prohibited. This manual is being used during review sessions conducted by PharmacyPrep.

9-2

www.pharmacyprep.com

Urinary System

B. Prostatitis C. Benign prostatic hyperplasia D. Urinary incontinence E. Cystitis Ans: C 12. Hyperplasia is? A. Increased in cell size B. Increased in cell numbers C. Decreased in cell size D. Decreased in cell numbers E. Increase cell size and numbers Ans. B 13. Hypertrophy A. Increased in cell size B. Increased in cell numbers C. Decreased in cell size D. Decreased in cell numbers E. Increase cell size and numbers Ans. A 14. Which of the following cause tumors? A. hypertrophy B. hyperplasia C. Atrophy D. hyperacidity Ans. b 15. KP is a 37 year old women. Presents to doctor with symptoms of frequent urination at work place. Doctor has diagnosed with urinary incontinence. Her social history, denies taking alcohol and smoking. She is mother of 2 children and gave a birth to her 3rd child recently a month ago. What type of incontinence? A) Urge incontinence B) Overflow incontinence C) Stress incontinence D) Urinary tract infection 16. What drugs are least likely used in patient with urinary incontinence? A. oxybutynin B. diuretics C. amitriptyline D. All of the above ans. B Copyright © 2000-2012 TIPS Inc. Unauthorized reproduction of this manual is prohibited. This manual is being used during review sessions conducted by PharmacyPrep.

9-3

www.pharmacyprep.com

Urinary System

17. Urinary incontinence in children is defined as repeated daytime or night time voiding urine into the bed or cloths. It is termed as? A. Bed sores B. Bed wetting C. Wetting Phenomenon D. All of the above Ans. B 18. Enuresis is bed wetting is treated by? A. Diuretics B. Desmopressin C. Fluoxetine D. Diphenhydramine Ans. B Tips: Desmopressin is antidiuretic hormone analog. 19. Tamsulosin is used to treat benign prostatic hyperplasia symptoms. It blocks alpha 1a receptors and facilitates urine flow? A) coronary arteries B) bladder arteries C) prostate arteries D) cerebral arteries E) all of the above Ans. B 20. Testosterone to dihydrotestosterone is catalyzed by 5-alpha reductase. The dihydrotesterone cause prostate growth. What drugs inhibit 5-alpha reductase and reduce prostate size? A) Alpha blockers B) tamsulosin C) terazosin D) androgen agonist E) androgen antagonist Ans. E Tips. Finasteride is an androgen antagonist. The drug of choice for BPH. Finasteride and dutasteride are androgen antagonist. act by inhibiting 5-alpha reductase. 21. Which of the following is least likely associated with UNCOMPLICATED urinary tract infections? A. dysurea B. urinary urgency C. fever D. turbid urine E. urinary frequency Copyright © 2000-2012 TIPS Inc. Unauthorized reproduction of this manual is prohibited. This manual is being used during review sessions conducted by PharmacyPrep.

9-4

www.pharmacyprep.com

Urinary System

Ans. d Ans.D

Copyright © 2000-2012 TIPS Inc. Unauthorized reproduction of this manual is prohibited. This manual is being used during review sessions conducted by PharmacyPrep.

9-5

www.PharmacyPrep.com

Human anatomy

PHARMACY PREP The Eye 1. What is the other name of blind spot? A. Optic disc B. Retina C. Cornea D. Optic nerve E. None of the above Ans: A

2) What is the rate-limiting step in ophthalmic drops? A. Optic disc B. Retina C. Cornea D. Optic nerve E. None of the above Ans. C

Copyright © 2000-2012 TIPS Inc. Unauthorized reproduction of this manual is prohibited. This manual is being used during review sessions conducted by PharmacyPrep.

10-1

www.PharmacyPrep.com

Human anatomy

3. At night dark vision (dim light), what is the most sensitive for motion, photoreceptors in retina? A) Cones B) Rods C) Cones and rods D) Optic nerve E) Cornea Ans. B

14. The major light absorbing pigment in retinal photoreceptors is: a. rhodopsin b. melanin c. glutamate d. chlorophyll Copyright © 2000-2012 TIPS Inc. Unauthorized reproduction of this manual is prohibited. This manual is being used during review sessions conducted by PharmacyPrep.

10-2

www.PharmacyPrep.com

Human anatomy

Ans: A

16) What is incorrect about sympathetic blockers, example timolol in treatment of glaucoma? A) Decrease glaucoma B) Decrease IOP by inhibiting formation of aqueous humor. C) Decrease IOP by increase outflow of aqueous humor. D) First line therapy for glaucoma E) Ophthalmic drops are available Ans. C

Copyright © 2000-2012 TIPS Inc. Unauthorized reproduction of this manual is prohibited. This manual is being used during review sessions conducted by PharmacyPrep.

10-3

www.PharmacyPrep.com

Human anatomy

MP is a 55 year old male using Timolol 1 gtts bid ou. If patient medication profile have the following drugs. Which of the following medications are the pharmacist concern? A) Hydrochlorothiazide B) Salbutamol C) Ramipril D) Metformin E) Levothyroxine Ans. B

In the above prescription Timolol 1 gtt bid ou what is correct direction? A) instil one drop in both eyes B) instill one drop two times daily in both eyes C) Instill two drops once daily in both eyes D) Instill one drop two times daily in right eye E) Instill one drop two times daily in left eye Copyright © 2000-2012 TIPS Inc. Unauthorized reproduction of this manual is prohibited. This manual is being used during review sessions conducted by PharmacyPrep.

10-4

www.PharmacyPrep.com

Human anatomy

Ans. B

A prescription of ciprofloxacin+dexamethasone otic prep? What is correct statement? A. instill in eye only B. instill in ear only C. can be instilled in eye and ear D. taken orally E. applied topically Ans. B

What is the drug of choice to treat open angle glaucoma? A. Latanoprost B. Latanoprost + timolol C. Timolol D. Ciprofloxacin ophthalmic E. Dexamethasone Ans. C Copyright © 2000-2012 TIPS Inc. Unauthorized reproduction of this manual is prohibited. This manual is being used during review sessions conducted by PharmacyPrep.

10-5

www.PharmacyPrep.com

Human anatomy

A doctor prescribed timolol 1 gtt daily as directed. What is correct? A. Instill one drop in both eyes B. Instill one drop in left eye C. Instill one drop in right eye D. Instill one drop in affected eye E. Instill one drop in eye and ear Ans. D

MK is a 55 year old asthma patient. Recently diagnosed for open angle glaucoma. Doctor is considering to prescribe therapy for glaucoma. Which of the following is least likely suitable? A) latanoprost B) Dorzolamide C) Acetazolamide D) Timolol E) Pilocarpine Ans. D Copyright © 2000-2012 TIPS Inc. Unauthorized reproduction of this manual is prohibited. This manual is being used during review sessions conducted by PharmacyPrep.

10-6

www.PharmacyPrep.com

Human anatomy

17. What are the more sensitive photoreceptors in eye? A. Cones B. rods C. rods and cones D. cataract E. pupil Ans: B All of the following drug have effect on eye, except? a) amiodarone B) hydrochloroquine C) rifampin D) ethambutol E) isotretinoin ans: B

JP is a 50 year old asthma patient and also receiving treatment for hypertension and using metoprolol 50 mg daily. Recently diagnosed for Copyright © 2000-2012 TIPS Inc. Unauthorized reproduction of this manual is prohibited. This manual is being used during review sessions conducted by PharmacyPrep.

10-7

www.PharmacyPrep.com

Human anatomy

glaucoma. Doctor considering to prescribe latanoprost to treat glaucoma. Why doctor want to choose prostaglandin analog? A) patient has asthma and also using metoprolol B) patient has hypertension also using metoprolol C) Patient has asthma and hypertension D) Patient is 50 year old, asthma and hypertension E) Patient has open angle glaucoma Ans.

Which of the following is associated with aqueous humor? A. retina B. lens C. celiary muscle D. sweat glands E. tear duct Ans. C

Q. What dosage form of drug is preferred post cataract management? A. oral B. Injection C. Topical D. Sublingual E. OTIC Copyright © 2000-2012 TIPS Inc. Unauthorized reproduction of this manual is prohibited. This manual is being used during review sessions conducted by PharmacyPrep.

10-8

www.PharmacyPrep.com

Human anatomy

Ans. c An optometrist prescribed ciprofloxacin drops 2 gtt OD for 10 days. What is correct? A. Instill 2 drops in both eye for 10 days B. Instill 2 drops in affected eye for 10 days C. Take ciprofloxacin 500 mg daily for 10 days D. Instill 2 drops in right eye for 10 days Ans. D

A patient currently using dexamethasone eye drops daily for 3 to 4 wks after eye surgery. Another eye doctor prescribed tobramycin/dexamethasone 2 gtts for 10 days. What is pharmacist concern? A. tobramycin B. dexamethasone C. duration of therapy D. prescription from other doctor Ans. b

Q. What is the drug of choice for dental prophylaxis for a pt. having root canal treatment? A. amoxicillin Copyright © 2000-2012 TIPS Inc. Unauthorized reproduction of this manual is prohibited. This manual is being used during review sessions conducted by PharmacyPrep.

10-9

www.PharmacyPrep.com

Human anatomy

B. clindamycin C. azithromycin D. clarithromycin E. cephalexin ans. a

What is the drug of choice for dental prophylaxis for a pt. having root canal treatment. Pt. allergic to penicillin? A. amoxicillin B. clindamycin C. cloxacillin D. ciprofloxacin E. Cephalexin ans. B

Copyright © 2000-2012 TIPS Inc. Unauthorized reproduction of this manual is prohibited. This manual is 10-10 being used during review sessions conducted by PharmacyPrep.

PharmacyPrep.Com

Blood and Anemia PHARMACY PREP BLOOD AND ANEMIA

1. Microcytic anemia is characterized as? A. Hypochromic anemia B. Vitamin B 12 deficient anemia C. Folate deficient anemia D. Sickle cell anemia E. Decreased total iron binding capacity Ans: A 18) In microcytic anemia (iron deficient), all of the following decrease, except? A) Mean cell volume (MCV) B) Hematocrit (Hct) C) Hemoglobin D) Serum ferritin E) Total iron binding capacity (TIBC) Ans. E Tips. In microcytic anemia total iron binding capacity increase. Hematocrit is actual volume of RBCs in a unit volume of whole blood). 2. Megalobastic anemia is characterized as? A. Iron deficient anemia B. Vitamin B 12 or folic acid deficient anemia C. Folate deficient anemia D. Sickle cell anemia E. Pernicious anemia Ans: B 8. Which of the following is screening test is used to confirm iron deficient anemia? A. MCV B. Serum folate levels C. Serum ferritin levels D. RBC levels E. Transferrin level Ans. C Tips. The screening test for iron deficient anemia is a ferritin level. Iron deficient anemia is typically defined as ferritin level CO 2 + H20 l l (anaerobic) Lactate -----------------------> CO2 + H20 3. Linolenic acid is type of essential fatty acid? 9 15 12

A. omega 3

Alpha-Linolenic Acid (omega-3) B. omega 6 C. Omega 9

1 COOH

D. EPA E) DHA

4. All of the following can cause denaturation of proteins, except? A) acids B) heating C) bases D) mechanical mixing E) Buffers Ans. E 5. Gluconeogenesis is? Copyright

2000-2016 TIPS Inc

12-1

PharmacyPREP.COM

Biochemistry

A) formation of glucose from glycogen B) formation of glucose from proteins C) formation of glucose from fats D) formation of glucose from fats and proteins E) formation of glucose from maltose Ans. D 6. KP is a 35 year old women is using ferrous gluconate to treat anemia. What decreases absorption of ferrous gluconate? A) Food B) Proton pump inhibitors C) citrus juice D) Apple juice E) grapefruit juice Ans: A 7. What is primary protein structure? A. linear sequence of amino acids B. Alpha helix C. Beta pleated sheet D. Protein consisting more than one protein chain E. Three dimensional structure Ans. A 8. The protein three dimensional structure and linked with disulphide bonds is? A. primary B. secondary C. tertiary D. quaternary Ans. C 9.Which of the following protein structure describes more than one amino acid chain? A. primary B. secondary C. tertiary D. quaternary Ans. D 10. What aminoacid is precursor for synthesis of –CH2- in tetrapyrrole structure of porphyrin ring in hemoglobin. A) alanin B) Tyrosine C) Glycine

Copyright

2000-2016 TIPS Inc

12-2

PharmacyPREP.COM

Biochemistry

D) Phenyl alanin E) Acetyl CoA Ans. C 11. Heme containing enzymes are formed from protoporphyrin. What are the examples of heme containing enzymes or proteins? A) hemoglobin B) myoglobin C) cytochrome oxidase D) all of the above Ans. D 12. Which of the following is the rate limiting step in atherosclerosis formation? A. HMG-COA to mevalonate formation B. Mevalonate to cholesterol formation C. Low density lipoprotein formation' D. Lipid synthesis Ans. A

Copyright

2000-2016 TIPS Inc

12-3

www.Pharmacyprep.com

Nutrition

PHARMACY PREP NUTRITION 1) Choose correct answer about linolenic acid and linoleic acid: A) Linolenic acid is should be taken with diet. B) Linolenic acid and linoleic acid have only difference of double bond in chemical structure. C) Linolenic acid is omega 3 essential fatty acid D) Linoleic acid omega 6 essential fatty acid E) All of the above Ans. E Tips. Linolenic is omega 3 essential fatty acids. Omega 3 has three double bonds at position, 9, 12 and 15. Whereas omega 6 has two double bonds at position 9 and 12. 2. Rhodopsin is an eye pigmentation present in rods photoreceptor. Transform the following conversion of 11 retinal to produce night vision? A. 11-Cis-retinal to 11-trans retinal B. 11-Trans retinal to 11-cis retinal C. 13-Cis retinal to 13- trans retinal D. 13 trans retinal to trans retinal E. 12 cis retinoic acid to trans retinoic acid Ans: A 3. Which of the following vitamin should you recommend for smokers? A. Vitamin A B. Vitamin B C. Vitamin C D. Vitamin D E. Vitamin E Ans. C 4 What vitamin overdose could cause toxicity? A. Vitamin A B. Vitamin B C. Vitamin C D. Vitamin D E. Vitamin E Ans. A 5. A breast fed infant should recieved which of the following vitamin drops? A. Vitamin A drops B.Vitamin B 12 Copyright © 2000-2012 TIPS Inc. Unauthorized reproduction of this manual is prohibited. This manual is being used during review sessions conducted by PharmacyPrep.

14-1

www.Pharmacyprep.com

Nutrition

C.Vitamin C D.Vitamin D drops E. Vitamin E drops Ans. D 6. Age over 70 years and elderly commonly have deficiency of vitamin? A. Vitamin B1 B.Vitamin B 12 C. Vitamin B 6 D. Vitamin D E. Vitamin K Ans. B 7. A person is allergic to gluten, which of the following should not recommend? A. Milk B. Rice C. Wheat D. Wine E. Eggs Ans. C 8. What is the closes option to breast milk for infant at age of 3 mo? A. Formula milk B. Cow based formula milk C. Soy based formula milk D. Lactose free formula milk E. Cow milk Ans. B 9.Inadequate intake of vitamin D over 50 years age can cause? A. renal disease B. osteoporosis C. osteoarthritis D. hypercalcemia E. Hyper vitamin D syndrome Ans. B 10. Nephrolithiasis caused by? A. Deficiency of vitamin D B. Deficiency of Vitamin A C. Overdose of vitamin A D. Overdose of vitamin B 12 E. Overdose of vitamin D Ans. E Copyright © 2000-2012 TIPS Inc. Unauthorized reproduction of this manual is prohibited. This manual is being used during review sessions conducted by PharmacyPrep.

14-2

www.Pharmacyprep.com

Nutrition

11. What is the best source vitamin D A-Dairy product B. Milk product C. Dark green leafy vegetable D. Sunlight E. Sunscreen Ans.D

Copyright © 2000-2012 TIPS Inc. Unauthorized reproduction of this manual is prohibited. This manual is being used during review sessions conducted by PharmacyPrep.

14-3

PharmacyPrep.Com

Microbiology

PHARMACY PREP MICROBIOLOGY

1. Colon is large intestine which contains which of the following type of bacteria: A. 95% to100% anaerobic B. 95% to 100% aerobic C. 30 to 50% anaerobic D. 30 to 50% aerobic E. There is no bacteria in lower gut Ans . A 2. The most common causative organism of community acquired pneumonia (CAP) is: A. S. pneumonia B. M. pneumonia C. H. influenza D. S. aureus E. E. coli Ans. A 3. The following organism least likely causes pneumonia: A. S. pneumonia B. M. pneumonia C. H. influenza D. M. catharhalis E. E. coli Ans. E 4.Toxic shock syndrome (TSS) is caused by: A) E. coli B) Gonorrhea C) S. aureus D) Syphilis E) Chlamydia Ans. C 5. Which of the following NOT associated with Toxic shock syndrome? A. Tampons B. Condoms C. Contraceptive sponges D. Intra uterine devices (IUD) Copyright protected by Tips Inc

Protect copyright and prevent piracy

15-1

PharmacyPrep.Com

Microbiology

E. None of the above Ans: B 6. Annually flu season in Canada. Flu is caused by? A. Influenza A only B. Influenza B only C. Influenza A and B D. Influenza A and Flu vaccine E. Flu vaccine only Ans. C 7. Which of the following infection are commonly associated in nosocomial infections? I) S. aureus II) P. aeruginosa III) C. difficle A. I only B. III only C. I and II only D. II and III only E. All are correct Ans. E 8. What is true about cervical cancer? I) caused by oral contraceptives pills II) caused by papilloma virus III) It is sexually transmitted infection A. I only B. III only C. I and II only Ans: D

D. II and III only

E. All are correct

9. MK is a 27 year old man presents with persistent ear pain for last 2 days and associated with drainage. Pharmacist refer patient to physician because? A. otitis externa caused by P. aeruginosa and S. aureus B. Because it is associated with drainage C. Because ruptured tympanic membrane D. because ear pain with drainage could be perforation of tympanic membrane or drainage from middle ear. E. Because it can cause hearing loss Ans.D

Copyright protected by Tips Inc

Protect copyright and prevent piracy

15-2

PharmacyPrep.Com

Copyright protected by Tips Inc

Microbiology

Protect copyright and prevent piracy

15-3

www.Pharmacyprep.com

Cell and Molecular Biology PHARMACY PREP CELL AND MOLECULAR BIOLOGY

1. What is difference in nucleotide and nucleoside? A. phosphate B. Sugar C. base D. DNA Ans. A

E. RNA

2. Translation occurs: A) Before transcription B) After transcription C) Same time as transcription D) in mitochondria E) in chromosomes Ans. A 3. Hemophilia type A results from deficiency of clotting factor? A) factor 5 B) factor10 C) factor 8 D) factor 2 Ans. C

E) factor 9

4. A woman carrying hemophilia and has a child from a man with hemophilia. Which of the following is NOT her child? A. A girl with hemophilia B. A boy with hemophilia C. A girl having no gene of hemophilia D. A girl having the gene of hemophilia E. A boy having no hemophilia Ans. C

m w X X

X

Y

xx xx

xy xy

XX = Women with hemophilia XY = Man with hemophilia XY = Man with NO hemophilia XX = Women with hemophilia carrier 5. What is cDNA? A. DNA template produced by mRNA, is complementary DNA (cDNA) 16-1

www.Pharmacyprep.com

Cell and Molecular Biology

B. specific set of DNA and RNA C. specific set of DNA D. Specific set of DNA bases E. specific set of RNA bases Ans. A 6. Which of the following is NOT a complimentary base pair? A. G-C and A-T B. C-G and A-T C. G-C and T-A D.T-G and G-C E. A-U and G-C Ans. D 7. Information transfer from DNA to M-RNA is referred as? A. Transcription B. Translation C. DNA-gyrase D. protein synthesis E. DNA-Recombination Ans. A 8. Centrosomes? A) Centrosomes contains acid hydrolyzing enzyme B) Centrosomes are main organizing microtubule centre C) Centrosomes contain protein (rough) and lipid (smooth) synthesis d) Centrosomes are primary site for biological protein synthesis. E) Centrosomes are enzymes that produces and decomposes hydrogen peroxides. peroxisomes breakdown long chain fatty acids Ans. B Tips. Centrosomes are organizing microtubule centre Lysosome contains acid hydrolyzing enzyme. Endoplasmic reticulum contain protein (rough) and lipid (smooth) synthesis. Ribosomes are primary site for biological protein synthesis. Peroxisomes contains enzymes that produces and decomposes hydrogen peroxides. Peroxisomes breakdown long chain fatty acids. 9 Antisense RNA targets specific viral or microbial nucleic acid sequences that interferes with normal replication and expression. The antisense technology targets? A. Tcell B. B Cell C. DNA D. mRNA E. cDNA Ans. D 16-2

www.Pharmacyprep.com

Cell and Molecular Biology

10. Transduction is? A. the process by which DNA is transferred from one bacterium to another by a virus B. Microbial nucleic acid sequences that interferes with normal replication and expression C. woman carrying hemophilia and has a child from a man with hemophilia. D. Information transfer from DNA to M-RNA is referred Ans. A Tips: Transduction is the process by which DNA is transferred from one bacterium to another by a virus. It also refers to the process whereby foreign DNA is introduced into another cell via a viral vector. 11. Antisense technology is? A) Codons that stops proteins synthesis are used to stop formation of defected proteins. B) Antisense technology targets mRNA C) Antisense technology target DNA synthesis D) Antisense technology target translation in protein synthesis E) A and B Ans. A 12. Enzymes that catalyze coupling of two molecules are classified as A. Hydralases B. Ligases C. Oxidoreductases D. Transferases E. Isomerases Ans. B

16-3

PHARMACY PREP PHARMACOGENETICS 1) What is definition of Pharmacogenetics? A) The pharmacogenetics is integration of pharmacology and genetics. B) The study of pharmacogenetics allows designing and developing drugs that are customized to each person’s genetic mark up. C) The pharmacogenetics also utilized to study cytochrome enzymes that are responsible for drug interactions. D) The pharmacogenetics identifies the genetic variation that cause drug cause differences in drug response. E) All of the above Ans. E 2. What is SNP? A. Single nucleotide polymorph B. Single nucleotide pharmacogenetic C. Single genetic markup D. Single genetic variation E. Single nucleotide product Ans. A Tips. The SNP is single nucleotide polymorphism (SNP). It occurs when one base pair of nucleotide replaces another. A single base differences that exist between individual. This is the most common genetic variation in DNA. 3. If you know person genetics mark up, and target a medicine is defined as? A. pharmaceutics B. biotechnology C. Nanotechnology D. Pharmacogenetics E. Molecular biology Ans. D

Copyright © 2000-2016 TIPS Inc. Unauthorized reproduction of this manual is prohibited. This manual is being used during review sessions conducted by PharmacyPrep.

PharmacyPREP.Com

Immunology

PHARMACY PREP. IMMUNOLOGY and Immunizations 1. Which of the following is a pro-inflammatory cell responsible for initiation of an acute inflammation? A. Eosinophils B. Platelets C. Mast cells D. Basophiles E. Neutrophils Ans. C 2 Poison ivy is what type of hypersensitive reactions? A) Type I B) Type II C) Type III D) Type IV E) Type V Ans. E 3. A 70 yo elderly patient is treated with antibiotics for 7 days. However, still have signs of symptoms of infections such as fever. The patient neutrophil count is still increased over 70%. What kind of infections patient have? A. parasitic B. viral C. fungal D. inflammation E. bacterial Ans. E 4. All of the following can transmit HIV, except? A) Sexual contact B) Maternal transmission C) Drug abuse and sharing needle D) cut and wounds contact E) Orofecal Ans. E 5. A HIV patient currently has CD 4 T cell count is 300. What treatment doctor may target for prophylaxis? A. M. Tuberculosis B. Cytomegalvoirus C. Pneumocytisis pneumonia jerovicii or carinii D. Non-Hodgkins lymphoma Copyright © 2000-2016 TIPS Inc. Unauthorized reproduction of this manual is prohibited. This manual is being used during review sessions conducted by PharmacyPrep.

18-1

PharmacyPREP.Com

Immunology

E. Kaposis sarcoma Ans. A 6. What is true about immunoglobulin structure? A. consist of protein and amino acids sequence B. Consist of long chain amino acids and disulfide bond linkage C. Consist of long chain and small chain protein linked with disulfide bonds bridge D. Consist of specific amino acids selective sites of infections. E. Humoral antibody structures Ans. C 7. What is average life span of survival for HIV patient? A. 10 yrs B. 11 yrs C. 1-2 yrs D. 2-3 yrs E. 20-30 yrs Ans. D 8. A customer of your pharmacy travelling to mountain. She has history of anaphylaxis. Which of the following product pharmacist should recommend to take? A) Travelers diarrhea Dukoral vaccine B) Hepatitis A vaccine C) Flu vaccine D) DEET mosquito repellant E) Epipen Ans. 9.All of the following are NOT recommended for flu vaccine, except? A) Child under 6 months age B) A person allergy to eggs C) Person with currently acute bronchitis symptoms D) Pregnant E) Person with current symptoms of pneumonia Ans. D 10. Which of the following populations is LEAST considered high risk for influenza so that vaccination with influenza vaccine is not necessary? A) Person age > 5 yrs to 50 yrs B. Residents of long-term care facilities (e.g., nursing homes) C. Patients with diabetes mellitus D. Patients over the age of < 65 yo E. Patients with COPD or asthma Ans. A

Copyright © 2000-2016 TIPS Inc. Unauthorized reproduction of this manual is prohibited. This manual is being used during review sessions conducted by PharmacyPrep.

18-2

PharmacyPREP.Com

Immunology

11. Flu immunization season in Canada? A) November to April B) October to Mid November C) October to April D) October through December E) October through April Ans. B

Copyright © 2000-2016 TIPS Inc. Unauthorized reproduction of this manual is prohibited. This manual is being used during review sessions conducted by PharmacyPrep.

18-3

PharmacyPrep.Com

Biotechnology

PHARMACY PREP BIOTECHNOLOGY

1. Herceptin (Trastuzumab), is A. Treatment of HER2-overexpressing metastatic breast cancer B. Tumor necrosis factor alpha inhibitor C. Glycoprotein inhibitor D. Interleukin E. Interferon's Ans: A

6) Filgrastim is a granulocytes CSF indicated in the treatment of? A) renal, cardiac and hepatic graft rejection B) neutropenia associated with cancer chemotherapy C) thrombocytopenia associated with cancer chemotherapy D) anemia associated with chronic renal diseases E) chemotherapy induced anemia Ans. b

7. Filgrastim is? A. Colony stimulating factor B. Erythropoietin's C. Interferon's D. Human growth hormones E. Monoclonal antibodies Ans: A

Copyright © 2000-2012 TIPS Inc. Unauthorized reproduction of this manual is prohibited. This manual is being used during review sessions conducted by PharmacyPrep.

20-1

PharmacyPrep.Com

Biotechnology

8) Operlaveukin is? A) Interleukin 11 B) Colony stimulating factor C) Interleukin 3 D) Interleukin 2 E) Epoeitin Ans. A

9) Operlaveukin (IL-11) is indicated for? A) Chemotherapy induced thrombocytopenia B) Bone marrow suppression C) Acute graft rejection D) Neutropenia E) Growth factor Ans. A

10. Muromonab (OKT 3 ) is monoclonal antibody is used for treatment of ? A) Renal, cardiac and hepatic graft rejection B. Neutropenia associated with cancer chemotherapy C. Thrombocytopenia associated with cancer chemotherapy D. Anemia associated with chronic renal diseases E. None of the above Ans: A

HAMA is? A) B)

Human antigen and Mouse antibody Human antimouse antibody

Copyright © 2000-2012 TIPS Inc. Unauthorized reproduction of this manual is prohibited. This manual is being used during review sessions conducted by PharmacyPrep.

20-2

PharmacyPrep.Com

Biotechnology

Murine antibody D) Humanized antibody E) Human antibody Ans. B C)

Trastuzumab is produced by? A) Cell culture B) Monoclonal antibody method C) Chimeric monoclonal antibody D) murine monoclonal antibody E) Humanized monoclonal antibody Ans. E Tips. Transtuzumab is humanized MAB, which is produced from trangenic mice.

Chimeric antibody chain is? A) humanized chain linked to mouse chain. B) Humanized chain linked to rat chain C) Humanized chain binding with different antigen D) One quarter the antibody is from one species origin where as the other 3 quarters from other species Ans.

Mouse 100% mouse Example

drugs produced by murine antibody? Muromonab,

Chimeric 25% mouse

Humanized 10% mouse

Human 100% human

drugs produced by 5 chimeric antibodies? Rituximab, abciximab,

drug produced by 12 Humanized antibody? (transgenic) Trastuzumab,

drug produced by 4 human antibody? Adalimumab

Copyright © 2000-2012 TIPS Inc. Unauthorized reproduction of this manual is prohibited. This manual is being used during review sessions conducted by PharmacyPrep.

20-3

PharmacyPrep.Com capromab

Biotechnology infliximab, cetuximab, basiliximab.

omalizumab, daclizumab

Example of drug produced by 4 immunoadhesin (protein)? Etanercept, abatacept.

Copyright © 2000-2012 TIPS Inc. Unauthorized reproduction of this manual is prohibited. This manual is being used during review sessions conducted by PharmacyPrep.

20-4

www.Pharmacyprep.com

Toxicology

PHARMACY PREP TOXICOLOGY

2. Which of the following is a benzodiazepinics antagonist: A. Naloxone B. Physostigmine C. Flumazenil D. Naltrexone E. Penicillamine Ans. c Tips: Flumazenil is a benzodiazepinic antagonist therefore used as antidote in benzodiazepinic overdoses.

4) Carbon monoxide can be highly toxic because it easily binds to: A. Hemoglobin B. Myoglobin C. Cytochrome oxidase D. All are correct Ans. D

29) Specific antidote for iron preparations overdose include: A. Dimercaprol B. Deferoxamine C. Penicillamine D. Naloxone E. Sucimmer Ans. B

Copyright © 2000-2011 TIPS Inc. Unauthorized reproduction of this manual is prohibited. This manual is being used during review sessions conducted by PharmacyPrep.

21-1

www.Pharmacyprep.com

Toxicology

35) Which of the following is considered the warfarin antidote? A) Vitamin K B. Vitamin D C. Vitamin A D. Vitamin B 12 E. Vitamin E Ans. A

What is initial symptoms of acetaminophen overdose? A. vomiting B. severe nausea and vomiting C. bleeding D. Jaundice E. liver failure Ans. A Tips: acetaminophen large doses can cause toxicity. Initially for 2 to 4 hr no symptoms. Then toxic symptoms can occur in 4 stages: Stage 1: vomiting can occur Stage 2: after 24 hr nausea, vomiting can get severe Stage 3: can cause toxicity, symptoms bleeding, jaundice. Stage 4: can recover or if not recovered it can cause liver failure

42. Glutathione conjugation produce the product called? A. Mercapturic acid B. Sulfonation C. Oxidation D. Glutathione conjugation E. Reduction Ans. A

Copyright © 2000-2011 TIPS Inc. Unauthorized reproduction of this manual is prohibited. This manual is being used during review sessions conducted by PharmacyPrep.

21-2

www.Pharmacyprep.com

Toxicology

In hospital ward a nurse mistakenly administered high dose of heparin dose. Which of the following is used to treatment of overdose? A. Vitamin K oral B. Vitamin K injection C. Protamine sulphate D. Check INR Ans. C

A grandmother administered 2 tbsp tid of amoxicillin 300 mg/5 ml instead of 2 tsp tid to 9 mo old child. What to do? A) refer to emergence B) refer to doctor C) wait and watch D) Give antidote E) Give oral rehydration solution Ans. B Copyright © 2000-2011 TIPS Inc. Unauthorized reproduction of this manual is prohibited. This manual is being used during review sessions conducted by PharmacyPrep.

21-3

www.Pharmacyprep.com

Toxicology

At your pharmacy dispensed wrong medication to patient by a relief pharmacist. You as pharmacy manager, the next day you realized the dispensing error happened. What is appropriate action? A. Inform the doctor B. Inform college of pharmacy C. Inform the head office of your pharmacy store D. Contact patient immediately and inform about error and tell bring medication back to pharmacy. E. Call immediately to relief pharmacist and inform him that he did dispensing error. Ans. D

Patient brought wrong medication back to pharmacy. What is appropriate to do? A. Tell patient to make complain against pharmacist to college of pharmacy B. Change medication to correct medication and do not charge additional prescription fee. C. Ask patient if has taken medication and ask to continue if has no problems D. Change medication to correct medication and ask if patient if has taken medication and or any symptoms and document. E. Take back wrong medication from patient and ask to get a new prescription. Ans,D

What is appropriate action after any dispensing error? Copyright © 2000-2011 TIPS Inc. Unauthorized reproduction of this manual is prohibited. This manual is being used during review sessions conducted by PharmacyPrep.

21-4

www.Pharmacyprep.com

Toxicology

A. Complain to college of pharmacy B. document and discuss with pharmacy team member about error and prevention strategies. C. make incident report to ISMP D. Tell pt. that we will refund prescription fee Ans. b

47. B

BIBLIOGRAPHIC REFERENCE 1- COMPREHENSIVE PHARMACY REVIEW – Lippincott William & Wilkins – Fourth edition 2- CPS-COMPENDIUM OF PHARMACEUTICALS AND SPECIALITIES - Canadian Pharmacist Association – 2001 edition. 3- MEDICAL DICTIONARY – Dorland’s illustrated – 27th edition. 4- PHARMACY PREP – Lectures series & study guide for Evaluating Examination-TIPS 2003/2004 5- THERAPUTIC CHOICES – Canadian Pharmacist Association -Third edition

Copyright © 2000-2011 TIPS Inc. Unauthorized reproduction of this manual is prohibited. This manual is being used during review sessions conducted by PharmacyPrep.

21-5

www.Pharmacyprep.com

Toxicology

6- USP DI – Drug Information for the Health Care Professional–15th edition – Volume I.

Copyright © 2000-2011 TIPS Inc. Unauthorized reproduction of this manual is prohibited. This manual is being used during review sessions conducted by PharmacyPrep.

21-6

www.pharmacyprep.com

PHARMACY PREP. PHARMACOKINETICS

PHARMACOKINETIC FORMULAS Vd = D/Cp Cp = D/Vd D = Vd x Cp V d = CL T /K el t 1/2 = 0.693 /K el K el = 0.693/t 1/2

1) A dose of 240 mg was given to a patient, his total body clearance is 3.5 L/min and the drug excreted unchanged in the urine is 80 mg. What is his non-renal clearance? a) 240 ml/min b) 2.3 L/min c) 3.3 L/min d) 1.16 L/min e) 160 ml/min Ans. B Renal clearance is drug excreted unchanged or metabolized in urine. CL T = CL R +CL NR CL NR = CL T - CL R 240 mg................3.5 L 80 mg....................? =1.16 L = 3.5 - 1.16 = 2.3L or 3.5/240 mg = ?/80 mg

2. A drug hepatically metabolized and renally eliminates. Patient receiving this medication has chronic renal disease thus has low renal clearance. What is correct about drug affect? Copyright © 2000-2016 TIPS Inc. Unauthorized reproduction of this manual is prohibited. This manual is being used during review sessions conducted by PharmacyPrep.

1

www.pharmacyprep.com A) B) C) D) E)

metabolites excrete in stools metabolites accumulates in blood metabolites half life decrease hepatic metabolism decrease hepatic metabolism saturates

Ans. B 3.A dose was given, the plasma concentration was 64 mg, t 1/2 =0.7 hrs. After 7 hours from the initial dose, what is the drug's concentration? a) 0.0mg b) 1mg c) 2mg d) 3mg e) 4mg Ans. A Tips. 64 --> 32 --> 16 -->8-->4 -->2-->1-->0.5 -->0.25 -->0.125-->0.06 or C = C o . e-kt K = 0.693/t 1/2 K = 0.693/0.7 = 0.999 C = 64 x e(-0.999x7) = 0.06 or ln C = -kt + ln C o How to use calculator (Sharp EL-510RN) Enter 64 Multiply Enter 2ndF Enter ex Enter ( Enter (-) Enter Enter ) Enter =

Copyright © 2000-2016 TIPS Inc. Unauthorized reproduction of this manual is prohibited. This manual is being used during review sessions conducted by PharmacyPrep.

2

www.pharmacyprep.com 4. A 40 year old patient who weighs 70 kg, needs intravenous infusion of amoxicillin. The desired C ss of drug is 15mg/dL. The physician ordered antibiotic infused for 10 hours. Amoxicillin has t 1/2 = 1 hour and Vd = 9L. What rate of iv is recommended for this patient? (no loading dose was given); A. 135 mg/hr B.936 mg/hr C. 1000 mg/hr D. 333 mg/hr E. 400 mg/hr Ans. B Tips: C ss = R___ K x Vd R = Rate of infusion K = first order elimination constant V d = volume of distribution R = C ss x k x V d R = 15 0.693 x 9000 = 100 1 hour = 935.54 mg/hr 5) If the rate of infusion of the drug is 500 mg q8h and clearance is 7.3 L/hr. Find steady state concentration a) 85.6 mg/L b) 856 mg/L c) 8.56 g/L d) 8.56 mg/L e) 68.4 mg/L C ss = rate of infusion/clearance Ans. D

Css = R/CL CL = R/Css R = Css x CL CL = K x Vd

C ss = Rate of infusion = 500/8 = 8.56 mg/L Clearance 7.3

A patient parameter include elimination rate contant 0.17 h-1 and volume of distribution 11.8 L. What is clearance? 2.02 L/h

Copyright © 2000-2016 TIPS Inc. Unauthorized reproduction of this manual is prohibited. This manual is being used during review sessions conducted by PharmacyPrep.

3

www.pharmacyprep.com 6. For an effective antimicrobial to treat UTI it must be all, except? A) low level of protein binding B) short biological half life thus can reach high concentration rapidly C) high distribution in body D) high distribution in urinary tract E) highly active against gram negative bacteria Ans. D 7. A drug was found to have zero order kinetic 100 mg after 6 days long it will take to eliminate the entire drug from the body? a) 100 days b) 200 days c) 300 days d) 600 days e) 900 days Ans. d

99 mg. How

8. A patient was given 100 mg drug orally. The drug absorbs at rate of 10 mg/min and follows zero order kinetics. After 5 min how much drug will be absorbed? A. 10 mg B. 50 mg C. 75 mg D. 100 mg Ans. b 9. A patient was given 100 mg drug orally. The drug absorbs at rate of 10 mg/min and follows zero order kinetics. How long it takes to absorb all the drug? A. 10 min B. 50 min C. 75 min D. 100 min Ans. A 10. Ampicillin in 5% dextrose degrades by first order kinetic, at rate constant of 0.026 h. What is the shelf life of ampicillin? a) 4 hr b) 8hr c) 2hr d) 16hr e) 12hr Tips. T 90 = 0.105/K T 90 = 0.105/0.026 = 4hrs

Copyright © 2000-2016 TIPS Inc. Unauthorized reproduction of this manual is prohibited. This manual is being used during review sessions conducted by PharmacyPrep.

4

www.pharmacyprep.com 11. Approximately 50% of cloxacillin is excreted unchanged in the urine. If the normal dosage schedule for cloxacillin is 125 mg q6h, a patient with renal function 20% of normal should receive? a) 25 mg q6h b) 31.25 mg q6h c) 62.5 mg q6h d) 75 mg q6h e) 125 mg q12h Tips:d Renal Cloxacillin 50% 62.5

Non renal 50%

mg

62.5 mg

12.5 mg

62.5 mg

or

50-20 = 30% 50% .............125 30%................?

12. Drug which dose is 1500 mg is given every 24 hours. The renal clearance of this drug is 1.2 mg/dL. Calculate the clearance in ml/min. a) 98.6 ml/min b) 76.8 ml/min c) 86.8 ml/min d) 66.8 ml/min e) 43.6 ml/min Ans. C Tips 1.2 mg...........100 mL 1500 mg..........? = 125,000 mL for Q24h (1440 min) 125,000............1440 min ? mL.........................min = 86.8 ml/min or 1.2 mg --------100 ml

125000 mls--------1440 min (60 min x 24 hrs)

Copyright © 2000-2016 TIPS Inc. Unauthorized reproduction of this manual is prohibited. This manual is being used during review sessions conducted by PharmacyPrep.

5

www.pharmacyprep.com 1500 mg---------X X = 125000 mls

X---------------1 min X = 86.8 ml/min

or 125000 ml/1440 min = 86.6 ml/min 13. If a drug is 50% metabolized and half life drugs is 1 hr . What is the percentage of blood concentration after 4hrs? a) 50% b) 25% c) 12.5% d) 6.25% e) 3.12% Ans. D Tips: 100 50% --> 25 -->12.5 --> 6.25%--> or Drug concentration = 100% t 1/2 (decrease the drug by half) = 50% 2 t1/2 t1/2 t1/2 = 6.25% 14. A new antifungal was given at a dose of 5mg/Kg by a single intravenous bolus injection to a 32 years old female who weighed 75Kg. The antifungal has an elimination half life of 2 hours and apparent volume of distribution of 0.28L/Kg. What is the initial plasma drug concentration in this patient? a) 36 mg/L b) 1.8 mg/L c) 17.8 mg/L d) 1.79 mg/L e) 19 mg/L Ans. C C p = D o C p = 5 mg/Kg = 17.8mg/L Vd 0.28L/kg 15. Following the anterior information, calculate the predicted plasma concentration at 8 hours after the dose: a) 15mg/L b) 111mg/L c) 11.1mg/L d) 1.11mg/L e) 2.64mg/L Ans. D Cp initial = 17.9mg/L and elimination t 1/2 is 2hours Cp = 17.9 after 2 hours = 8.95 Cp = 8.95 after 2 hours = 4.47 Cp = 4.47 after 2 hours = 2.23 Copyright © 2000-2016 TIPS Inc. Unauthorized reproduction of this manual is prohibited. This manual is being used during review sessions conducted by PharmacyPrep.

6

www.pharmacyprep.com Cp = 2.23 after 2 hours = 1.11 IN 8 HOURS

16. Following the same anterior information, calculate how much drug remains in the patient body, 8 hours after the administrated dose: a) 23mg b) 112mg c) 100mg d) 15.3mg e) 84.4mg Ans. A

D = 375mg after t/1/2 of 2 hours = 187.5mg after t1/2 of 2 hours = 93.75..Continue the same as the anterior question!!! After 8 hours you will have 23mg 17. Following the anterior information, how long after the dose is exactly 75% eliminated from the patient body? a) 2 hours b) 4 hours c) 6 hours d) 8 hours e) 10 hours Ans. B Initial dose = 375mg and elimination t1/2 is 2hours Cp = 375mg after rs a er rs 4 hours → 75% Let us assume the following characteristics of the drug in a 70kg man. -Therapeutic effective plasma concentration: 2.0mg/L -Biologic half-life: 80minutes -Apparent volume of distribution: 0.70Kg/L 18. What is the recommended rate of infusion for this drug? a) 85mg/min b) 58mg/min c) 0.85mg/min d) 8.5mg/min e) 0.085mg/min Ansss K = 0.693/80min = 0.0087 Vd = 0.70 x 70Kg = 49L

n

Copyright © 2000-2016 TIPS Inc. Unauthorized reproduction of this manual is prohibited. This manual is being used during review sessions conducted by PharmacyPrep.

7

www.pharmacyprep.com

19. What is the total body clearance for this patient? a) 33L/min b) 43mls/min c) 43L/min d) 0.43L/min e) 0.43mls/min Ans. D CL T T = 0.0087 x 49 = 0.43L/min 20. Still following the information of the anterior questions, calculate the corresponding loading dose: a) 98mg b) 160mg c) 48mg d) 24mg e) 100mg 16- A LD = V ss

21. Drug A bind to 28% with protein, when given at dose 150 mg/day. If given to 300 mg/day, what % of drug A bind with protein. Mol.wt of protein is more than drug A. A-56% B-28% C-14% D-80% E-60% Ans-B

22. Two drugs that metabolized by the kidney and liver. Which one has first pass metabolism? drug x drug y CL t 1100 460 CL r 110 20 Vd 3.3 2 ProteinBinding 85% 90% A) Drug x = oral B) drug y = oral C) drug y = rectal D. drug x = rectal Ans. B

Copyright © 2000-2016 TIPS Inc. Unauthorized reproduction of this manual is prohibited. This manual is being used during review sessions conducted by PharmacyPrep.

8

www.pharmacyprep.com 23. Mr. Florence Backs is a 72-year-old patient weighing 71.8 kg who is taking aminophylline 250mg by i.v. at stat. and then was given immediately by i.v. drip at a rate of 40mg/hour. Theophylline level at stat = 7.5mg/ml and V d of theophylline is 0.5L/kg. What is the expected serum concentration after the initial dose? a. 5.55mcg/ml b. 13.1mcg/ml c. 7.5mcg/ml d. 6.8mcg/ml Ans-B What is the expected serum concentration after the initial dose? Co = Dose x F x S Vd = 250 x 1 x 0.8 (0.5) (71.8 kg)

= 5.55 ug/ml

5.55 + 7.5 = 13.05 = 13.1 ug/ml

24. During gentamycin therapy which parameter would you measure? a. Creatinine clearance for renal toxicity (nephrotoxicity) b. serum creatinine, BUN, blood concentration c. auditory function because of ototoxicity d. all of the above Ans: D 25. What is correct about drug excretion in to breast milk? A. acidic drug excrete more than base drugs B. base drug excrete more than acidic drugs C. neutral drug excrete more than basic drugs D. drug excretion in to breast milk does not depends drug pH Ans. B 26. Absolute bioavailability is the result of comparing a. oral dosage from and IV dosage form b. oral bioavailability from and sublingual form c. oral bioavailability from and IM dosage form d. oral bioavailability from and rectal form Ans: A 27. What is the F value for an experimental drug tablet based on the following data? Copyright © 2000-2016 TIPS Inc. Unauthorized reproduction of this manual is prohibited. This manual is being used during review sessions conducted by PharmacyPrep.

9

www.pharmacyprep.com

Form Tablet Solution (control) Injection (control) a) 0.25 b) 0.38 c) 0.50 d) 0.66 e) 0.90 Ans. A

Drug Dose Dose 100 mg po 100 mg po 50 mg IV push

AUC (µg/ml/mL/h) 20 30 40

28. If an oral capsule formulation of the drug A products a serum concentration-time curve having the same area under the curve as that produced by an equivalent dose of drug A given IV, it can generally be concluded that a) the IV route is preferred to the oral route b) the capsule formulation is essentially completely absorbed c) the drug is very rapidly absorbed d) all oral dosage forms of drug A will be bioequivalent e) there is not advantage to the IV route Ans. b Tips; if the AUC are same, it can be said that the total amount of drug delivered to the body by each dose form is equal. For oral dosage for 100% bioavailability does not mean that the drug has 100% absorption.

Product Company

Dosage Form

A A B B

Parenteral Injection Tablet Tablet Capsule

Dose Administered 10 mg IV 20 mg po 20 mg po 15 mg po

Cumulative Urinary Amount (mg) 9.4 12.0 8.2 6.8

29. Using the above data the absolute bioavailability of Company B tablets is best estimated to be a) b) c) d)

25% 40% 44% 68%

Copyright © 2000-2016 TIPS Inc. Unauthorized reproduction of this manual is prohibited. This manual is being used during review sessions conducted by PharmacyPrep.

10

www.pharmacyprep.com e) 87% Ans. C Tips: The best measure of absolute bioavailability is considered to be AUC of iv. Because AUC iv is not available for this drug, the next best comparison will be the cumulative drug amount found in the urine. Because iv dose was 10 mg, whereas the oral tablet dose was 20mg, a correction factor of 2 x is needed. Cumulative amount is 20 mg injected. 9.4 x 2 = 18.8 8.2/18.8 = 0.44 or 44% 30. Patient on phenytoin therapy. Therapeutic level is 10 – 20 mg/L. Blood level of phenytoin is within therapeutic range. But still patient shows some side effects of phenytoin Following is blood phenytoin analysis? 13.6 mg/L morning 14.2 mg/L evening What is reason increase level of phenytoin A-Phenytoin toxicity B-Phenytoin has very strong protein binding C-Liberation of drug from protein binding D-None of the above Ans-C 31. If blood flow to liver decrease? A-Affects intrinsic metabolism B-Reduce protein binding of drug C-Increase protein binding of drug D-Reduce metabolism of drugs E-Do not change metabolism of drug Ans. D 32. A drug completely renally eliminates, it depends on all pharmacokinetic factors, except? A. drug half life B. Volume of distribution C. Hepatic clearance D. Renal function E. rate of infusion Ans. C 33. Steady state concentration is factor of a. AUC B. half life C. plasma protein binding D. dose

Ans. B Copyright © 2000-2016 TIPS Inc. Unauthorized reproduction of this manual is prohibited. This manual is being used during review sessions conducted by PharmacyPrep.

11

www.pharmacyprep.com

PHARMACY PREP. RATES AND ORDERS OF REACTIONS Plot of log of concentration against time produces a straight line with a: A. Slope of –K/2.303 B. Slope of –K/t 1/2 C. Slope of –K o D. slope of K/2.303 E. slope of K/0.693 Ans. A 2. The time it takes to infused drug to reach plasma steady state concentration (Css) depends on: I-Elimination half-life of drug. II-Fraction of free excreted in urine III-Plasma concentration curve A. I only B. III only C. I and II only D. II and III only E. All of the above Ans. a 3. A drug with zero order reaction eliminating, which is independent of initial concentration, What is correct about slope? A. k o B. k/2.303 C. 0.693/k D. 1/ka E. None Ans. A 4. Use of loading dose? I-To achieve therapeutic concentration immediately II-Loading dose = desired concentration x V d III-Steady state will be achieved immediately and maintained A. I only B. III only C. I and II only D. II and III only E. All of the above Ans. E 5. Time to reach steady state is determined by the? A) Elimination half life B) Loading dose C) Maintenance dose D) The absorption rate E) dose interval ans. A Copyright © 2000-2016 TIPS Inc. Unauthorized reproduction of this manual is prohibited. This manual is being used during review sessions conducted by PharmacyPrep.

24

www.pharmacyprep.com

6.A drug completely renally eliminated. The patient has renal failure. This drug is hepatic metabolized. What will happen to drug? A. Drug metabolite accumulates in blood B. Drug is completely in metabolized in liver C. Drug is completely eliminate renally D. Drug is completely protein bound E. Drug have linear kinetic Ans. A 7. What is correct about a drug with first order elimination? A) drug elimination depends on initial concentration B) drug elimination depends on time only C) drug elimination depends and concentration and time D) drug elimination depends renal and hepatic function E) drug elimination depends on renal function only Ans. B 8. What is correct about a drug with zero order elimination? A) drug elimination depends on initial concentration B) drug elimination depends on time only C) drug elimination does NOT depends and concentration and time D) drug elimination depends renal and hepatic function E) drug elimination depends on renal function only Ans. b

Copyright © 2000-2016 TIPS Inc. Unauthorized reproduction of this manual is prohibited. This manual is being used during review sessions conducted by PharmacyPrep.

25

www.pharmacyprep.com

Copyright © 2000-2016 TIPS Inc. Unauthorized reproduction of this manual is prohibited. This manual is being used during review sessions conducted by PharmacyPrep.

26

www.pharmacyprep.com

PHARMACY PREP PHARMACODYNAMICS 1. Competitive (reversible) antagonism Ag (agonist) + E + Ag*(antagonist) AgE + Ag*E If we use more concentration of Ag what happen? A. Concentration of AgE increase and Ag*E decrease B. Concentration of Ag*E increase and AgE decrease C. Concentration of AgE and Ag*E decrease D. Concentration of AgE and Ag*E increase E) No change in AgE and Ag*E rate Ans. A Competitive (equilibrium) Competitive inhibitors are molecules that bind to the same site as the substrate preventing the substrate from binding as they do so but are not changed by the enzyme. Parallel rightward shift of agonist dose response curve shift. Q. Intensity of response depends on concentration of both agonist and antagonist. The same maximal response is attained by increasing the dose of agonist

Non competitive (irreversible) Non competitive inhibitors are molecules that bind to some other site on the enzyme reducing its catalytic power.

Flattening of dose response curve Q. Response depends on only on the concentration of antagonist Maximal response is suppressed

2. The k m value of an enzyme is numerically equal to, in first order kinetics A. Half the maximum velocity (V max ) expressed in moles/liter B. Velocity of a reaction divided by substrate concentration C. Substrate concentration in moles/liter necessary to achieve half the maximum velocity of a reaction D. Maximum velocity divided by half the substrate concentration in moles necessary to achieve maximum velocity E. Substrate concentration divided by velocity of a reaction Ans. C First order Km = 1/2 V max Zero order Km = V max

27-1

Pharmacyprep.com

Medicinal Chemistry

PHARMACY PREP. MEDICAL CHEMISTRY

1. Which of the following is the molecular structure of isotretinoin, used in oral treatment of acne? A. 13-CIS Retinoic acid B. 13-TRANS Retinoic acid C. 13-CIS-TRANS Retinoic acid D. Retinoic acid E. None of the above 2. What functional group is responsible for sulfa allergy A) Sulfonamide B) Sulfide C) Sulfhydril group D) Proline ring E) Carbamate group Ans. A 3.Fexofenadine, how many optical isomers are possible? H3C

H3C CH3 CH3

HO

N

HO

OH CH3

N OH

OH

Terfinadine

A) 1 B) 2 Ans.B

O

Fexofenadine

C-3

D) 4

E) None

4. If a drug chemical structure has 3 chiral carbons. How many optical isomers are possible? A) 2 B) 4 C) 8 D) 9 E) 12 Ans. C 2n = 22 =2x2=4 n= number of chiral centers 23 = 2 x 2 x 2 = 8

Copyright © 2000-2016 TIPS Inc. Unauthorized reproduction of this manual is prohibited. This manual is being used during review sessions conducted by PharmacyPrep.

28-1

Pharmacyprep.com

Medicinal Chemistry

Copyright © 2000-2016 TIPS Inc. Unauthorized reproduction of this manual is prohibited. This manual is being used during review sessions conducted by PharmacyPrep.

28-2

PharmacyPREP.Com

Medicinal Chemistry of autonomic nervous system

Pharmacy Prep MEDICINAL CHEMISTRY OF AUTONOMIC NERVOUS SYSTEM 1.What is incorrect about structure activity of acetylcholine agonist? A) acetylcholine dissociate into acetyl and choline by acetylcholinesterase enzyme B) methanocoline has one extra methyl group than Ach C) Carbacholine has one carbmate group D) Bethanacholine has one carbamate and methyl group than Ach E) acetylcholine is the major neurotransmitter of sympathetic system Ans: E 2.What is incorrect in catecholamine biosynthesis? A) tyrosine is the precursor B) dopamine produce norepinephrine hydroxylation C) epinephrine produce norepinephrine by methylation D) levodopa catalyzed by dopadecarboxylase to dopamine E) norepinephrine catalyzed by methyl transferase to epinephrine Ans: C 3)The difference between dopamine and amphetamine? A) one methyl group B) one hydroxyl group C) 2 hydroxyl group and 1 methyl group D) 2 methyl group and 1 hydroxyl group E) One methyl and one hydroxyl group NH2

HO

Dopamine OH

OH CH3

NH2

N

NH2

CH3

H

OH CH3

N

CH3

Amphetamine

Dextroamphetamine

Ephedrine

A

B

C

H

H CH3

N

CH3 Pseudoephedrine

D

CH3

CH3 Methamphetamine

E

4.Difference between pseudoephedrine and methyl amphetamine is? A) one methyl group B) one hydroxyl group C) 2 hydroxyl group and 1 methyl group D) 2 methyl group and 1 hydroxyl group Copyright © 2000-201 TIPS Inc. Unauthorized reproduction of this manual is prohibited. This manual is being used during review sessions conducted by PharmacyPrep.

24-1

PharmacyPREP.Com

Medicinal Chemistry of autonomic nervous system

E) One methyl and one hydroxyl group Ans. b 5.What chemical reaction converts pseudoephedrine to crystal meth? A) Hydroxylation B) dehydroxylation C) Hydrolysis D) methylation E) Glucuronidation Ans. B 6.Beta blockade effect can be reversed by? A. metoprolol B. salbutamol C. prazosin D. propranolol E. pseudoephedrine ans. B Tips: beta blockade can be reversed by beta agonist by displacement from occupied receptors if large amount of agonist is given because all beta blockers are competitive antagonist.

Copyright © 2000-201 TIPS Inc. Unauthorized reproduction of this manual is prohibited. This manual is being used during review sessions conducted by PharmacyPrep.

24-2

PharmacyPrep.Com

Medicinal Chemistry of autocoids and analgesics PHARMACY PREP MEDICINAL CHEMISTRY OF AUTOCOIDS AND ANALGESICS

1 . The following groups are essential for antihistaminic pharmacological activity: I-Aromatic or heteroaromatic rings in structure II-Tertiary amines in structure III-Chain of alkyl group in structure A. I only B. III only C. I and II only D. II and III only E. I, II, III Ans. E 2. Ecosonides are? A. Leukotrienes B. Prostaglandins C. serotonins D.Histamines Ans. b 3.Ondansetron is? A. Serotonin type 1 antagonist B. Serotonin type 2 antagonist C. Serotonin type 3 antagonist D. Serotonin type 3 agonist Ans. C 4.Which of the following is an end product of glutathione conjugation in acetaminophen metabolism? A. N-benzoquinineamine B. Mercapturic acid C. Glutathione D. cysteine E. Glycine ans. B 5.Glutathione is? A. cysteine, glycine and lysine B. cysteine, glycine and tryptophan C. cysteine, glycine and glutamate D. Cysteine, glycine and isoleucine Ans. C Copyright © 2000-2016 TIPS Inc. Unauthorized reproduction of this manual is prohibited. This manual is being 25-1 used during review sessions conducted by PharmacyPrep.

PharmacyPrep.Com

Medicinal Chemistry of autocoids and analgesics

6.) The following structure is prostaglandin analogue of? O

HO

9

COOH

11

A) Prostaglandin E B) Prostaglandin F C) Prostaglandin G D) Prostaglandin H E) Prostaglandin I Ans. A 7.Morphine analgesic action is due to mu receptors by hydrogen bonding of? A) Endorphine peptides B) Enkephalin peptides C) Leukotrienes D) prostaglandins E) histamines Ans. B 8.Morphine analgesic actions is mainly produced due to? A. mu, kappa, and delta receptors B. mu, and kappa agonist C. mu agonist D. mu, and delta agonist E. kappa and delta agonist ans. C 9.Naloxone is an opioid antagonist and blocks the actions of? A. mu, kappa, and delta receptors B. mu, and kappa receptors C. mu receptors D. mu, and delta receptors E. kappa and delta receptors Ans. C 10.Synthetic estrogen like diethyl stilbenes gives? A) enantiomers B) diastereomers C) geometric isomers D) optical isomers E) confirmers Ans. c Copyright © 2000-2016 TIPS Inc. Unauthorized reproduction of this manual is prohibited. This manual is being 25-2 used during review sessions conducted by PharmacyPrep.

www.pharmacyprep.com

PHARMACY PREP. MEDICINAL CHEMISTRY AND PHARMACOLOGY OF CARDIOVASCULAR SYSTEM 1. What is incorrect about site of action of diuretics? A) carbonic anhydrase and osmotic diuretics like acetazolamide act on proximal tubule B) loop diuretics act on loop of henle C) Thiazides act on distal convoluted tubule D) Potassium sparing act on collecting duct E) Potassium sparing diuretic act on proximal tubule Ans. E 2. Diuretics effect on acid and base balance of blood. What is incorrect? A) thiazides and loop cause metabolic alkalosis B) acetazolamide cause hyperchloremic metabolic acidosis C) spironolactone cause intracellular alkalosis D) acetazolamide decrease intraocular pressure E) Furosemide may cause metabolic acidosis Ans. E 3.) Diuretic characterized for A. Acetazolamide B. Spironolactone C. Hydrochlorothiazide D. Furosemide E. Triamterene Ans. D

e

a

an

a

4.. Imidazoline is a molecular structure that contains a heterocyclic ring and belongs the molecular structure of: A. Prazosin B. Nifedipine C. Atropine D. Clonidine E. Allopurinol Ans. D 5. Which of the following cardiovascular medications chemically classified as glycoside? A. Digoxin B. Warfarin C. Nitrates D. Nitrites E. Cholestyramine Ans. A Copyright © 2000-2012 TIPS Inc. Unauthorized reproduction of this manual is prohibited. This manual is being used during review sessions conducted by PharmacyPrep.

31-1

www.pharmacyprep.com

6. Nitroglycerine is classified as? A. Nitrites B. Nitrates C. Nitriles Ans. B

D. Nitrol

E. None of the above

7) What functional group is responsible for metallic taste or taste changes in captopril? A) Sulfonamide B) Sulfide C) Sulfhydril group D) Proline ring E) Carbamate group Ans. A 8) Pyridine is a mono heterocyclic compound very important in the activity of many eren r s e ec ar s r c re -dihydropyridine is found in: A. Verapamil B. Digoxin C. Clonidine D. Nifedipine E. Diltiazem Ans. D 9. Thrombolytics (fibrinolytic) agents are contraindicated in? a. Myocardial infarction B. Ischemic stroke C. Bleeding from the GIT D. Pulmonary embolism E. Deep vein thrombosis ans. C 10 ce a a e a car n c an rase re c an r c r a e a a e diuretic have an identical molecular group very important in their structure knows as: A. Sulfonamide group B. Pyrimidine group C. Purine group D. Pyrrolidine group E. Ether group Ans. A

Copyright © 2000-2012 TIPS Inc. Unauthorized reproduction of this manual is prohibited. This manual is being used during review sessions conducted by PharmacyPrep.

31-2

www.pharmacyprep.com 11. Which of the following statements regarding HEPARIN is/are right I. It is a muc sacar e r an c ac resen na ra n an ss es es ec a lungs and liver. II. It has anticoagulant proprieties that inhibit the conversion of prothrombim to thrombin and fibrinogen to fibrin III. It has antithrombin III as it co-factor A. I only B. III only C. I and II only D. II and III only E. All are correct

n

12. What class of drug represents the following chemical structure?

O HS - H2C

N

CH3

CO2H

A) Diuretics B) Angiotensin converting enzyme (ACE) Inhibitors C) Calcium channel blockers D) Selective Serotonin Receptor Inhibitors E) Angiotensin Receptor Blockers (ARBs) Ans. B 13) What is incorrect about ACE Inhibitors? A) sulfhydril group is present in captopril B) captopril and fosinopril are NOT prodrugs because these do not require hydrolysis to produce active products C) it essential to have esterase for the activation of ACEi. In re en r a n an ens n II E) Angiotensin II is strong vasodilator Ans. E Tips: Angiotensin II is strong vasoconstrictor 14 sa an s s n ra r a rea reports cough. What is incorrect? A) recommend to see doctor to change to ARBs B) recommend cough suppressant dextromethorphan C) ARBs have no cough SEs c s n ce ra n ns n E) CCBs have no cough side effects Ans: B

er ens n recen

Copyright © 2000-2012 TIPS Inc. Unauthorized reproduction of this manual is prohibited. This manual is being used during review sessions conducted by PharmacyPrep.

31-3

www.pharmacyprep.com 15) What is active pharmacophore in Angiotensin receptor blockers? N Imidazole ring (essential)

Cl

N OH

Tetrazole ring N N N

N H Losartan

A) Tetrahydropyrazole ring B) On position 2 ethyl alcohol on imidazole ring C) Two ring structure D) Alkyl long chain on imidazole ring E) AT 1 receptors Ans. B 16) HMG Co A reductase inhibitor like statins effective lowers LDL. What is incorrect? -dihydroxycarboxylate is essential for SARs B) Myopathy SE is e ass c a e s a ns s s n re -MM C) Initial SEs include stomach upset like diarrhea a ns ncrease er en es s n r s E) Statins combined with niacin decrease the risk of myopathy Ans: E 17 sa ear a e patient has been prescribed amlodipine 10 mg daily. After 2 ee s n a n s a en re r s re en n ee ea ac e s n r an a a n ar ac s s s ec ese are s e e ec a ne c of the following is not side effects of CCBs? A) ankle edema B) flushing C) headache D) palpitation E) dry mouth Ans. E

Copyright © 2000-2012 TIPS Inc. Unauthorized reproduction of this manual is prohibited. This manual is being used during review sessions conducted by PharmacyPrep.

31-4

www.pharmacyprep.com 18) Which of the following drugs represent the structure below?

A. Leukotriene antagonists B. Beta blockers C. Coumarins D. Calcium channel blockers E. Benzodiazepines Ans: D

19 n na rec a e e erc se r e A. Propranolol B. Nitroglycerine SL C. Diltiazem D. Nifedipine Ans-B Tips: nitroglycerine SL or spray

na s ress r

c

ce

20) Clopidogrel is classified as an: A) Anti-hypertensive. B. Inhibitor of platelets' aggregation. C. Anti-diabetic. D. Anti-Parkinson’s. Ans: B Tips: Clopidogrel (Plavix) is a blood thinning agent. Agents of this class generally act by inhibiting aggregation of platelets which is thought to be the primary mechanism of blood clotting. It is indicated for prevention of heart stroke in patients with CHF. 21.Which of the following oral anticoagulant is safe in renal disease? A. Apixaban B. Heparin C. Dalteparin D. Warfarin E. Dabigatran Ans.D

Copyright © 2000-2012 TIPS Inc. Unauthorized reproduction of this manual is prohibited. This manual is being used during review sessions conducted by PharmacyPrep.

31-5

www.pharmacyprep.com 22.Factor Xa selective inhibitor is? A. Dabigatran B. Warfarin C. Tinazaparin D. Apixaban E.Heparin Ans. D 23.Nitrates produce smooth muscle vascular relaxation by producing? A. Nitric acid B. Nitric oxide C. Nitrates D. Vasodilator ans. B

Copyright © 2000-2012 TIPS Inc. Unauthorized reproduction of this manual is prohibited. This manual is being used during review sessions conducted by PharmacyPrep.

31-6

PHARMACY PREP MEDICINAL CHEMISTRY AND PHARMACOLOGY OF PYCHIATRIC AND NEUROLOGICAL DISORDERS 1. What are the physiological function of serotonin? A) mood B) Sleep C) platelet aggregation D. weight E. All of the above Ans. E 2.What is NOT a side effect of SSRIs? A) GI bleeding B) insomnia C) nausea and vomiting D) sexual dysfunction E) anticholinergic SEs Ans: E 3.What is incorrect about SSRIs? A) if combine with MAOi, can cause serotonin syndrome B) fluoxetine has long half life, thus have no serotonin syndrome C) Abrupt discontinuation of SSRIs can cause withdrawal symptoms D) Do not combine SSRIs with other SSRIs E) SSRIs require washout period Ans. B 4. Which of the following SSRI least likely associated with withdrawal symptoms? A. Trazodone B. Amitriptyline C. Phenelzine D. Fluoxetine E. Mirtazapine Ans. d

5.Topiramate can be used for all, except? A. seizures B. migraine prophylaxis C. weight loss D. dementia E. bipolar disorders ans. D

Copyright © 2000-2016 TIPS Inc. Unauthorized reproduction of this manual is prohibited. This manual is being used during review sessions conducted by PharmacyPrep.

27-1

6.MP is 20 yo man using lithium carbonate 400 mg QID for the treatment of manic depression. Patient presents with tremors. All of the following drugs increase lithium toxicity by increasing lithium serum concentration, except A. Ramipril B. Naproxen C. Caffeine D. Hydrochlorothiazide Ans. C 7. Which of the following drug combination least likely cause serotonin syndrome? A. Phenelzine and fluoxetine B. Phenelzine and amitriptyline C. Phenelzine and Tranylcypromine d. Bupropion and trazodone E. Selegiline and tramadol Ans. e Tips. bupropion and trazodone have weak affect on serotonins, thereby less likely serotonin syndrome. The most likely are combination with MAOi like C. 8. Which of the following is least likely side effect of haloperidol? A. drowsiness B. loss of appetite C. weight gain D. dizziness/fainting E. Parkinsonism Ans. A

9. Haloperidol can cause hypotension. It is due to the? A. Inhibition of D 2 receptor B. Inhibition of M 1 receptor C. Inhibition of H 1 receptor D. Inhibition of alpha 1 receptor E. Inhibition of alpha 2 receptor Ans. D

Copyright © 2000-2016 TIPS Inc. Unauthorized reproduction of this manual is prohibited. This manual is being used during review sessions conducted by PharmacyPrep.

27-2

10. Which of the following is correct about dibenzazepine drugs like clozapine and quetiapine? A. Act more effectively on D 2 receptors than 5HT 2 B. Acts more effective on 5HT 2 than D 2 receptors C. Acts more effective on 5HT 2 and M 1 D. Acts more effectively on D 2 and 5HT 2 , H 1 , M 1 and alpha1 E. acts on D 2 receptors only Ans. B 11. Tobacco smoking? A. induce CYP3A4 B. induce CYP1A2 C. inhibit CYP3A4 D. inhibit CYP1A2 E. Substrate of CYP1A2 Ans.A

12. Tardive dyskinesia is? A. Parkinson's disease symptom B. Extra pyramidal symptoms C. Side effect of anti-Parkinson's drugs D. Side effect of levodopa E. Early symptom of antipsychotic drugs Ans. b

Copyright © 2000-2016 TIPS Inc. Unauthorized reproduction of this manual is prohibited. This manual is being used during review sessions conducted by PharmacyPrep.

27-3

PharmacyPrep.Com

Medicinal Chemistry and Pharmacology of Endocrine System

PHARMACY PREP MEDICINAL CHEMISTRY AND PHARMACOLOGY OF ENDOCRINE SYSTEM

1.What drugs increase insulin secretion? A. Acarbose B. Sitagliptine C. Nateglinide D. Liraglutide E. Metformin Ans. C 2. All of the following drugs increase endogenous insulin secretion, except? A. glyburide B. Glicalizide C. Metformin D. Nateglinide E. Repaglinide Ans. C 3. What is the common side effect of acarbose? A. Diarrhea B. Constipation C. GI bleeding D. ulcerative colitis Ans. A 4.Examples of ketones that appears in blood in DKA? A) beta hydroxyl butyric acid B) acetoacetic acid or acetoacetate C) acetone D) All of the above Ans. D

28-1 Copyright © 2000-2016 TIPS Inc. Unauthorized reproduction of this manual is prohibited. This manual is being used during review sessions conducted by PharmacyPrep.

PHARMACY PREP MEDICINAL CHEMISTRY AND PHARMACOLOGY OF RESPIRATORY DRUGS 1. Drug that may cause bronchospasm? A. Ipratropium B. Tiotropium C. Betaxalol D. Salbutamol E. Montelukast Ans.C 2. An asthma patient using salbutamol PRN but her asthma is not controlled. What drug is added next? A. Salmeterol B. Fluticasone inhaler C. Prednisone oral D. Montelukast E. Theophylline Ans. b 3. A patient using salbutamol for the past 6 months, but asthma is not controlled. Which drug is added next? a. Increase salbutamol dose B. Inhaled corticosteroids C. Formterol D. Oral prednisone Ans. b 4.In asthma patient concomitant anti-inflammatory therapy (inhaled corticosteroids) should be part of regimen if? A. Salbutamol needs to be used >3 times per week (not including its use to prevent exercise induced asthma).

5. Which of the following is the least likely use of montelukast? A. ASA induced asthma B. Acute attack of asthma C. Uncontrolled asthma D. nocturnal asthma Ans. B

29-1 Copyright © 2000-2016 TIPS Inc. Unauthorized reproduction of this manual is prohibited. This manual is being used during review sessions conducted by PharmacyPrep.

6. COPD patient currently using salbutamol PRN, ipratropium PRN, tiotropium daily, theophylline bid and Formoterol daily. Which of the following drug is the pharmacist concern? A) salbutamol B) ipratropium C) tiotropium D) formoterol E) theophylline ans. E Tips. Theophylline can cause tachycardia, tremors, nervousness, insomnia, abdominal cramps, Nausea and vomiting. 7.A 50 yr old COPD patient exacerbations, which of the following antibiotics is least likely used? a. amoxicillin B. cotrimoxazole C. azithromycin D. doxycycline E. ciprofloxacin Ans. E Tips. ciprofloxacin is least likely used for respiratory tract infections. However respiratory tract fluroquinolones levofloxacin and moxifloxacin can be used.

29-2 Copyright © 2000-2016 TIPS Inc. Unauthorized reproduction of this manual is prohibited. This manual is being used during review sessions conducted by PharmacyPrep.

www.pharmacyprep.com

Med Chem and Pharma of Musculoskeletal Drugs

PHARMACY PREP MEDICINAL CHEMISTRY AND PHARMACOLOGY OF MUSCULOSKELETAL DRUGS

Bisphosphonates mechanism by permanently binding surfaces of mineralized bone? A) Prevent osteoblast activity B) Prevent osteoclast activity C) Prevent bone minerilization D) Prevent bone deminerilzation E) Increase calcium deposits on bone Ans. B Tips: osteoclasts cause bone resorption thus preventing osteoclast increase bone mineral density . Teriparatide increases cortical thickness and bone size. It is? A. Anabolic agent B. Bisphosphonates C. SERM D. Hormone replacement therapy Ans. A

Which of the following prescription is the pharmacist concern? A. Azithromycin 250 mg daily for 3 days B. Alprazolam 0.5 mg SL as needed C. Methotrexate 7.5 mg daily D. Acetaminophen 650 mg Q4-6hrs E. Fentanyl patch 25 mg for 3 days for pain

What is the drug of choice for rheumatoid arthritis in pregnancy? A. Methotrexate B. Hydroxychloroquine C. Azathioprine D. Leflunomide E. Infliximab Ans. B Copyright © 2000-2011 TIPS Inc. Unauthorized reproduction of this manual is prohibited. This manual is being used during review sessions conducted by PharmacyPrep.

35-1

www.pharmacyprep.com

Med Chem and Pharma of Musculoskeletal Drugs

What analgesics drugs are NOT used to treat acute gout attacks? A) allopurinol B) acetaminophen C) colchicine D) dexamethasone E) indomethacin Ans. b

Prednisone used in treatment of gout arthritis (intra articular inj.) and severe asthma (oral). What is correct? A) gout patient may have more systemic SEs B) asthma patient may have more systemic SEs of steroid C) Asthma & gout patient have same SEs D) Only gout patient have prednisone SEs E) None of the above

What is NOT a side effect of alendronate if not taken properly? A) esophagitis B) heartburn C) GI bleeding D) Dysphagea E) Colorectal cancer Ans: E

All of the following drugs have uricosuric effect, except? A. allopurinol B. Fenofibrate C. Losartan D. Febuxostat E. Colchicine Ans. E

Q. Monoclonal antibodies consist of human and mouse proteins are? A. Humanized MAB Copyright © 2000-2011 TIPS Inc. Unauthorized reproduction of this manual is prohibited. This manual is being used during review sessions conducted by PharmacyPrep.

35-2

www.pharmacyprep.com

Med Chem and Pharma of Musculoskeletal Drugs

B. Murine MAB C. Mice MAB D. Chimeric MAB D. Human MAB Ans.d

Q. Bisphosphonates are? A. osteoclasts B. Osteoblast C. Pyrophosphate D. Phosphoric acids E. calcium phosphate Ans. c

Copyright © 2000-2011 TIPS Inc. Unauthorized reproduction of this manual is prohibited. This manual is being used during review sessions conducted by PharmacyPrep.

35-3

www.pharmacyprep.com

Pharmacy Prep MEDICINAL CHEMISTRY AND PHARMACOLOGY OF ANTIMICROBIALS

MP is 49-year-old women using calcium carbonate 500 mg QD and bring a new prescription of ciprofloxacin 500 mg BID for 3 days to treat UTI. What is appropriate? a) Skip calcium supplements for 3 days and take ciprofloxacin b) Take ciprofloxacin morning and supper time and take calcium 2 tab morning. C) take ciprofloxacin morning 2 tab and calcium 2 tab at supper time D) Take ciprofloxacin morning and supper and calcium 2 tab at noon. Ans. D

Which of the following is NOT an antifungal? A. Ketoconazole B. Clotrimazole C. Miconazole D. Metronidazole E. Fluconazole. Ans. D What is correct about ketoconazole? A. It is strong inhibitor of CYP3A4 B. It is strong inhibitor of CYP3A4 and 2C9 C. It is strong inducer of CYP3A4 and 2C9 D. It is substrate of CYP3A4 and 2C9 E. It is inhibitor of CYP2D6 Ans. b All of the following drug can cause serotonin syndrome when combined with MAOi, except? A. St. John wart B. Linezolid C. Fluoxetine D. Amitriptyline E. Amoxicillin Ans. E

Copyright © 2000-2016 TIPS Inc. Unauthorized reproduction of this manual is strictly prohibited and it is illegal to reproduce without permission. This manual is being used during review sessions conducted by PharmacyPrep.

36-1

www.pharmacyprep.com

PHARMACY PREP. METABOLISM

Q. What is the common metabolic reaction of acetyl salicylic acid? A. glucuronidation b. glutathione conjugation C. Glycine conjugation D. Hydroxylation E. oxidative deamination Ans. C Which of the following enzyme catalyzes glycine conjugation? A) UDP glucoronyl transferase B) Glutathione-S-transferase C) Acetyl co-A D) Cytochrome CYP 450 E) Esterase Ans: C

In hepatic detoxification related to glutathione conjugation, which aminoacid related? A) Glycine B) Cysteine C) Phenylephrine D) Glutathione E) Glutamic acid Ans. B If a patient taking simvastatin 40 mg daily, and brings a prescription of clarithromycin for 5 days. What is pharmacist appropriate action? A. dispense clarithromycin and counsel patient to space for 2 hr from simvastatin B. dispense clarithromycin and counsel patient to stop taking simvastatin for 5 days while taking clarithromycin C. Do not dispense clarithromycin and call doctor to change to other antibiotic D. Call doctor to change simvastatin to other atorvastatin E. No worries just dispense clarithromycin because patient has right to take antibiotics Ans. B

Copyright © 2000-2015 TIPS Inc. Unauthorized reproduction of this manual is prohibited. This manual is being used during review sessions conducted by PharmacyPrep.

37-1

www.pharmacyprep.com

PHARMACY PREP BIOPHARMACEUTICS

13. if a medication if pH = Pka? A-100% solute is ionized B-40% solute is ionized C-50% solute is ionized (1/2) D-99% solute is ionized E-95% solute is ionized Ans. C 14. A drug degradation at room temperature is determined by? A. Noye-Whittney equation B. Arrhenius equation C. Hasselbach-Handerson equation D. Ficks law E. Laplaws Law Ans. B

16. Which of the following forms buffer? A. weak acid and salt of acid B. Weak base and salt of acid C. Weak acid and salt of base D. Strong acid and salt of weak base E. Strong base and salt of weak acid Ans. A What is correct statement? Compare Cmax and Tmax Drug A Drug B Dose 100 100 Cmax 80 50 Tmax 1 6 A. Drug A is immediate release and Drug B is Sustain release. B. Drug A is controlled or sustained release C. Drug B is immediate release D. Drug A and B is controlled relase Copyright © 2000-2012 TIPS Inc. Unauthorized reproduction of this manual is prohibited. This manual is being used during review sessions conducted by PharmacyPrep.

38-1

www.pharmacyprep.com Ans. A

Rx for Dimicron MR 30 mg. A pharmacist dispensed gliclazide MR 30 mg. What is concern? A. Need prescriber approval B. Need to inform patient about change C. Cannot be changed because they are not equal D. This is therapeutic interchange E. This is Therapeutic alternate Ans. B

Q. What type drug absorption across placenta in pregnancy? A. Passive diffusion B. Active facilitated transport C. Facilitated diffusion D. Endocytosis E. Exocytosis Ans. A

3. Determine the F value for a capsule AUC 20 mg/dL/hour with 100 mg dose when i.v of same drug AUC is 25 mg/dL/hour with 100 mg dose. A. 20% B. 60% C. 40% D. 80% Copyright © 2000-2012 TIPS Inc. Unauthorized reproduction of this manual is prohibited. This manual is being used during review sessions conducted by PharmacyPrep.

38-2

www.pharmacyprep.com E. 100% Ans. D AUC po x 100 AUC iv

= 80%

4. Determine the F value for a capsule AUC 20 mg/dL/hour with 50 mg dose when i.v of same drug AUC is 25 mg/dL/hour with 100 mg dose. A-20% B-60% C-40% D-80% E-160% Ans. E AUC po x 100 AUC iv AUC po = 20 x 2 = 40 mg = 1.6 x 100 = 160% 25 mg 5. Two drugs to be pharmaceutical equivalent, the drugs must: I-contain same exipient II-Contain it salts III-Same therapeutic moiety A. I only B. III only C. I and II only D. II and III only E. All of the above

ans. d

3- What is the absolute bioavailability of tablet A? TABLET A-----------------50mg---------------AUC 40 IV PUSH -------------------10mg---------------AUC 50 a) 80% b) 0.16% c) 16% d) 8% e) 25% Ans- C We need to multiply the IV dose by 5 to find the same dosage so: IV Push---------------50mg---------------AUC 250 Bioaval = AUC Tablet A = 40 = 0.16 X 100 = 16% AUC IV Push 250

Physiochemical factors affecting the rate and extent of drug distribution includes? a) plasma protein binding b) types of surfactant in preparation Copyright © 2000-2012 TIPS Inc. Unauthorized reproduction of this manual is prohibited. This manual is being used during review sessions conducted by PharmacyPrep.

38-3

www.pharmacyprep.com c) the partition coefficient the drug d) percent of ionization e) Intracellular binding Ans. d

Area under the curve (AUC) of drug is double, the affect on clearance A. Drug clearance is doubled B. Drug clearance is increases C. Drug clearance is decreases D. Drug clearance does not change Ans. C Tips. AUC = (dose x F)/clearance or clearance = (dose x F)/AUC

Copyright © 2000-2012 TIPS Inc. Unauthorized reproduction of this manual is prohibited. This manual is being used during review sessions conducted by PharmacyPrep.

38-4

PharmacyPREP.Com

Physical Pharmacy

PHARMACY PREP. PHYSICAL PHARMACY

1. Which of the following equations can be used to easily calculate the pH. of a buffer solutions constituted of a weak acid and its salts. A. Hixson-Crowell equation B. Arrhenius equation C. Stokes equations D. Henderson-Hasselbalch equation E. Noyes-Whitney equation Ans. D Tips: Henderson-Hasselbalch equations used to mathematically calculate the pH of a buffer solution. 2. Arrhenius equation is used when we need calculate the: A. pKa of an acid solution B. Stability of a drug at room temperature C. pH of a buffer solution D. Rate of ionization of a drug E. Solubility of a solution Ans. B Tips: Arrhenius equation is used to calculate the stability of a drug at room temperature. 3. All are characteristics of a polymorph drug, EXCEPT: A. It has a two different physical form B. It melts in some determinate temperature, normally high temperatures (>30°C) C. Theobroma oil is a classic example of polymorph drug D. Polymorph drugs are widely used as suppository base E. Polymorph drugs melt in very low temperature (30C), and keeping solid in low temperature (Cost of Drug B Note: usually used for calculating different methods surgical techniques. 20. Review example cost utility analysis A Cost Drug Administration Monitoring Side effects Total cost

$250 $75 $100 $100 $525

B $430 $0 $10 $15 $455

Utilities Extra years of life Quality of life

2 0.33

3 0.25

Copyright © 2000-2016 TIPS Inc. Unauthorized reproduction of this manual is prohibited. This manual is being used during review sessions conducted by PharmacyPrep.

44-4

Pharmacyprep.com QALY 0.5 Cost utility ratio $525/0.5 $1050

0.40 $455/0.4 $1137

QALY = Quality Adjusted Life Years

Copyright © 2000-2016 TIPS Inc. Unauthorized reproduction of this manual is prohibited. This manual is being used during review sessions conducted by PharmacyPrep.

44-5

Pharmacyprep.com PHARMACOECONOMICS ANSWERS: 1. Ans: C 2. Ans: A Tips: Pharmacoeconomics is the description and analysis of the costs of drug therapy to health care system and society. 3. Ans: B 4. Ans: A 5. Ans: A 6. Ans: B 7. Ans: E 8. Ans: B 9. Ans: D 10. Ans: A 11. Ans: B 12. Ans: E 13. Ans: A Tips: Short form 36 (SF-36) most frequently used in pharmacoeconomic general health status instrument measuring 8 dimensions: physical functions, social functions, role emotional, role physical, bodily pain, mental health, general health, and vitality. Cost 14. Ans: C 15. Ans: A 16. A

Copyright © 2000-2016 TIPS Inc. Unauthorized reproduction of this manual is prohibited. This manual is being used during review sessions conducted by PharmacyPrep.

44-6

PHARMACY PREP DRUG DISCOVERY AND DEVELOPMENT 1. What is correct about phase I clinical trials? A. Healthy human volunteers are tested B. Disease human volunteers are tested C. Larger population than Phase II is tested D. Drugs are approved for sale after phase III clinical trials. 2. All of the following are true about Phase III human clinical trial, EXCEPT? A. Healthy human volunteers are tested B. Disease human volunteers are tested C. Larger population than Phase II is tested D. Drugs are approved for sale after phase III clinical trials. 3. Medications that are prescribed and sold over the counter in Canada are approved by? A. Canadian Health Act B. NAPRA C. Health Canada D. Federal Government of Canada E. Provincial governments 4. Healthy volunteers are tested in? A. Pre clinical studies B. Phase I clinical studies C. Phase II clinical studies D. Phase III clinical studies E. Phase IV clinical studies 5. Which of the following is decisive phase in new drug approval A. Pre clinical studies B. Phase I clinical studies C. Phase II clinical studies D. Phase III clinical studies E. Phase IV clinical studies 6. Who approves and authorizes the sale of medications in Canada? A. Therapeutic Product Directorate of Health Canada B. Food & Drug and Administration C. Pharmacy manager D. Pharmaceutical Manufacturer E. National association of pharmacy regulatory authority (NAPRA) Copyright © 2000-2016 TIPS Inc. Unauthorized reproduction of this manual is prohibited. This manual is being used during review sessions conducted by PharmacyPrep.

45-1

7. Who sets prescription drug prices in Canada? A. Health Canada B. Patented medication review board (PMPRB) C. Pharmacy manager D. Provincial college of pharmacy E. Pharmaceutical Manufacturer 8. Who sets OTC drug prices? A. Health Canada B. Patented medicine review board (PMPRB) C. Pharmacy manager/owner D. Provincial college of pharmacy E. Pharmaceutical Manufacturer

Copyright © 2000-2016 TIPS Inc. Unauthorized reproduction of this manual is prohibited. This manual is being used during review sessions conducted by PharmacyPrep.

45-2

DRUG DISCOVERY AND DEVELOPMENT ANSWERS: 1.Ans: A 2. Ans: A 3. Ans: C 4. Ans: B 5. Ans: D 6. Ans: A 7. Ans: B 8. Ans: C

Copyright © 2000-2016 TIPS Inc. Unauthorized reproduction of this manual is prohibited. This manual is being used during review sessions conducted by PharmacyPrep.

45-3

Copyright © 2000-2016 TIPS Inc. Unauthorized reproduction of this manual is prohibited. This manual is being used during review sessions conducted by PharmacyPrep.

45-4

Pharmacyprep.com

PHARMACY PREP EPIDEMIOLOGY AND CLINICAL STUDIES

1. Which of the following is an example of non interventional trials or observational trials and follow up like a comparison of two patient with COPD, one smokes and the other non smoker? A. Cohort study B. Clinical trials C. interventional trials D. all of the above 2. What is included in "intent-to-treat analysis"? A. Data from patients originally assigned B. Data from patient participated from entire treatment C. Data from patient discontinued treatment D. Data from dropped out patient for non therapeutic reason E. All of the above Ans. E 3. How to test effectiveness for treating pneumonia? A. randomized placebo trials B. randomized comparative trial C. cohort. D. Case control studies Ans. B

Copyright © 2000-2016 TIPS Inc. Unauthorized reproduction of this manual is prohibited. This manual is being used during review sessions conducted by PharmacyPrep.

46-1

www.pharmacyprep.com

PHARMACY PREP. BIOSTATISTICS CALCULATIONS 1.What is incorrect about the relation of confidence interval and sample size A. There is a relationship between sample size and the width of the confidence interval. B. The general rule is that the width of the confidence interval will be proportional to 1/ n, so to halve the interval you need 4 times the sample size. C. Larger samples give more precise estimates with lower standard error and tighter confidence intervals. D. As the sample size increases, the interval and its width decrease, thus providing a more precise estimate of the population value. E. The larger the sample size the larger the confidence interval Ans. E Tips. The larger the sample size the 2. A study experiment event rate or treatment reduced 10% risk and control reduced 7.5% risk. What is absolute risk reduction? A. 7.5% B. 2.5% C. 10% D. 15% Ans. B ARR = EER-CER or 10-7.5 = 2.5% NNT = 1/ARR = (1/2.5) x 100 = 40 NNT are always rounded up to the nearest whole number. 3. If a drug reduces the risk of a bad outcome from 50 per cent to 30 per cent. The ARR is? A. 50% B. 30% C. 20% D.10% Ans. What is NNT? 1/0.2 = 5

4) The results of the trial of vasodilators in patient with congestive heart failure. The mortality in the ACEi arm was 33% and in hydralazine plus nitrate group it was 39%. What is absolute risk difference? A) 0.86 B) 6% C) 14% D) 29% E) 3.3% Ans. B

Copyright © 2000-2016 TIPS Inc. Unauthorized reproduction of this manual is prohibited. This manual is being used during review sessions conducted by PharmacyPrep.

47-1

www.pharmacyprep.com 5.In the above case how many patient do you need to treat to benefit one case (NNT)? A. 10 B. 5 C. 6 D.17 Ans. D 6.A new drug absolute risk reduction is 4% for preventing stroke. Means? A. Four patient in 100 actually benefit from new drug B. Four patient of all study group benefit from new drug c. New drugs four times better than old drug D. New drugs is 4% better than old drug ans. a 7.The NNT for the above case is 25, means? A. 25 patient had to receive the new drug in order to one patient to benefit B. New drug benefit 25 people patient in 100 patients C. 25 patient had to receive the new drug in order to benefit 100 patients. D. 100 patient had to receive the new drug in order to benefit 25 patients Ans. A

8) What is relative risk reduction? A) 0.86 B) 5% Ans. C Tips:( 39-33/39 )x 100 = 14%

C) 14% D) 29% E) 3.3%

Copyright © 2000-2016 TIPS Inc. Unauthorized reproduction of this manual is prohibited. This manual is being used during review sessions conducted by PharmacyPrep.

47-2

Pharmacy Prep HOSPITAL PHARMACY 1. Director of pharmacy reports to? A. CEO B. president of hospital C. board of trustees D. pharmacy manager E. vice presidents Ans. e 2. What is not included in unit dose system? A. package containing the amount of drug for one dose B. Oral medications contain 24 hr dosages C. contain individual drawer or tray for each patient D. nurses prepare unit dosages Ans. D 3. Which of the following is NOT responsibility of P&T committee A. Formulating policies regarding selection and therapeutic use of drugs B. Educates professional staff such as physician, pharmacist and nurse about matters related to patient quality and safety of drugs. C. Develops list of formulary D. Develops generic and therapeutic equivalent list of drugs E. Recommends therapies to patients Ans. E 4. Listing of drug by their generic name followed by strength, form, toxicology, use & recommended quantity dispensed B. Prepared locally by its own clinical staff or P&T committee C. Each hospital has their own formulary D. Formulary is federal and all hospital in Canada has the same formulary Ans. d 5. Which of the following is least likely done by the regulated pharmacy technician? A. sterile preparation of IV B. Checking formulary stability reconstituted IV preps C. Preparing TPN D. Final check before delivery to wards E. Counseling to patients Ans. E Copyright © 2000-2016 TIPS Inc. Unauthorized reproduction of this manual is prohibited. This manual is being used during review sessions conducted by PharmacyPrep.

48-1

6. A hospital doctor ordered a drug that is not present in hospital formulary list. What is appropriate action? A. Dispense equivalent drug present in formulary B. Check in non formulary list or order from community pharmacy C. Call doctor to change the medication from list of formulary D. Contact other hospital pharmacy to get the medication. Ans. b 7. Hospital doctor prescribed amoxicillin 500 mg for 8 hourly administration. What is correct? A. unit dose is dispensed with one dose of amoxicillin 500 mg one capsule B. unit dose is dispensed with three capsule of amoxicillin 500 mg for an 8 hourly administration C. unit dose is dispensed with nine capsule of amoxicillin 500 mg for 8 hourly for 3 days administration D. Call the doctor to clarify Ans. b 8. Dispensing errors in hospitals can be minimized by A. Automation B. Computer generated prescription and eMAR C. Continue education of hospital staff. D. All of the above Ans. D

Copyright © 2000-2016 TIPS Inc. Unauthorized reproduction of this manual is prohibited. This manual is being used during review sessions conducted by PharmacyPrep.

48-2

www.Pharmacyprep.com

Pharmacy Calculations PHARMACY PREP.

Basic CALCULATIONS 1.A child weighs 22 lbs, and doctor wants to give a drug 1 mg/kg/day bid, how much you should give per day? a. 10 mg/day B. 5 mg/day C. 15 mg/day D. 20 mg/day Ans. a

2.A child weighs 22 lbs, and doctor wants to give 1 mg/kg/day bid drug, how much you should give per dose? a. 10 mg/dose B. 5 mg/dose C. 15 mg/dose D. 20 mg/dose Ans. b 3.A patient weight 180 lbs has admitted to emergence for congestive heart failure and severe edema. Patient was give furosemide iv infusion for the past 24 hours. After discharge the patient weight was 173 lbs. How many kg patient weight is lost? A. 7 kg B.3.2 kg C. 2.2kg D. 172 kg

180 lbs-173 lbs = 7 lbs 7/2.2 = 3.2 kg 4.A prescription order of cisplatin 750 mg/m2/day, IV tid. The patient weighs 75 kg and is 162 cm tall. Calculate the dosage to administer? BSA in m2 =

c

= =

3.37

= 1.84 m2 = 750 mg/m2

m2 = 1380 mg per day

1380/3 = 460 mg TID How to use Calculator SHARP EL-510R 2ndF press square root. 3.37 Copyright © 2000-2016 TIPS Inc. Unauthorized reproduction of this manual is prohibited. This manual is being used during review sessions conducted by PharmacyPrep.

49-1

www.Pharmacyprep.com

Pharmacy Calculations

5.Patient age 60 yo, weight 78 kg and height 184 cm. Doctor prescribed 5-flurouracil 300 mg/m2 BSA =

se

ac a e

e

se

ra a

s n body surface area?

c

1.99 mg/m2 er a

6.A chemotherapy protocol for a pt. body weight 68 kg and height 1.7 m. Chemo drug leucovorin 400 mg/m2 is recommended what is daily dose? c = 1.79 m2

= 716 mg per day For leucovorin, in day 2 cycle each dose of medication is reduced by 50%? 716/2 = 358 mg

7.You are to administer 5 ml of ferrous gluconate that must be diluted in water to protect the client’s teeth and gastrointestinal track. The directions in your drug reference state that each ml of this medication must be diluted in 20 ml of water. Once diluted, the total volume to be administered is A. 100 ml B.105 ml C. 125 ml D. 120 ml Ans. B Note. Diluted in, added to, added in may increase total volume or weight. Dilute up to, quantity sufficient or qs, mark up to the volume should not increase volume. 8. Syrup is an 85% w/v solution of sucrose in water. It has a density of 1.313 g/ml. How an ra s a er s e se a e s r A. 85 g B. 106 g C. 125 g D. 164 g E. 58 g

Copyright © 2000-2016 TIPS Inc. Unauthorized reproduction of this manual is prohibited. This manual is being used during review sessions conducted by PharmacyPrep.

49-2

www.Pharmacyprep.com

Pharmacy Calculations

100 ........... 85 125.............? = 106 g 164-106 = 58 g 9.Ampicillin 1 g drug. Add 6.8 ml sterile water to provide final concentration volume 8 mL. What is concentration of the reconstituted solution in the vial in mg/mL? A. 8 mg/mL B. 125 mg/mL C. 100 mg/mL D. 150 mg/mL Unknown

Known

x mg/mL

1000 mg/8mL

x mg = (1mL x 1000 mg)/8 mL 125 mg/mL 10.Given ampicillin 1 g vial. If the final concentration has been calculated to 125 mg/1 mL. What is the volume taken if dose of the ampicillin is 500 mg? A. 8 mL B. 4 mL C. 2 mL D.1 mL Ans. B 125 mg...........1 mL 500 mg.............? x/500 mg

1 ml/125 mg

= 4mL

11.A sterile methylprednisone powder vial contain 125 mg drug. Reconstituted by adding 1.8 mL of sterile water to final volume 2 mL. What is final concentration of reconstituted solution in the per mg/mL? A. 125 mg/mL B. 62.5 mg/mL C. 31.5 mg/mL D.16.5 mg/mL ans. B

Copyright © 2000-2016 TIPS Inc. Unauthorized reproduction of this manual is prohibited. This manual is being used during review sessions conducted by PharmacyPrep.

49-3

www.Pharmacyprep.com

Pharmacy Calculations

2 mL ..................125 mg (known) 1 mL .................? (unknown) uknown x mg/ 1mL

known 125mg/2mL

x mg = (1 mL x 125 mg) / 2 mL = 62.5 mg/mL

12.An order for product preparation to maintain alkaline urine. na c ncen ra n c ra e citric acid 6% w/w Sterile water for irrigation added up final volume 1000 mL How many grams of sodium citrate powder needed? A. 100 g B. 10 g C. 60 g D. 50 g Xg/1000 mL

1g/100 mL

Xg = (1000 mL x 1g)/100 mL = 10 g

13.Product order to prepare 100 ml of hydrocortisone oral suspension with final concentration 1 mg/mL. Available hydrocortisone 20 mg tablets. How many tablets needed to prepare 100 mL? A. 20 tablets B. 10 tablets C. 5 tablets D. 1 tablet Ans. C

Copyright © 2000-2016 TIPS Inc. Unauthorized reproduction of this manual is prohibited. This manual is being used during review sessions conducted by PharmacyPrep.

49-4

www.Pharmacyprep.com

Pharmacy Calculations

Copyright © 2000-2016 TIPS Inc. Unauthorized reproduction of this manual is prohibited. This manual is being used during review sessions conducted by PharmacyPrep.

49-5

www.Pharmacyprep.com

Dose Calculations

Dose Calculations

1) How many tablets are needed for following prescription? Provera 5 mg tab QD 16 –25 days for 3 months One tablet containing 5 mg a. 50 tablets b. 60 tablets c. 80 tablets d. 75 tablets e. 30 tablets Ans. e

2) A physician wants to switch a terminally-ill patient from slow-release morphine sulphate tablets, 15 mg twice daily, to a liquid morphine sulfate dosage form because the patient has difficulty in swallowing tablets. If a morphine sulfate solution containing 5 mg per mL is prescribed q4h, what volume should be dispensed for a 20-day supply to provide the same pain relief as the tablet regimen? a) 20 mL b) 60 mL c) 80 mL d) 100 mL e) 120 mL Ans. e Working: 5 mg/1ml x 6 times daily = 6 ml 6 ml x 20 = 120 ml or i. ii. iii.

15 mg BID = 30 mg daily For 20 days: 30 mg x 20 = 600 mg are dispensed Solution contains 5 mg/ml. 600 mg are contained in 600/5 = 120 mL

Ans: E

Copyright © 2000-2016 TIPS Inc. Unauthorized reproduction of this manual is prohibited. This manual is being used during review sessions conducted by PharmacyPrep.

50-1

www.Pharmacyprep.com

Dose Calculations

3. A patient is currently taking 220 mg anhydrous zinc sulfate. To receive the equivalent amount of elemental zinc, how many milligrams of zinc sulfate heptahydrate (.7H 2 O) would be patient need to take? (MW: zinc 65, ZnSO 4 161, H 2 O 18). a) 123 mg b) 220 mg c) 300 mg d) 392 mg e) 545 mg ZnSO4 = 161 ZnSO4 heptahydrate= (7x18 =126) 161+126 = 287 161..............220 mg 287................? (220 x 287)/161 = 392 OR Working: i. ii. iii.

220 mg ZnSO 4 provides 220/161 = 1.366 mmol Zn (and 1.33 mmol SO 4 , but this is not relevant) Each mmol of ZnSO 4 . 7H 2 O also provides 1 mmol Zinc Calculate the molecular weight of ZnSO 4 . 7H 2 O as follows:

ZnSO 4 = 161 7H2O = (7 X 18) = 126 ZnSO 4 7H2O = 161 + 126 = 287 iv.

Therefore, 1.366 mmol of ZnSO 4 . 7H2O (which provides 1.366 mmol Zinc) are provided by 1.366 x 287 = 392 mg

Ans: D 4) A physician has prescribed 60 g of 0.01% fluocinolone acetonide cream. You have available the commercially prepared cream containing 0.2% fluocinolone acetonine (Synalar cream) and cream base (Dermabase ) for dilution. To prepare the Rx you would require: a) 1.2 g Synalar and 58.8 g Dermabase b) 3 g Synalar and 60 g Dermabase c) 0.3 g Synalar and 59.7 g Dermabase d) 3 g Synalar and 57 g Dermabase C1. V1 = C2. V2 0.01% x 60 g = 0.2% . V 2 Copyright © 2000-2016 TIPS Inc. Unauthorized reproduction of this manual is prohibited. This manual is being used during review sessions conducted by PharmacyPrep.

50-2

www.Pharmacyprep.com

Dose Calculations

X = (60 g x 0.01)/ 0.2 = 3 g Synalar Order is for 60 g - 3 g = 57 g dermabase Working: i. ii. iii.

60 g of 0.01% fluocinolone acetonide (required in the Rx) contains 0.01/100 x 60 = 0.006 g fluocinolone acetonide) You have 0.2% fluocinolone acetonide (0.2 g per 100 g = 0.002 g fluocinolone per gramme of cream) Therefore, for the Rx you need 0.006 / 0.002 = 3 g Synalar cream

Ans: D B is wrong, because 3 g Synalar and 60 g of Dermabase will give 63 g of total cream, with a concentration of 0.0095% (0.006 / 63 x 100) 5. In the International System of Units (SI), most drugs levels are to be reported in the units of micromoles/litre (µmols/litre). The therapeutic concentration range for theophylline (MW 180) is 10-20 µg/mL. the SI equivalent would be: a) 0.055- 0.11 µmol/L b) 27-55 µmol/L c) 55-110 µmol/L d) 110-220 µmol/L e) 180-360 µmol/L Working: i. Careful!!! Theophylline therapeutic concentration is given in µmol/mL, while answers are given in µmol/L. ii. Convert therapeutic concentration to µg/L: 10-20 µmol/mL = 10,000 – 20,000 µg/L iii. Lower parameter: 10 µg/mol = 10,000 µg/L = 10,000/180 = 55 µmol/L iv. Upper parameter: 20 µg/mL = 20,000 µg/L = 20,000/180 = 111µmol/L Ans-C 6. Prednisolone each tablet containing 10mg. Taper 1/2 tablet every week .How tablets are needed? 1st week = 7 tab 2nd week = 3.5 tab 10.5 tablet

7. A family of 1 adults (300 mg/wk) and 8-year-old child (75mg/wk), 14 year old (225mg/wk) teenager, going for 2 weeks trip to malaria infections area. You have 300

Copyright © 2000-2016 TIPS Inc. Unauthorized reproduction of this manual is prohibited. This manual is being used during review sessions conducted by PharmacyPrep.

50-3

www.Pharmacyprep.com

Dose Calculations

mg base drug. How many tablets will you give for whole family? (Prophylaxis require 4 weeks before trip, during trip and 2weeks after trip) A-17 tablets B-18 tablets C-16 tablets D-19 tablets Ans. C Total = 8 week therapy Adult = 8 tablet 14 yo = 6 tablet (225 mg/wk) 8 yo = 2 tablet (75 mg/wk) Total tablet = 16 tablets 8) A parenteral solutions used in hospital pharmacy. If 250 g of parenteral solution dissolved in 1000 mL of glycerin (density of glycerin is 1.25 g/mL), the concentration of parenteral solution is: A) 26% w/w B) 24% w/w C) 16 % w/w D) 20%w/w TIPS: D = M/V Concentration = Solute x 100 Solute + Solvent =

250 g x 100 250g+1250 g

= 16% w/w M=DxV M = 1.25 x 1000 mL = 1250 g Then 250 g+1250 g = 1500 g (total volume) (250g/250 g+1250 g) x100 = 16%

List of drugs require tapering. Prednisone/prednisolone Benzodiazepine SSRI Beta blockers

Copyright © 2000-2016 TIPS Inc. Unauthorized reproduction of this manual is prohibited. This manual is being used during review sessions conducted by PharmacyPrep.

50-4

www.Pharmacyprep.com

Dose Calculations

17. Prednisolone each tablet containing 5mg. Start 35 mg and then taper by 5mg every 1 day. How many tablets are needed? A-20 tab B-56 tab C-14 tab D-28 tab Tips: 7+6+5+4+3+2+1 = 28 tab For above questions, how many days patient takes prednisone? 7days 17. Prednisolone each tablet containing 5mg. Start 35 mg and then taper by 5mg every 2 day. How many tablets are needed? A-20 tab B-56 tab C-14 tab D-28 tab 7+7+6+6+5+5+4+4+3+3+2+2+1+1 = 56 tab = 14 days For the above questions. Patient take for how many days? 14 days 18) If three tablets contain 975 mg of ASA, how many grams of ASA would be contained in 12 tablets? A) 4g B)325g C) 3.9g D) 39g E) 390g

19) How many milliliters of a heparin sodium injection containing 200,000 heparin units in 10 mL should be used to obtain 5,000 heparin units? A-0.50mL B-0.25mL C-0.10mL D-0.75mL E-250mL

200,000 ---- 10 ml 5000 ---- ? 5000 x 10 / 200,000 = 0.25 ml 20) A physician asks the pharmacist to compound codeine sulfate tablet triturates containing 10 mg of codeine. The patient need a 5-day supplies and may take 1-2 q4-6h PRN How much codeine is needed to fill this prescription? Copyright © 2000-2016 TIPS Inc. Unauthorized reproduction of this manual is prohibited. This manual is being used during review sessions conducted by PharmacyPrep.

50-5

www.Pharmacyprep.com

Dose Calculations

A. 300 mg B.400 mg C-600 mg D-1200 mg Ans: c PRN = as needed or on demand 12 tab per day for 5 days = 60 tab each tab contain 10 mg so 10 mg x 60 tab = 600 mg 23) Prednisolone each tablet containing 5 mg. Start 10 mg and then taper by 1/2 tab every week. How many tablets are needed? A-21 tab B-56 tab C-14 tab D-28 tab E-35 tab Ans. Tips: 14 tab+10.5tab+7tab+3.5 tab = 35 tab 2 tab daily x 7 d = 14 tab 1.5 tab daily x 7 d = 10.5 tab 1 tab daily x 7 d = 7 tab 1/2 tab daily x 7 d = 3.5 tab 5 tab x 7 days How many weeks therapy? 26) Dexamethasone 8 mg bid x 7 days then taper by 1 mg twice day every 2nd day until finish? Available strength 4 mg and 0.5 mg. Calculate how many tablets of both strengths you will dispense? A) 112 tablets of 4 mg or 800 tablets of 0.5 mg B) 126 tablets of 4 mg or 1008 tablets of 0.5 mg C) 56 tablets of 4 mg and 96 of 0.5 mg D) 136 tablets of 4 mg or 990 tablets of 0.5 mg E) Give both 44 of 4 mg and 96 of 0.5 mg tablets

Copyright © 2000-2016 TIPS Inc. Unauthorized reproduction of this manual is prohibited. This manual is being used during review sessions conducted by PharmacyPrep.

50-6

www.Pharmacyprep.com

8 mg Bid 7 mg bid 6 mg bid 5 mg bid 4 mg bid 3 mg bid 2 mg bid 1 mg bid Total

days 7d 2d 2d 2d 2d 2d 2d 2d

Dose Calculations

4 mg TAB 28 4 4 4 4 0 0 0 44

0.5 mg TAB 0 24 16 8 0 24 16 8 96

1st week 112 mg 2 days = 28 mg 2 days = 24 mg 2 day = 20 mg 2 day = 16 mg 2 days = 12 mg 2 day = 8 mg 2 day = 4 mg Total = 224 mg for 21 days

7 d x 16 mg = 112 mg = 28 tab of 4 mg 2 d x 7 mg bid =14 mg for 2 d 28 mg = 4 of 4 mg & 24 of 0.5 mg 2 d x 6 mg bid 24 mg = 4 of 4 mg and 16 of 0.5 mg 2 d x 5mg bid= 20 mg = 4 of 4 mg and 8 of 0.5 mg 2 d x 4mg bid =16 mg = 4 of 4 mg 2 d x 3 mg bid = 12 mg = 24 of 0.5 mg 2 d x 2 mg bid = 8 mg = 16 of 0.5 mg 2 d x 1 mg bid = 4 mg = 8 of 0.5 mg Ans. Tips. If you give both 44 of 4 mg and 96 of 0.5 mg Total 224 mg Total therapy 21 days

Copyright © 2000-2016 TIPS Inc. Unauthorized reproduction of this manual is prohibited. This manual is being used during review sessions conducted by PharmacyPrep.

50-7

Pharmacyprep.com

Dilutions and Concentrations Pharmacy prep DILUTIONS AND CONCENTRATIONS

Q& A

1) A prescription for hydrocrotisone cream 0.1%. Pharmacy has 0.25% available in 30 g tube. How many grams diluents base (vanishing cream) should be added? A) 30 g

B) 45 g

C) 50 g

D) 75 g E) 25 g

V2 = (C1 x V1)/C2 V2 = (0.25% x 30 g)/0.1% V2 = 75 g but the added tube contains 30 g so the used base = 75 g - 30 g = 45 g

2) Dexamethasone is available as 4 mg/mL preparation. in infant is to receive 0.35 mg. Prepare a dilution so that the final concentration is 1 mg/mL. How much diluents will you need if the original product is in a 1mL vial and you use the full vial? A) 4 ml

B) 3ml

C) 1ml

D) 0.35ml

Ans: B Step1: determine the volume of final product. Since dexamethasone is 4mg/mL, a 1 ml vial have 4mg of drug X ml/ 4 mg = 1 ml/1 mg = 4 mL Step2: subtract the volume of concentrate from the total volume to determine the amount of diluents needed. 4 ml-1ml = 3 mL

3.If a 600 ml of a 15% (v/v) solution of methyl salicylates in alcohol are diluted to 1500 ml what will be the percentage strength. 51-1

Pharmacyprep.com

Dilutions and Concentrations

C2 = (C1xV1)/V2 C2 = (15 x 600 ml)/1500 mL C2 = 6%

4) A physician has prescribed 60 g of 0.01% fluocinolone acetonide cream. You have available the commercially prepared cream containing 0.2% fluocinolone acetonine (Synalar cream) and cream base (Dermabase ) for dilution. To prepare the Rx you would require: a)

1.2 g Synalar and 58.8 g Dermabase

b)

3 g Synalar and 60 g Dermabase

c)

0.3 g Synalar and 59.7 g Dermabase

d)

3 g Synalar and 57 g Dermabase

C1. V1 = C2. V2 0.01% x 60 g = 0.2% . V2 X = (60 g x 0.01)/ 0.2 = 3 g Synalar Order is for 60 g - 3 g = 57 g dermabase

5.Prescriber orders to prepare 1% hydrocortisone 60 g. Your pharmacy has stock of 0.5% hydrocortisone and 2.5% hydrocortisone. How many each part should be taken?

0.5

1.5 1

2.5

0.5

2.5% of HC ...........( 0.5/2) x 60 = 15 g 0.5% of HC............(1.5/2)x60 = 45 g

51-2

Pharmacyprep.com

Dilutions and Concentrations

6.Hydrocortisone 1% 60 g. Your pharmacy has 2.5% hydrocortisone and petrolatum base. What fraction of each hydrocortisone needed use to prepare above prescription

2.5%

1 1%

0%

1.5

2.5% HC = 24 g petrolatum base = 36 g 7. How many milliequivalents of sodium is in 92 mg of NaCl salt? M.wt of Na = 23 mEq = [mg x valence]/(Mol.Wt) mEq = 92/23 = 4 mEq 8) You are given ZnCl 2 0.7%, phenylephrine 0.1% and boric acid 1.1% with E values 0.16, 0.32 and 0.5 respectively. This solution will be: A) Hypotonic B) Hypertonic C) Isotonic D) Non isotonic Solution. [(0.7 x0.16] + [(0.1 x 0.32] + [ (1.1 x 0.5)] = 0.112 + 0.032 + 0.55 = 0.694 = 0.9>0.69 = Hypotonic How much NaCl required making isotonic solution? 0. 9 - 0.69 = 0. 21 or 2 mg 51-3

Pharmacyprep.com

Dilutions and Concentrations

9) Zinc sulfate is a two-ion electrolyte, dissociating 70% in weak solutions. Calculate its dissociation factor. ZnSO 4

Zn + SO 4 + ZnSO 4 70+ 70

+30

170/100 = 1.7

51-4

www.pharmacyprep.com

Sterile Preparations PHARMACY PREP Brand and Generic Name Indexes

Copyright © 2000-2015 TIPS Inc. Unauthorized reproduction of this manual is prohibited. This manual is being used during review sessions conducted by PharmacyPrep.

62-1

www.Pharmacyprep.com PHARMACY PREP PRESCRIPTION PROCESSING AND DISPENSING

1.A 65 yo customer at your pharmacy using nitroglycerine patch daily for the prevention of coronary artery disease. If patient a new prescription. Which of the following Rx is the pharmacist concern? A. Nitroglycerine 0.3 mg SL tab B. Nitroglycerine 0.4 mg SL spray C. Sildenafil 50 mg as needed D. Tylenol # 3 for as needed for pain E. Ritonavir soft gelatin cap 100 mg daily Ans. c 2.A very frequent customer at your pharmacy, request for 15 days supply of Lipitor 10 mg tablets because customer has no prescription and no refills of Lipitor. In the next few hours he has to take a flight for vacation. Currently all walk-in-clinics are closed and his physician is not available for prescription.? According to extended scope of pharmacy practice pharmacist dispensed 15 days of drugs? This is defined as? A. Adaptation B. Advancing a drug C. Pharmaceutical opinion D. Professional discretion E. Professional judgement Ans.b 3.MP came from the British Columbia, during the journey he lost his medications. He approaches at your pharmacy with a rough list of medication, he was using. The pharmacy and doctor clinic is currently closed due to long weekend in BC. What is appropriate? A. dispense the list of medication B. refer him to walk in clinic C. refuse to dispense any medications D. write prescription E. dispense OTC medications Ans. b 4.A 75 yo patient has been discharged from the hospital. Presents at your pharmacy with the following discharge report prescription. What is the pharmacist concern? a. Metformin 1000 mg, PO, evening with dinner and metformin 1000 mg, po, morning with breakfast B. Levodopa/carbidopa 25/100 mg, po, four times per day C. Digoxin 120 mg once daily in the morning Copyright © 2000-2016 TIPS Inc. Unauthorized reproduction of this manual is prohibited. This manual is being used during review sessions conducted by PharmacyPrep.

53-1

www.Pharmacyprep.com D. Donepezil 5 mg, po, daily E. Tramadol 50 mg as needed for pain Ans. c 5.Which of the following prescription is written incorrectly? A. Pantoprazole 40 mg, po, daily before breakfast B. Spironolactone 10 mg, po, daily C. Acetaminophen 650 mg, every 4 hours D. Fentanyl patch 50 mcg/hour daily E. Alprazolam 0.5 mg SL tab daily at bedtime Ans. d

Copyright © 2000-2016 TIPS Inc. Unauthorized reproduction of this manual is prohibited. This manual is being used during review sessions conducted by PharmacyPrep.

53-2

www.pharmacyprep.com PHARMACY PREP DRUGS IN SPECIAL POPULATIONS 1. Which of the following incorrect about elderly? A. Increased pH of stomach secretion B. Slow gastric emptying time C. Reduced enzyme production D. Increased excretion of renal elimination drugs E. Decreased metabolism of drugs Ans. D 2. Which of the following incorrect about pregnancy? A. Increased pH of stomach secretion B. Slows gastric emptying time C. Increased enzyme production D. Increased excretion of renal elimination drugs E. Increased metabolism of drugs Ans. A 3. which of the following is the most critical of organ development in pregnancy? A. Biogenesis B. Organogenesis (2 to 8 wks) or emryotoxic C. Fetal Period (9wk to labor) D. The first 15 -21 days (biogenesis) E. Ninth week to birth Ans. b Tips. Biogenesis - The first 15-21 days after fertilization. Cleavage, and germ layer formation. Organogenesis 2-8 wks - The major organ start developing. (congenital defects) Fetal period- at 9th wk, the embryo referred to as fetus. 4. when you considering medication therapy in pregnancy. A. both mother and fetus must be considered B. weigh risk and benefits C. rule of thumb is lowest effective dose for shortest period D. All of the above ans.d

Copyright © 2000-2016 TIPS Inc. Unauthorized reproduction of this manual is prohibited. This manual is being used during review sessions conducted by PharmacyPrep.

Pharmacyprep.com

PHARMACY PREP CLINICAL TOXICOLOGY 1- An adult patient who ingested 30 acetaminophen tablets (325 mg/tab) 6 hours ago should be treated with/by A. EDTA infusion B. Ipecac syrup C. Activated charcoal D. N-acetylcysteine E- Probenecid 2. Glutathione conjugation are catalyzation is extremely important in preventing toxicity from variety of drugs, toxic products are excreted as: A) S-adenosyl transferase B) Glutamic acid C) Marcapturic acid D) Mercaptopurines E) Sulfadryl groups 3-A 33-year-old male receiving Acetaminophen 300 mg + codeine 30 mg + caffeine 15 mg (Tylenol # 3) for pain relief, which of the following laxative is appropriate choice for constipation associated with codeine; A-Sennakot B-Bulk laxatives C-Lactulose D-Psyllium E-Glycerine 4-A patient brings the prescription of amitriptyline 300 mg 3 month’s supply. Patient medical history includes, major depression and in the past has 2 unsuccessful attempts to suicide. What is the appropriate, a pharmacist should do? A-Dispense 3 months supply of amitriptyline and caution about overdose symptoms. B-Dispense in part fills or small quantities and ask patient pick up in parts every 15 days C-Refuse the prescription and ask patient to see other doctor D-Refer patient to other physician for mental assessment. 5-Overdose of acetaminophen is treated by? A-N-acetylcysteine, administered within 1 hour B-N-acetylcysteine, administered within 8 hours C-N-acetylcysteine can be administered anytime after overdose D-No antidote is available for acetaminophen overdose Copyright © 2000-2016 TIPS Inc. Unauthorized reproduction of this manual is prohibited. This manual is being used during review sessions conducted by PharmacyPrep.

55-1

Pharmacyprep.com E-Sodium bicarbonate diuresis 6-What is daily maximum dose of acetaminophen? A-10 g B-4 g C-7 g D-1 g E-8 g 7- A grand mother gave 2 tbsp (250 mg/5ml) of amoxicillin suspension instead of 2 tsp to a child 9 mo age. What is appropriate to do? A-Monitor diarrhea B-Refer to doctor C-Refer to emergency D-Call poison control center E-Wait and watch 8- A 5 yr child swallowed 5 g of nystatin cream, what to do? A-Monitor diarrhea B-Refer to doctor C-Refer to emergency D-Call poison control center E-Wait and watch 9- A regular customer of your pharmacy brings a prescription of amitriptyline 300 mg daily for 3 mo. However patient profile shows he had attempted suicide in the past but was not successful. What is appropriate to do? A) recommend not to overdose B) recommend to take as doctor prescribed C) give a part fill of monthly dose D) talk to doctor to prescribe other antidepressant E) refuse to dispense 10- A grand mother gave 2 tbsp of amoxicillin of 500 mg/5ml bid instead of 2 tsp to a 9 month old child. What to do? A) refer to emergency B) refer to doctor C) wait and see D) recommend to skip next dose E) contact regional poison control centre 11) A 2 yr old child swallowed nystatin 10 g suspension, What to do? A) refer to emergency B) refer to doctor C) wait and see D) recommend to skip next dose E) not a concerned

Copyright © 2000-2016 TIPS Inc. Unauthorized reproduction of this manual is prohibited. This manual is being used during review sessions conducted by PharmacyPrep.

55-2

Pharmacyprep.com 12) A doctor comes to your pharmacy and write a prescription for oxycontin 100 tablets for himself? What to do? A)dispense 50 tablets only B)dispense 200 tablets only C)it is illegal dispense narcotics to doctor D)It is unethical to dispense narcotics for themselves E)Refer doctor to Centre of addiction and mental health (CAMH) 13) A regular customer of your pharmacy, using recently picked up prescription of Tylenol # 3 (acetaminophen 300 mg + codeine 30 mg + caffeine 15 mg) 90 tablets. Now wants to buy acetaminophen 500 mg box of 200 tablets. What is the pharmacist concern? I) overdose II) constipation III) hepatotoxicity A) I only B) III only C) I and II D) II and III E) I, II, III 14- Pharmacy tech received a phone call from your pharmacy customer. She has swallowed 10 tablets of acetaminophen 500 mg. What is appropriate recommendation? A) Suggest to take lots of fluids B) refer to emergence C) ask to contact poison control centre D) ask wait and see E) give N acetyl cysteine 15- What is the initial symptoms of acetaminophen toxicity? A) stomach upset B) hepatotoxicity C) headache D) confusion E) vomiting A child admitted to hospital following ingestion of overdose of aspirin and some other OTC sleeping pills, which contain antihistamines, and anticholinergic drugs. The child is in coma and flushed and flaccid. Her pupils were fixed and dilated. The lab results indicated her blood aspirin drug levels are 5.8 H is 6. Her an symptoms r ne also include deep and rapid respiration. 16) The symptoms of deep and rapid respiration may lead to: A-Hypertension B-Respiratory acidosis C-Respiratory alkalosis D-Increase in temperature E-Reduction in temperature Ans-C Tips: Respiratory Acidosis. This occurs due inadequate ventilation of CO 2 by lungs.

Copyright © 2000-2016 TIPS Inc. Unauthorized reproduction of this manual is prohibited. This manual is being used during review sessions conducted by PharmacyPrep.

55-3

Pharmacyprep.com Predisposing factor for respiratory acidosis:, Asthma, Beta blockers, Sleep apnea, CNS depressants, Pulmonary edema or embolism, Cardiac arrest RESPIRATORY ALKALOSIS. Due to increase in excretion of CO 2 However this condition is not very common. 17) As result of aspirin mechanism, the child would likely to progress? A-Metabolic acidosis B-Metabolic alkalosis C-Renal failure D-Hyperglycemia E-None of the above Ans-A 18) In order to increase the excretion of aspirin from body, which of the following should be administered: A-Probenecid B-Ammonium chloride C-charcoal D-Ipecac E-Sodium bicarbonate Ans-E

ANSWERS: 1- D 2. C 3- A Tips: Stimulants laxative are the drug of choice for the treatment of constipation associated with opioids use. 4-B 5- B 6- B 7- B 8- E Copyright © 2000-2016 TIPS Inc. Unauthorized reproduction of this manual is prohibited. This manual is being used during review sessions conducted by PharmacyPrep.

55-4

Pharmacyprep.com

9- C 10- E Tips: serious toxicity is unlikely following large doses of amoxicillin. Acute ingestion of large doses (250 mg/kg) of amoxicillin may cause nausea, vomiting, diarrhea, abdominal pain. Acute oliguric failure and hematuria may occur following large doses Reference: CPS 2009 11- C Tips: Nystatin suspension have no systemic absorption, so overdose monitor symptoms so wait and see. 12- D 13- E 14- C 15- E

Copyright © 2000-2016 TIPS Inc. Unauthorized reproduction of this manual is prohibited. This manual is being used during review sessions conducted by PharmacyPrep.

55-5

Pharmacyprep.com

PHARMACY PREP PHARMACY COMMUNICATION SKILLS 1) Which of the following is the most effective communication skill? I-Verbal skills II-Nonverbal listening skills III-Verbal skills only A-I only B-III only C-I and II only D-II and III only E-All of the above 2-A pharmacist is required to give knowledge of drugs and supply the patient with enough pieces of information to overcome the patient ignorance. This is done by: a) Written information only b) Verbal information only c) Verbal and written information d) Magazines and folders information e) Video information 3-The most effective communication skills include? A-Written only B-Verbal and written C-Verbal and non-verbal D-Verbal only E-Listening only 4-All of the following communication distractions, except: A-Telephone B-Language C-Cell phone D-Music E-Television 5) A patient who does not speak English and you don't understand his language. How do you resolve this problem? A-Find a interpreter or translator B-Patient problem do not worry C-Pharmacist problem learn his language D-Both pharmacist and technician problem E) Use non verbal techniques

Copyright © 2000-2016 TIPS Inc. Unauthorized reproduction of this manual is prohibited. This manual is being used during review sessions conducted by PharmacyPrep.

56-1

Pharmacyprep.com 6-A 90-year-old senior patient cannot hear, how to counsel? A-Speak louder B-Counsel in quite area of pharmacy C-Give recorded tape of counseling D-speak vivid and closer to customer E-Don’t speak to customer 7) As a pharmacist you should have speaking skills. All the following are true except: A) Speak in a monotonous voice pitch. B) Speak with precise language. C) Write when verbal is not sufficient. D) Greet the patient. 8) All the non verbal skills to use during telephone conversations except: A) Give your full attention to phone call. B) Put the patient on hold if you are busy. C) Be prepared. D) Smile 9) In non verbal communication you should: A) Use appropriate body language. B) Look down repeatedly. C) Tilt to one side. D) Have the barriers between you and the patient like at desk or counter. 10) A patient coming to take OTC cough syrup and pharmacist noticed that patient took same medication yesterday, what you should do? a. Tell her no more medication available b. Ask her what medical conditions she needs the medication c. Give her the name of the addiction group d. Call the police 11) If you open a pharmacy in multicultural area, what will you do increase sales? A) Display signs of different culture B) Hire staff of different cultures C) Offer only one culture service D) Refuse all the customer who disagree with opinion E) Offer services in different languages 12) What is the best marketing in pharmacy? A) Advertise in national TV B) Advertise in national news paper C) Advertise door to door in neighborhood Copyright © 2000-2016 TIPS Inc. Unauthorized reproduction of this manual is prohibited. This manual is being used during review sessions conducted by PharmacyPrep.

56-2

Pharmacyprep.com D) Advertise in mall 13) What are the characteristics of culturally competent pharmacist? A) Understanding other cultures and respecting sensitive issues. B) Offer services to all customers of different belief system C) Offer privacy and confidentiality to customers of their personal information D) All of the above 14) Customer of your pharmacy with kids. Parents discipline their kids but they are beating and spanking? What to do? A) Talk to parents and kids B) Call police and inform about it C) Call children society to protect kids D) Ignore and let them go 15) Adult women come to your pharmacy to buy analgesics for her wounds. She seems to physically and sexually abused what to do? A) Call police B) Ask her to go emergence C) Refer to doctor D) Give her police phone number to contact E) Ask her to contact sexually abused support groups 16) Which of the following is NOT an environmental barrier? A) Background noise B) Telephone rings C) Height of pharmacy counter D) Dealing with a depressed patient E) Loud music

ANSWERS: 1) C 2- C 3- B 4- B Tips:The language can be barrier. However it is not distraction. 5. A Copyright © 2000-2016 TIPS Inc. Unauthorized reproduction of this manual is prohibited. This manual is being used during review sessions conducted by PharmacyPrep.

56-3

Pharmacyprep.com

6- D 7. A Tips: It is boring to speak in a monotonous language 8. B 9. A 10) B 11) E 12) C 13) D 14) A 15) E 16) D

Copyright © 2000-2016 TIPS Inc. Unauthorized reproduction of this manual is prohibited. This manual is being used during review sessions conducted by PharmacyPrep.

56-4

www.pharmacyprep.com Pharmacy PREP ETHICAL STANDARDS 1. The ethical standard beneficence in pharmacy practice means: A. Pharmacist act with justice B. Pharmacist act with fairness C. Pharmacist act with honesty and without deception D. Pharmacist does well to patient E. Pharmacist do prevent harm 2. The ethical standard nonmaleficence in pharmacy practice means: A. Pharmacist act with justice B. Pharmacist act with fairness C. Pharmacist act with honesty and without deception D. Pharmacist does well to patient E. Pharmacist do prevent harm 3. The ethical standard veracity in pharmacy counselling means: A. Pharmacist act with justice B. Pharmacist act with fairness C. Pharmacist act with honesty and without deception D. Pharmacist does well to patient E. Pharmacist do prevent harm 4. The ethical standard justice in pharmacy practice means: A. Pharmacist act with equality B. Pharmacist act with fairness C. Pharmacist act with honesty and without deception D. Pharmacist does well to patient E. Pharmacist do prevent harm 5. Patient cannot afford to buy new medication that is not covered by his insurance plan. Pharmacist called his doctor and asked him to change the medication into another medication covered by the patient’s insurance plan. Which ethical principle did the pharmacist follow? A. Beneficence B. Veracity C. Justice D. Nonmaleficence E. Fidelity

Copyright © 2000-2016 TIPS Inc. Unauthorized reproduction of this manual is prohibited. This manual is being used during review sessions conducted by PharmacyPrep.

57-1

www.pharmacyprep.com

6) A pharmacist stops counseling a patient who repeatedly takes the medication incorrectly. Which ethical principle does pharmacist violate? A) Justice B) Beneficence C) Nonmaleficence D) Autonomy E) Veracity 7. Pharmacy professional ethical standards include nonmaleficence, which means: A. Pharmacist should act with honesty and veracity B. Pharmacist should act with fairness C. Pharmacist should prevent harm D. Pharmacist should place the benefit of the patient above all else. E. Pharmacist should respect patient choice. 8. A physician phones a pharmacist asking him not to tell the patient about the side effects of a drug because if he does, the patient will not take it. The pharmacist complies. Both the physician and pharmacist are seeking which ethic for the patient? A. Nonmaleficence B. Beneficence C. Autonomy D. Veracity E. Justice 9. A physician phones a pharmacist asking him not to tell the patient about the side effects of a drug because if he does, the patient will not take it. The pharmacist complies. Both the physician and pharmacist have violated which ethic principle? A. Nonmaleficence B. Beneficence C. Autonomy D. Veracity E. Justice 10. A pharmacist who refuses to counsel and AIDS patient violates which ethic? A. Nonmaleficence B. Beneficence C. Autonomy D. Veracity E. Justice

Copyright © 2000-2016 TIPS Inc. Unauthorized reproduction of this manual is prohibited. This manual is being used during review sessions conducted by PharmacyPrep.

57-2

www.pharmacyprep.com 11. A father of teenage girl approach to your pharmacy. Asks if her teenage 16 yo daughter using oral contraceptives? What ethical dilemma of pharmacist? A. confidentiality and veracity B. confidentiality and autonomy C. confidentiality and beneficence D. confidentiality and fidelity E. confidentiality and justice 12. A father of teenage girl approach to your pharmacy. Asks if her teenage 16 yo daughter using oral contraceptives? What ethical dilemma of pharmacist? A. Veracity and confidentiality B. Beneficence and confidentiality C. Justice and confidentiality D. Nonmaleficence and confidentiality E. Autonomy and confidentiality 13. When can you break confidentiality? A. patient who has HIV with partner B. Patient inmate who broke the law C. Patient who has Alzheimer's with care giver D. A patient could be harmful to themselves and community risk Ans. D 14. What is the principle of confidentiality does not break if a pharmacist disclose the following medical conditions to the society? A. diabetes B. Harm full C. HIV D. high BP E. cancer Ans. B 15. In which of the following situation you can break confidentiality of patient? A. patient is suffering from HIV B. patient is on parole C. suspect patient can harm himself D. patient is in jail Ans. C 16. A 30 yo women comes to your pharmacy to buy oral contraceptives. While conversation she says she found antiretroviral drugs in her husband bag. If her partner has HIV. Your medication record show patient is HIV? What is appropriate to do? I) Tell her to take HIV tests II) Tell her to talk his doctor III) Tell her that she got HIV infection from her partner A. I only B. III only C. I and II only Copyright © 2000-2016 TIPS Inc. Unauthorized reproduction of this manual is prohibited. This manual is being used during review sessions conducted by PharmacyPrep.

57-3

www.pharmacyprep.com D. II and III only E. All are correct 17. Can you release information to a prescriber if you have firsthand knowledge from the third party or DUR about double doctoring? A. Patient information is confidential B. Let third part inform to prescriber C. Yes, you can inform to prescriber because it is in circle of care D. Ask patient first, before informing doctor 18. Pam is working in rural pharmacy. She is friend with all colleagues, including Chris. Chris had vision problems in the past, but underwent laser surgery and now is ok. He does not work pharmacy anymore, but Pam is still has his profile in her computer. Should she date him? 19. Patient is using Terbutaline inhaler a lot. The pharmacist failed to advise patient to see his doctor to add corticosteroids therapy to manage his condition. What is violated? A. Beneficence B. Nonmaleficence C. veracity D. autonomy E. Paternalism 20) A patient has diagnosed with terminal ill cancer. His doctor advised him about his serious illness. What ethical principle is followed? A. Beneficence B. Nonmaleficence C. veracity D. autonomy E. Paternalism 21. Which of the following is express consent? I) given verbally II) written II) given electronically A) I only B) III only C) I and II only D) II and III only

E) All are correct

22. A pharmacist Joe is being restricted from practice because of history of substance abuse. Based on what the college makes this decision? A. Incompetent B. Incapacity C. Negligence D. Mis-conduct E. accountability 23) A returned medication is placed on shelf by the technician? what you have to as staff pharmacist? A) talk to technician B) inform to manager C) complain to college of pharmacist D) Discuss in meeting

Copyright © 2000-2016 TIPS Inc. Unauthorized reproduction of this manual is prohibited. This manual is being used during review sessions conducted by PharmacyPrep.

57-4

www.pharmacyprep.com

24) Which of the following functions cannot be delegated to pharmacy technician? A) Preparing prescription B) Checking expiry date C) Demonstrating a device to customer D) Selecting a over the counter drug to a patient E) Providing a disease literature to customer 25) Pharmacist suspects a junior technician preparing sterile IV preparations. However technician did not followed proper sterile prep proper technique. What is appropriate to do? A. Discard the sterile preps and redo new batch B. Tell the tech to follow standard techniques C. Inform the pharmacy manager D. Document and ask technician to sterile prep training E. None of the above 26. While preparing prefilled syringes, in laminar airflow hood. After packaging you find some syringes leaking. What you have to do? A. Call the company complain B. Discard all the batch leaking syringes and document C. Select the good ones and through the damage D. Just discard one leaking syringe from the batch and rest can be used E. Continue processing of this batch of syringe but investigate to prevent leakage again 27. A doctor is asking a patient to sign a consent form for a particular treatment. Which of the following applies? A. Inform consent B. Implied consent C. Express consent D. All of the above Ans. C 28) A patient comes to your pharmacy with a prescription of his wife who is not covered under his prescription drug plan. He requests you to process her prescription under his name. He will pay cash for the cost and later send the receipt to his insurance company to process the claim? A) it is unethical B. It is illegal C. It is ok D. He can claim from insurance

Copyright © 2000-2016 TIPS Inc. Unauthorized reproduction of this manual is prohibited. This manual is being used during review sessions conducted by PharmacyPrep.

57-5

www.pharmacyprep.com

29. Which of the following is incorrect combination? A. Autonomy = Right of self determination B. Beneficence = doing good C. Nonmaleficence = Preventing harm D. Justice = unequal distribution of benefits and burden E. Veracity = Honesty without deception 30. This principle expresses the concept that pharmacist have a duty to be honest and trustworthy in their dealings with people. A. Principles of Ethics B. Code of Professional Conduct C. Principle of veracity ("truthfulness") D. Principle of honesty 31. A physician write prescribed for own use or family member for emergency? A. Unethical B. Illegal C. physician can prescribe can prescribe in emergency D. physician to refer to emergency 32. A very frequent customer at your pharmacy, request for 15 days supply of Lipitor 10 mg tablets. Next few hours he has to take a flight for vacation. Currently all walk-inclinics are closed and his physician is not available for prescription. Customer has no prescription and no refills of Lipitor? pharmacist dispensed meds. what ethical principles were followed (uphold)? A. Beneficence B. Nonmaleficence C. veracity D. Justice E. Autonomy 33) A terminally ill cancer patient. Doctor did NOT informs patient that patient is seriously ill and may die in few weeks? What ethics violated? A) Beneficence B) Veracity C) Justice D) Nonmaleficence E) Fidelity 34) A police officer comes to your pharmacy, wants a patient profile of medication, who has been caught on street drugs and in traffic rules violation? What is appropriate action? A) Ask police ID and patient ID and give patient profile B) Ask police ID and if he is real police officer give patient profile C) Refuse to give any patient information to police D) Ask police officer if he has any court letter (search warrant) or letter of authorization from patient to release patient information. Copyright © 2000-2016 TIPS Inc. Unauthorized reproduction of this manual is prohibited. This manual is being used during review sessions conducted by PharmacyPrep.

57-6

www.pharmacyprep.com

35) A lawyer calls to your pharmacy wants to get medication information of a divorced spouse. Children? What to do? A. Talk to lawyers, client and provide information B. Refuse to provide information C. Children information only can be shared by parents D. Lawyer should provide official letter of patient authorization from adults Ans. d 36) A phone call from emergency medical services, wants to a patient profile of patient who is found unconscious in his apartment? What is appropriate action? A) Ask ID of EMS and refuse to give patient information B) Never trust any phone calls ask EMS to come personally to pharmacy. C) Verify and give patient profile to EMS, it is in best interest of patient. D) Ask EMS to contact to doctor of a patient to get information E) Ask EMS to contact family member of patient. 37. A frequent customer, on benzodiazepine for long time, his doctor away and there are no walk in clinics open. He wants to get some pills advance? What you do? A. talk to that person B. ask patient to see other doctor to get prescription C. advance benzodiazepine D. Refer to pharmacist 38. In what situation pharmacist call police? I) forged prescription II) Break - in happened III) narcotic theft A. I only B. III only C. I and II only D. II and III only

E. All are correct

39) A father of two adults children and wife customer of your pharmacy, wants you to print receipts of their medications for filing tax returns. A) Get the authorization from wife and adult children B) Receipts only without specifying medication just total amount can be printed to father C) Refuse to provide receipts D) Ask to contact revenue Canada Ans. A

Copyright © 2000-2016 TIPS Inc. Unauthorized reproduction of this manual is prohibited. This manual is being used during review sessions conducted by PharmacyPrep.

57-7

www.pharmacyprep.com 40. A physician called to pharmacy to get patient record of medications patient investigational drugs receipt. Patient is under the care of the doctor. Pharmaceutical company already paid to that doctor and as technician what you have to reply? A. need patient authorization B. If it is not legal way to give to doctor C. print out investigational drug receipt to the doctor D. tell doctor to contact with pharmaceutical company 41) Who sets the prices of patented medications? A) Patented Medicine Price Review Board (PMPRB) B) Health Canada C) Manufacturer D) Pharmacist associations 42. A patient consent may be essential in all, except? A. To collect patient information B. To release patient information to third parties C. To publish clinical trial results of a anticancer drug D. To prepare prescription by the technician of your pharmacy E. None of the above 43. A 16 yr old girl brings a prescription of oral contraceptive pills. However, her parents calls to your pharmacy and warns you to not dispense OCP to their daughter because using contraception is against of their belief system. What parents actions are: A. Justice B. Paternalism C. Fidelity D. Beneficence E. Veracity

44) A woman comes to your pharmacy, ask if her husband has HIV? As your pharmacy has her husband profile. He is HIV positive? What to do? A) Do check for yourself and speak to your husband B)Tell her yes! he has HIV C) Don't tell her the truth D) Tell her not to worry, HIV is treatable Ans. a 45) The pharmacist preserves the confidentiality of information about individual patient acquired in the course of his or her professional practice, and does not divulge this information except where authorized by the patient or required by law. Which of the following federal regulation protects personal health information of patient? A)Canadian health act B)Therapeutic directorate, health Canada C)Institute of safe medication practices D) Personal Health Information Protection Act E) All of the above Copyright © 2000-2016 TIPS Inc. Unauthorized reproduction of this manual is prohibited. This manual is being used during review sessions conducted by PharmacyPrep.

57-8

www.pharmacyprep.com Ans: D

46. Health information can be shared between? A. nurse B. doctor C. spouse D. circle of care Ans. d ANSWERS: 1. D 2. E 3. C 4. B 5. A 6. C 7. C 8. B 9. D 10. E 11. A 12. A 13 A 14. C 15. A 16. C

Copyright © 2000-2016 TIPS Inc. Unauthorized reproduction of this manual is prohibited. This manual is being used during review sessions conducted by PharmacyPrep.

57-9

www.pharmacyprep.com 17. C Tips. Information can be shared in circle of care (if patient is under the care of a doctor, pharmacist, nurse or caregivers). 18. 19. B Tips. By not preventing harm. The nonmaleficence is violated 20. C 21. E Tips. The consent can be obtained by three types express, implied, and informed. 22. B Incompetent: lack of knowledge, skills, judgment Incapacity: substance abuse or accidents Negligence: working lower than standard Misconduct: unethical Accountability: legally and morally liable when carrying out duty Advance directives = legal will 23. A 24. D Tips. According to pharmacy technician standards of practice. A regulated technician or pharmacy assistant cannot recommend prescription or non prescription therapies to patient. Any clinical recommendations or counseling is CANNOT be performed by pharmacy technicians. 25. A 26. B 27. C 28. B 29. D 30. C 31. C

Copyright © 2000-2016 TIPS Inc. Unauthorized reproduction of this manual is prohibited. This manual is being used during review sessions conducted by PharmacyPrep.

57-10

www.pharmacyprep.com 32. A 33. B 34. D 35. C 36. C Tips: It is circle of care, you can share information after confirmation. 37. B 38) E 39) B 40) C 41. A 42. D 43. B 44. A pharmacist believes, using Plan B is against his faith. If a customer wants to use plan B. would you dispense? 45. A physician write prescription for own use or family member for emergency? 46. A very frequent customer of your pharmacy, requests for 15 days supply of Lipitor 10 mg tablets. Next few hours he has to take a flight for vacation. Currently all walk in clinics are closed and his physician is not available for prescription. Customer has no prescription and no refills of Lipitor? If you dispensed what ethics are followed?

47. A patient of your pharmacy approaches to refill salbutamol inhaler. Patient asthma is getting worse and using more frequent salbutamol. However pharmacist did not suggest patient to see the doctor to get steroid inhaler prescription. What ethical principles have been violated? 48. There are two customers of pharmacy, one customer all family members come to your pharmacy. Other customer family members go to other pharmacy. A pharmacist

Copyright © 2000-2016 TIPS Inc. Unauthorized reproduction of this manual is prohibited. This manual is being used during review sessions conducted by PharmacyPrep.

57-11

www.pharmacyprep.com takes NO copayment from a customer who comes with all family members, where as customer comes as single persons pays co-payment, What ethical principle is violated? 49. A pharmacist refuses to dispense plan B, because of pharmacist beliefs are against of using contraception? 50. A family two adults and one 16 yo daughter are customer of your pharmacy. one of the family member comes to your pharmacy, ask for list medications their family members using for the past 6 months. What to do?

A cancer patient has one month to live. Doctor did not inform the patient about cancer. What ethical principles is broken? A) Beneficence B) Veracity C) autonomy D) Confidentiality E) Paternalism

Patient brings a new prescription. Pharmacist suspect the signature of prescription is forged? What is appropriate action? A) Send back patient to doctor B) Directly call the doctor to confirm C) Check previous records of prescription to compare the signature. D) Fax prescription back to physician

A Friday evening a neighboring storeowner brings a prescription of tadalafil 10 mg. Your pharmacy is busy and people are lined up. He gets off the line and quietly ask pharmacist that he is in hurry and catch up the traffic to reach up cottage for weekend. What is appropriate to do? A) Ask permission other people in line B) Tell him to line up C) Dispense him private counseling area not to repeat. D) Tell him to go other pharmacy

Copyright © 2000-2016 TIPS Inc. Unauthorized reproduction of this manual is prohibited. This manual is being used during review sessions conducted by PharmacyPrep.

57-12

www.pharmacyprep.com

Covenant relationship?

Failing to maintain a standards of practice of profession is? a. illegal B. unethical C. professional misconduct D. Pharmacy act A patient approach at your pharmacy for OTC drugs. Patient expressed that he did not like this medication. Pharmacy tech got upset and verbally abused patient? a. illegal B. unethical C. professional misconduct D. Pharmacy act

Pharmacy dispensing fee is? A. A federal law B. NAPRA C. By-law D. Controlled Act

Copyright © 2000-2016 TIPS Inc. Unauthorized reproduction of this manual is prohibited. This manual is being used during review sessions conducted by PharmacyPrep.

57-13

www.PharmacyPrep.Com

Information resources

PHARMACY PREP PHARMACY PRACTICE INFORMATION RESOURCES 1. The Canadian pharmaceutical reference CPS stands for: A. Compendium of pharmaceutical Sciences B. Compendium of Pharmaceutical Supplements C. Compendium of Pharmaceutical Society D. Compendium of Pharmaceutical Specialties E. Compendium of Pharmaceutical Sources 2. A patient presents a prescription that requires the extemporaneous preparation of a buffered eyedrop. Which of the following is the source for information on the correct buffering of the eyedrop? A. Facts and comparisons B. The Merck manual C. Pharmacy PREP index D. Remington’s pharmaceutical sciences E. Physicians desk reference 3. One of your regular patients comes to the pharmacy in search of Dogmatil® for her brother. He has just arrived from Singapore for a visit, but his medication was lost with his luggage. She says her brother needs the medication for a CNS condition and shouldn’t go without it. She wants to know if he can get Dogmatil® here. Which of the following appropriate reference: A. Canadian hospital journal B. American hospital journal C. Compendium of pharmaceutical specialties D. Martindale, The complete drug reference E. National formulary 4. Cochrane collaboration databases or collaborative library is the reference source of? A. Evidenced based medicine information B. A Canadian pharmacy journal C. Collaboration of medical professionals D. A natural products database E. The group of people with same characteristics Ans. A

Copyright © 2018 TIPS Inc. Unauthorized reproduction of this manual is prohibited. This manual is 58-1 being used during review sessions conducted by PharmacyPrep.

www.PharmacyPrep.Com

Information resources

5. Compendium of pharmaceutical specialties is useful resource for locating which of the following information? A. Approved indication for all prescription pharmaceuticals in world B. Detailed monographs for prescription pharmaceuticals available in Canada C. Clinical recommendations for dental prophylaxis of bacterial endocarditis D. Canadian consensus guidelines on drug therapies used to treat asthma and COPD E. Recommendation patient self selection products in Canadian pharmacies

6. What information is NOT present in compendium of pharmaceutical specialties? A. Monograph of prescription medications in Canada B. Clinical information of drug and food interactions C. Immunization schedules D. Recommendation of OTC drugs to treat headache E. Antibiotics and antiviral monographs 7. What information is not obtained from patient self care reference? A. Non pharmacology advise for foot care B. Red flags that require referral to doctor C. Non prescription of medication D. Treatment of cold sores E. Choosing a brand of calcium supplements 8. What information is not obtained from therapeutic choices? A. treatment options for initial therapy B. treatment options for alternate drug of choices C. Laboratory investigations for diagnosis of disease D. treatment options for self selection drugs E. treatment options of pneumonia 9. What is suitable reference source to locate information about immunization schedules in Canada? A. Martindale B.USP-DI C. Handbook of immunization D. Compendium of Pharmaceutical Specialties E. Vaccine manufacturer 10. If a patient comes and asking for drug information, you would do? A. search for compendium of pharmaceutical specialties B. Ask doctor C. search on Internet D. Call information center E. Search Medline 11. Which of the following are the most credible information resources? A. scientific journals containing clinical studies Copyright © 2018 TIPS Inc. Unauthorized reproduction of this manual is prohibited. This manual is 58-2 being used during review sessions conducted by PharmacyPrep.

www.PharmacyPrep.Com

Information resources

B. indexes C. abstracts D. text books E. TV and radio, internet 12. What information is NOT present in CPS? A. Prescription drugs available in Canada B. Drugs in dentistry C. Drugs in pregnancy D. Self care for nausea and vomiting E. Prescription drug monographs 13. What information is NOT present in patient self care? A. Antibiotics B. Mouth hygiene in halitosis C. choosing a sunscreen D. self care for nausea and vomiting E. treatment of nausea and vomiting in pregnancy 14. What is reference is used to recommend to initial therapy for hypertension? A. CPS B. Patient self care C. hypertension clinical guidelines D. Therapeutic choices E. USP-DI vol.1 15. Which of the following resources would provide the most up-to-date answer when checking if a drug is marketed in Canada? A. e-CPS B. CPS C. PubMed D. Martindale’s E. USP-DI vol.1 16. CPS is? A. primary literature B. secondary literature C. tertiary literature D. All of the above ans. c

Copyright © 2018 TIPS Inc. Unauthorized reproduction of this manual is prohibited. This manual is 58-3 being used during review sessions conducted by PharmacyPrep.

www.PharmacyPrep.Com

Information resources

17. A prescriber wants to know drug interaction of simvastatin with clarithromycin. What is initial reference pharmacist should search? A. primary research journal B. Internet search C. Compendium of pharmaceutical specialties D. Consumer product monograph of simvastatin E. Product package insert Ans. c Tips: Begin search from Tertiary or Secondary -----> Primary

18. What is not found in Compendium of pharmaceutical specialties? A. iron supplements B. Vaccine monograph C. Antacids drug interactions with tetracycline D. Calcium supplements interactions with levothyroxine E-non prescription drugs and self care Ans. e 19. What is true about off labeled drug? except A. Can be found in Martindale B. Can be found in Medline or Pubmed C. can be prescribed by a physician based published scientific evidence D. Pharmacist should refuse dispensing off label used drugs E. Patient should be informed about off label use of drug Ans. D

ANSWERS: 1. D 2. D 3. D 4. A 5. B 6. D

Copyright © 2018 TIPS Inc. Unauthorized reproduction of this manual is prohibited. This manual is 58-4 being used during review sessions conducted by PharmacyPrep.

www.PharmacyPrep.Com

Information resources

7. E Tips: brands of over the counter products can be found in compendium of self care products (CPSP) 8. D 9. D 10. D 11. A 12. D 13. A 14. C 15. A

Copyright © 2018 TIPS Inc. Unauthorized reproduction of this manual is prohibited. This manual is 58-5 being used during review sessions conducted by PharmacyPrep.

PharmacyPrep.Com

Medication Errors PHARMACY PREP MEDICATION ERRORS

1. Verbal prescriptions of drug "names sound like" drugs, can cause medication errors. What should manager do to decrease the error EXCEPT: A. Put them away from each other B. Put label on vial (call for error) C. Put in computer system alarm to work on dispensing D. Technician ask patient for what disease being treated for E. Make two persons double check before dispensing 2-Busy pharmacies a long line up in 15 customers, a person came he is insisting and fighting with pharmacist, do dispense him first? A-ignore that customer B-dispense him first C-don’t dispense D-just talk to him and explain there are other before you. E-Call police 3) A very angry customer comes to your pharmacy and complains that you have dispensed wrong medication. What is initial appropriate action? I-offer private counselling area II-check or verify medication III-if there is dispensing error, apologize and correct it A) I only B) III only C) I and II D) II and III E) I, II, III 4) In case of recall products, what is first thing to do? A-Call all patient still using it and inform about recall B-Inform the manager follow procedure for recall product C-Post a sign in the pharmacy regarding recall product. D-Inform doctor to stop dispensing this product in the future E-Discard all product in waste container 5) Angry person walks in your pharmacy and complains that you have dispensed wrong medications? What is appropriate first step in resolving this problem? A-Apologise patient for dispensing error B-Calm down patient C-Acknowledge his anger, ask and verify what is error D-Ignore and let him vent his anger E-Acknowledge his anger and refer to doctor

Copyright © 2000-2016 TIPS Inc. Unauthorized reproduction of this manual is prohibited. This manual is being used during review sessions conducted by PharmacyPrep.

59-1

PharmacyPrep.Com

Medication Errors

6) If dispensing error occur in pharmacy. Who are the responsible? I-Doctor II-Pharmacist III-Pharmacy Technician A) I only B) III only C) I and II D) II and III E) I, II, III 7) If prescription error occurred in pharmacy on phone. Who are the responsible? I-Doctor II-Pharmacist III-Pharmacy Technician A) I only B) III only C) I and II D) II and III E) I, II, III 8) Angry customer comes to your pharmacy. Says dispensing error occurred, which of the following first step should be done? A) Apologize customer B) Calm down and verify prescription error C) Calm down angry customer D) Apologize and return new drug E) Refuse to accept any error 9) Which of the following should first checked by the pharmacist to minimize the dispensing error? A) Patient name B) Dr name C) DIN D) Date of birth E) Drug name 10) A patient comes to your pharmacy and complains that he got 10 less tablets of oxycontin for his prescription. What is the better strategy to minimize these errors. A) always count tablets in front of patient B) double or triple count in pharmacy before dispense C) if some patient complains just give extra tablets D) narcotics cannot be dispensed extra tablets E) Check inventory and reassure patient we double check 11) Which of the following is the best method of minimizing calculations errors in preparations? A) Choose a person has multiple years of experience B) Choose a inexperience a new pharmacy graduate C) Ask a pharmacist or technician to double check the calculation before preparation D)The calculations should be performed by calculators E) None of the above 12) Which of the following medication error would not risk harm? A)Wrong drug B) Wrong instruction C)minor spelling mistakes D)inappropriate auxiliary labels E) expired medications

Copyright © 2000-2016 TIPS Inc. Unauthorized reproduction of this manual is prohibited. This manual is being used during review sessions conducted by PharmacyPrep.

59-2

PharmacyPrep.Com

Medication Errors

13) A pharmacy dispensing error dispensed Pradox (dabigatran) instead of Plavix (clopidogrel). If patient used. Which of the following problem expected? A) Increase risk myocardial infarction B) Increase risk of platelet aggregation C) Increase risk of bleeding D) Increase risk of blood clot formation 14) If patient used Lasix (furosemide) instead of Losec (omeprazole). What problem is expected? A) hyperglycemia B) hyperthyroidism C)hypotension D)Hypercalcemia E)None 15) A pharmacy technician prepared a prescription of Pradox instead of Plavix. What is the appropriate strategies to prevent this error. I) Advise technician to change to Plavix (clopidogrel) II) Educate technician on drug name sound similar III) Read label to patient while dispensing A) I only B) III only C) I and II D) II and III E) I, II, III 16) A pharmacy intern receive a phone call from doctor for a new prescription. Dr want to prescribe Losec. However pharmacy intern documented as Lasix. What is appropriate to minimize this error? A) Only pharmacist should take phone call for a new prescription B) Pharmacy intern should require more training on sound like name drugs C) Ask doctor to give generic name for the sound like name drugs D) Ask doctor to fax prescription for sound like name drugs E) Ask doctor to written prescription for all new prescriptions 17) Which of the following is NOT dangerous drug abbreviation? A) od B) qid C) daily D) IU E) qd 18) What is recommended to write instead of "IU" A) iu B) units C) international units D) IU E) I.units 19) What is the best recommendation to write prednisone and prednisolone to avoid errors. A) prednisone and prednisolone B) predniSONE and predniSOLONE Copyright © 2000-2016 TIPS Inc. Unauthorized reproduction of this manual is prohibited. This manual is being used during review sessions conducted by PharmacyPrep.

59-3

PharmacyPrep.Com

Medication Errors

C) predNISone and predNISOLone D) PREDNISONE and prednisolone E) predsnisone and PREDNISOLONE 20) What is correct tallman letters for diphenhydramine and dimenhydrinate A) dimenHYDRINATE and diphenHYDRAMINE B) diMENhydrinate and diPHENhydramine C) DIMENHYDRINATE and diphenhydramine D) dimenhydrinate and DIPHENHYDRAMINE 21) Which of the following abbreviation least likely cause error? A) q.d B) QD C) qid D) q6h E) I.U 22)What agencies in Canada are associated in reporting medication incidents? A) Institute of safe medication practices (ISMP) B) Health Canada’s “Med effect” program C) Canadian Coalition for Medication Incident Reporting D) All of the above 23. The practice of writing part of a drugs name in upper letter to help distinguish? A. Dispensing error of generic and brand name drugs B. Packaging error of medication C. Dose errors of drugs D. To prevent incidents of Sound alike and look alike drugs E. drugs name with same first and last letter 24. Health Canada MedEffect program is related to? A. adverse drug reaction monitoring B. advisories, warnings and recalls C. Safer health for Canadians D. To develop actions and strategies to prevent incidents E. National System for incident reporting

Copyright © 2000-2016 TIPS Inc. Unauthorized reproduction of this manual is prohibited. This manual is being used during review sessions conducted by PharmacyPrep.

59-4

PharmacyPrep.Com

Medication Errors

Medication Errors ANSWERS: 1. Ans: D 2- Ans: D 3) Ans: E 4) Ans: B 5) Ans: C 6) Ans: D Tips: Vicarious liability of pharmacist of dispensing 7) Ans: C Tips: pharmacist should contact doctor to correct prescription. 8) Ans: B 9) Ans: A 10) Ans: B 11) Ans: C 12) Ans: C 13) Ans: C Tips: Pradox is a new anticoagulant dabigatran act as factor Xa inhibitor. However Plavix is clopidogrel an antiplatelets drugs. So increase risk of bleeding 14) Ans: C Tips: Lasix is furosemide can cause hypotension. Losec is omeprazole. Immodium (Loperamide) and Motilium (domperidone) Dimenhydrinate and diphenhydramine Cyclosporine and Cyclophosphomide Proscar and Prozac Lamisil (terbinafine) and Lamictal (lamotrigine) 15) Ans: E Copyright © 2000-2016 TIPS Inc. Unauthorized reproduction of this manual is prohibited. This manual is being used during review sessions conducted by PharmacyPrep.

59-5

PharmacyPrep.Com

Medication Errors

16) Ans: C 17) Ans: C 18) Ans: B 19) Ans: C Tips: Tallman letter can be used to avoid error 20) Ans: B 21) Ans: D 22 Ans:) D 23. Ans: D 24. Ans: B

Copyright © 2000-2016 TIPS Inc. Unauthorized reproduction of this manual is prohibited. This manual is being used during review sessions conducted by PharmacyPrep.

59-6

www.pharmacyprep.com

Sterile Preparations PHARMACY PREP Health Promotion and Disease Prevention

1. Pharmacist offering immunization is categorized as? A. Primary care B. Secondary care C. Tertiary care D. Quaternary care E. All of the above Ans. A Tips. Levels of health care delivery systems. It can be categorized as primary, secondary and tertiary health care delivery systems. Primary care: This is first contact a person makes a with the system when a person feels the necessity of health care. This usually occurs through the family physician, pharmacist, or nurse at medical centers. Secondary care: This is specialized service from a specialist. This requires referral from primary health care levels. Tertiary care. This is specialized in diagnosing and highly technical care and treating complicated or unusual health problems. This generally takes place in hospital setting where generally diagnostic and complicated therapies can take place. 2. What is an example of activities are designed to completely prevent a disease which is categorized as primary level of health promotion activities? A. use of beta blockers to help remodel the heart in congestive heart failure B. colonoscopy to detect small cancerous polyps C. Immunization against pneumonia or influenza D. Prophylaxis antibiotics for endocarditic ans. c 3. Sunscreen protect? A. UVA B. UVB C. UVC D. All of the above Ans. B Tips. UVA aging, UVB burns, UVC is safe

Copyright © 2000-2016 TIPS Inc. Unauthorized reproduction of this manual is prohibited. This manual is being used during review sessions conducted by PharmacyPrep.

60-1

www.pharmacyprep.com

Sterile Preparations

Health Promotion and Disease Prevention Answers 1. Ans. A

Copyright © 2000-2016 TIPS Inc. Unauthorized reproduction of this manual is prohibited. This manual is being used during review sessions conducted by PharmacyPrep.

60-2

www.pharmacyprep.com

Sterile Preparations PHARMACY PREP Collaboration and Teamwork

1. Pharmacist identify a potential drug related problem for patient and then provide the prescriber with a clinical recommendation to resolve the problem, is described as? A. Advancing B. Adaptation C. Pharmaceutical opinion D. Beneficence E. Regulations Ans.c 2.Main goal of academic detailing is? A. Advancing prescription B. Providing pharmaceutical opinion C. Enhancing prescription practices D. Pharmaceutical care E. Counseling patient Ans. c 3.Unnecessary use or high dose of antibiotics cause? A. Tolerance B. Resistance C. Dependency D. Addiction E. Ineffective therapy Ans. b 4.A doctor switched patient to a new opioid therapy and started low dose. It is due to? A. Tolerance B. Resistance C. Dependency D. Addiction E. Ineffective therapy Ans. a 5. nurse practitioner can prescribe? A. injections B. narcotics C. inhalational D. all of the above ans.d

Copyright © 2000-2016 TIPS Inc. Unauthorized reproduction of this manual is prohibited. This manual is being used during review sessions conducted by PharmacyPrep.

61-1

www.pharmacyprep.com

Sterile Preparations

6. a nurse practitioner prescribed sitagliptine 50 mg daily for diabetic patient. What is appropriate? A. process, dispense and document B. tell patient to contact to doctor to get a new prescription c. call nurse and inform this is not in the scope of nurse practice D. complain of college of nursing ans. a

Copyright © 2000-2016 TIPS Inc. Unauthorized reproduction of this manual is prohibited. This manual is being used during review sessions conducted by PharmacyPrep.

61-2

www.pharmacyprep.com

Sterile Preparations

PHARMACY PREP STERILE PREPARATIONS

7) What type of laminar flow hood is used in preparation of cytotoxic (neoplastic) products? A-Horizontal laminar airflow hood B-Vertical Laminar airflow hood C-Vertical and Horizontal laminar airflow hood D-None of the above Ans-B 8) What size of filters is used in water sterilization? A-0.22 mm B-0.33 mm C-0.11 mm D-0.44 mm Ans-A

E-0.55 mm

9) What sizes of HEPA filters are used in laminar airflow hood that is used for parenteral preparations? A) 0.22 mm B-0.33 mm C-0.11 mm D) 0.44 mm E) 0.55 mm Ans-B

40) What is incorrect about hypodermic needle in sterile preparations? A) Avoid touching hub and plunger of needle B) Avoid touching bevel of needle C) gauge is diameter of syringe shaft D) Leur-lock syringes are used in cytotoxic preparation E) All parenteral preps are have to manipulate in laminar airflow hood Ans. C Tips: gauge is diameter of needle.

Copyright © 2000-2011 TIPS Inc. Unauthorized reproduction of this manual is prohibited. This manual is being used during review sessions conducted by PharmacyPrep.

44-1

www.pharmacyprep.com

Sterile Preparations

Which of the following correct size of HEPA filters are used in laminar airflow hood in sterile preps? A) 0.22 microns B) 0.22 micrometer C) 0.33 microns D) 0.33 micrometers E) none of the above

Copyright © 2000-2011 TIPS Inc. Unauthorized reproduction of this manual is prohibited. This manual is being used during review sessions conducted by PharmacyPrep.

44-2

PharmacyPrep.Com

Pharmaceutical Storage Conditions PHARMACY PREP PHARMACEUTICAL STORAGE CONDITIONS

1- You received a new drug in your pharmacy and the manufacture instructions tell you to keep this drug in a cool place. Which temperature should you keep this drug? a) 30 C 2-All of the following antibiotics extemporaneous suspensions are stored at room temperature, EXCEPT A-Metronidazole B-Clarithromycin C-Azithromycin D-Clindamycin E-Cotrimoxazole 3) Insulin is stored at? A-Room temperature B-Refrigerator C-Freezer D-Deep freezer E-Cool temperature 4) All vaccines should be stored in? A-Room temperature B-Refrigerator C-Freezer D-Deep freezer E-Cool temperature 5) What is the optimal temperature for vaccine fridge is? A. 5 °C B. 0°C C. 1 °C D. 2°C E. 3°C 6) When should pharmacy staff defrost refrigerator? A) More than 10 cm ice in the freezer compartment B) More than 5 cm ice in the freezer compartment C) More than 1 cm ice in the freezer compartment D) It should be never defrosted Ans. C 7. Storage of Dukoral should be: A. At room temp B. In the fridge C. Away from light Copyright © 2000-2016 TIPS Inc. Unauthorized reproduction of this manual is prohibited. This manual is being used during review sessions conducted by PharmacyPrep.

63-1

PharmacyPrep.Com

Pharmaceutical Storage Conditions

D. In a safe cabinet Ans (B) Tips: It is a vaccine. 8. All the drugs and supplies use in the sterile room should be unpack ? A. After bringing into the sterile room B. Before brining into the sterile room. C. Anytime and anywhere can be unpacked D. Manufacturer should not pack supplies that are used in sterile preps Ans. B 9. The temp of the fridge in the pharmacy should be recorded at least A. Once per day B. Twice per day C. Three times per day D. Once every other day Ans. (B) Tips: It should be recorded once in the morning and once in the night

PHARMACEUTICAL STORAGE CONDITIONS ANSWERS: 1- Ans: C 2- Ans: A 3- Ans: B 4- Ans: B 5. Ans: A

Copyright © 2000-2016 TIPS Inc. Unauthorized reproduction of this manual is prohibited. This manual is being used during review sessions conducted by PharmacyPrep.

63-2

PHARMACY PREP PHARMACEUTICAL CARE AND DRUG RELATED PROBLEMS 1. What is the most important focus of pharmaceutical care? A. The pharmacist B. The patient C. The prescription D. The patient chart E. Pharmacist salary 2. A 60-year-old patient with congestive heart failure who has been stabilized for 3 months on digoxin, furosemide, and potassium chloride is gradually placed on the following additional medicines. Which of these drugs is most likely to cause a problem if added to patient treatment? A. Quinidine B. Temazepam (restoril) C. Meperidine hci (demerol) D. Aspirin E. Nitroglycerin 3. What is NOT a requirement of pharmaceutical care? A. Prevention of disease B. Cure of disease C. Decrease of symptoms D. Slowing of disease progression E. Dispensing the cheaper drug 4. Which of the following does NOT included in drug related problems? A. Adverse effects of drugs B. Improper storage conditions C. Receiving too little drug D. Receiving inappropriate drug for condition E. Dispensing error 5. A frequent customer of your pharmacy medical profile includes; metaprolol 50 mg, metformin 500 mg tid, allopurinol 50 mg, ASA 81 mg daily for the past 3 yrs. Recently doctor have started atorvastatin 40 mg daily. Which of the following drug may give stomach upset? A. Metaprolol B. Metformin C. Allopurinol D. Atorvastatin E. ASA Copyright © 2000-2016 TIPS Inc. Unauthorized reproduction of this manual is prohibited. This manual is being used during review sessions conducted by PharmacyPrep.

64-1

6. A doctor has prescribed salbutamol inhaler prn for stuffy nose 3 months ago. However, patient experiencing tremor and shaky for past 6 days. What to do? A. Increase dose of salbutamol B. Decrease dose of salbutamol C. Ask patient how often use salbutamol puffs D. Refer to doctor E. Refer to emergency 7. A 65 yo COPD patient using ipratropium PRN inhalers and tiotropium once daily. Complains dry mouth and dry eye. What to do? A. refer to doctor B. recommend use frequent sip of water or ice chips C)recommend to use artificial eye drops D. refer to emergency E. B and C 8. A hypertensive patient who has COPD and who is non compliant would be best treated with which of following beta blockers? A. Propranolol B. Atenolol C. Esmolol D. Timolol 9. A regular customer of your pharmacy brings a prescription of Advair (salmeterol + fluticasone) bid. Currently patient profile has salbutamol prn and fluticasone bid. What is appropriate action to do? A. Counsel patient on how to use Advair device B. Counsel patient to space Advair from fluticasone C. Ask patient if doctor want him to discontinue fluticasone D. Ask patient to discontinue fluticasone, as the same fluticasone is present in combination product (Advair) E. Ask patient if his asthma is uncontrolled 10. A doctor has prescribed oxycontin for prn 7days after appendectomy surgery. Which of the following appropriate products to recommend? A. mineral oil B. psyllium C. sennakot/stool softener D. stool softener only E. Magnesium antacids 11. A pregnant women diagnosed UTI, and she allergic to penicillin, what is the drug of choice? A. Nitrofurontoin B. Fosphomycin C.Cephalosporin's D.Tetracycline E. Ciprofloxacin 12. A patient is using tetracycline for the treatment of acne. What is the best counseling? A. take with glass of water B. take with glass of milk C. must take empty stomach D. take with glass orange juice E. avoid taking with antacids

Copyright © 2000-2016 TIPS Inc. Unauthorized reproduction of this manual is prohibited. This manual is being used during review sessions conducted by PharmacyPrep.

64-2

13. A patient is medication shows ramipril, nadolol, ASA and rosuvastatin. What are the adverse effect may be related to ramipril? A. bradycardia, bronchospasm, cardiac decompensation B. skin rash, and proteinuria C. postural hypotension, fever, positive Coombs' test D. First dose syncope, postural hypotension, and palpitation 14. A patient is treated with hydrochlorothiazide 50 mg daily for uncomplicated hypertension should be monitored regularly for altered plasma levels of? I) increased levels of uric acid II) increase levels of glucose III) decreased level of potassium A. I only B. III only C. I and II only D. II and III only E. All are correct 15. A patient had ileostomy surgery, which of the following dosage form is not used? A. immediate release B. sustain release C. oral dosage form D. intra venous E. sublingual

Copyright © 2000-2016 TIPS Inc. Unauthorized reproduction of this manual is prohibited. This manual is being used during review sessions conducted by PharmacyPrep.

64-3

PHARMACEUTICAL CARE AND DRUG RELATED PROBLEMS ANSWERS 1. Ans: B 2. Ans: A Tips: Although digoxin and quinidine may be used together, it is well documented that administering quinidine to a patient previously stabilized on digoxin will cause serum digoxin levels to rise an average 2 to 2.5 fold. The mechanism of this interaction may involve both a displacement of digoxin from tissue-binding sites and a reduction in renal clearance of digoxin. Even though the significance of this interaction remains controversial, many clinicians suggest reducing the dose of digoxin by 50% when adding quinidine. In any case, the patient should be monitored carefully for signs of digoxin toxicity. 3. Ans: B 4. Ans: E 5. Ans: D 6. Ans: C 7. Ans: E 8. Ans: B Tips: Cardio selective BBs "EMAA". Esmolol is iv infusion. Acebutolol and Atenolol are long acting once daily. Timolol is long acting but not cardio selective 9. Ans: C 10. Ans: C Tips: stimulants laxative like senna, bisacodyl are effective to treat constipation caused by opioids. 11. Ans: A 12. Ans: C 13. Ans: B 14. Ans: E Tips: Thiazide cause "hyperGLUC" 15. Ans: C Copyright © 2000-2016 TIPS Inc. Unauthorized reproduction of this manual is prohibited. This manual is being used during review sessions conducted by PharmacyPrep.

64-4

www.pharmacyprep.com

Adverse Drug Reactions

PHARMACY PREP ADVERSE DRUG REACTIONS 1. Which of the following drugs give severe rebound hypertension? A. prazocin B. propranolol C. clonidine D. captopril E. enalapril 2. All of the drug cause orthostatic hypotension, except: A. propranolol B. prazocin C. captopril D. doxazocin E. enalapril 3. Neuroleptic malignant syndrome side effects is not associated with? A. haloperidol B. Levodopa C. clozapine D. Chlorpromazine E. Risperidone 4. Nitroglycerine patch comes off during shower, what is suitable action? A. refer to the physician B. put another patch C. reapply the same patch D. use glue and reapply the same patch E. None of the above 5. Which of the following drug should not be taken if it is not effective with first dose? A. Nitro glycerine spray B. Nitro-glycerine SL C. Beta blockers D. Nifedipine E. Sumatriptan 6. Drugs that gives bronchospasm? A. Salbutamol B. acebutolol E. Ipratropium

C. Salmeterol

D. Formoterol

7. Drugs that alter TSH levels? A. Lithium B. Amiodarone C. Levothyroxin D. L-thyroxin E. All of the above 8. All of the following drugs can cause weight gain except? A. Fluoxetine B. Paroxetine C. Risperidone 65-1 Copyright © 2000 - 2016 TIPS Inc. Unauthorized reproduction of this manual is prohibited. This manual is being used during review sessions conducted by PharmacyPrep.

www.pharmacyprep.com

Adverse Drug Reactions

D. Quetiapine E. topiramate 9. Which of the following drugs priapism side effects, patient should report to doctor? A. Trazodone B. Sildenafil C. fluoxetine D. Alprostadil E. All of the above 10. All of the following drugs can give pulmonary fibrosis, except? A. Bromocriptine B. Cisapride C. Amiodarone D. Ropinirole E. Methotrexate 11. Headache is side effects of? A. Amlodipine B. Sumatriptan E. Fluoxetine

C. Acetaminophen

12. Venous pooling is associated with? A. Statins B. Triptans C. Sodium nitroprusside E. Beta blockers

D. Sildenafil

D. Verapamil

13. Black stool and tongue can cause by? A. Loperamide B. Bismuth sub-salicylates C. Ciprofloxacin D. Tetracycline E. Erythromycin 14. The rare but serious side effect of clindamycin is? A. Constipation B. Diarrhea C. Stomach upset D. Nausea and vomiting E. Bloating 15. The most common side effect of erythromycin? A. Constipation B. Diarrhea C. GI discomfort D. Nausea and vomiting 65-2 Copyright © 2000 - 2016 TIPS Inc. Unauthorized reproduction of this manual is prohibited. This manual is being used during review sessions conducted by PharmacyPrep.

www.pharmacyprep.com

Adverse Drug Reactions

E. Bloating 16. Phenytoin is associated with? A. Gingival hyperplasia B. Diarrhea C. GI irritation D. Nausea and vomiting E. Bloating 17. Neutropenia is side effect of? A. Clopidogrel B. ASA C. Ticlopidine D. Warfarin E. LMWH 18. Angioedema is associated with? A. ACE I B. ARBs C. CCBs D. A and B E. Beta blockers 19. Lactic acidosis is associated with? A. metformin B. Chlorpropamide C. Gliclazide D. Roziglitazone E. Acarbose 20. Torse de pointes is associated with? A. Amiodarone B. quinidine C. Procainamide D. all of the above 21. What is most common side effects of SSRIs? A. Nausea and vomiting B. Sexual dysfunction C. Sedation D. GI upset E. weight gain 22. Bupropion side effects include? 65-3 Copyright © 2000 - 2016 TIPS Inc. Unauthorized reproduction of this manual is prohibited. This manual is being used during review sessions conducted by PharmacyPrep.

www.pharmacyprep.com

Adverse Drug Reactions

A. bulimia nervosa B. anorexia nervosa C. sexual dysfunction D. weight gain E. A and B 23. A 50 year old female currently she is on atorvastatin, and atenolol and Tylenol # 1 for arthritic pains. She visit pharmacy complaining of diarrhea, which of the following medication may associated to her problem: A. Atenolol B. Codeine in Tylenol C. Tylenol D. Atorvastatin E. All of the above 24. All of the following drugs may require WBC or CBC monitoring, except? A. clozapine B. ticlopidine C. levothyroxine D. propylthiouracil E. methimazole 25. All of the following drugs give constipation side effect, except? A. Atropine B. Ipratropium C. Tiotropium D. Scopolamine E. Donepezil 26. A 45 yo person using ramipril 10 mg daily for the treatment of hypertension. However, hypertension was not controlled, and now doctor have added prazosin 10 mg daily. What is the pharmacist concern? A. potassium levels B. low blood pressure C. side effects of prazosin D. drug interactions E. patient age 27. A 72 yo person on atenolol and ASA 325mg, clopidogrel 75 mg, therapy. Recently diagnosed with hypothyroidism. Doctor has recommend low dose, levothyroxine 12.5 mcg daily dose. Why is the low dose given? I)cardiovascular disease II)age III)drug interactions with atenolol, ASA and clopidogrel A. I only B. III only C. I and II D. II and III E.I,II, III 28. A customer of your pharmacy using sumatriptan for migraine headache. However, this person headache is not relieved after using sumatriptan. What is appropriate to do? A. double the dose of sumatriptan B. wait for 2 hour and try again sumatriptan C. add other triptan with sumatriptan D. Decrease dose and gradually increase the dose E. Do not use sumatriptan 65-4 Copyright © 2000 - 2016 TIPS Inc. Unauthorized reproduction of this manual is prohibited. This manual is being used during review sessions conducted by PharmacyPrep.

www.pharmacyprep.com

Adverse Drug Reactions

29. A 65 yr old Parkinson's patient is on levodopa/carbidopa therapy. Experiencing nausea and vomiting. What is incorrect? A. treat nausea vomiting using dimenhydrinate B. treat nausea vomiting using metoclopramide C. Recommend vitamin B 6 supplement D Recommend Diclectin (vitamin B 6 + doxylamine) E. Recommend ondansetron a serotonin antagonist 30. A 72 year old regular customer of your pharmacy brings a new prescription of levothyroxine 12.5 mcg daily. Recently patient is discharged from hospital after MI. Dr prescribed low dose of levothyroxine because? I) Myocardial infarction II) Age 72 yr III) Regular customer of your pharmacy A. I only B. III only C. I and II D. II and III E.I,II, III 31. A 55 yo man using warfarin 5 mg daily to treat deep vein thrombosis. Recently doctor prescribed metoprolol 50 mg bid to treat hypertension. What is the pharmacist concern? A. high dose of warfarin B. deep vein thrombosis at age of 55 y C. Metoporolol is a beta blocker D. Not a concern E. INR monitoring is required 32. Which of the following therapy is pharmacist concern? A. Patient using methotrexate but not receiving folic acid supplements B. Patient in pregnancy using carbamazepine but not receiving folic acid supplements C. Patient using isoniazid not receiving vitamin B 6 D. Patient using levodopa and using vitamin B 6 supplements E. All of the above

33. Which of the following drug pharmacist should counsel to stop taking and contact doctor immediately if you feel suicidal ideas? A. Haloperidol B. Mefloquine C. Lisinopril D.Hydrochlorothiazide E. Isoniazid Ans. B 34. What agency in Canada issue warning related adverse reactions for drugs in post marketing surveillance? A. Health Canada's, Med Effect B. Institute of Safe Medication Practices Canada 65-5 Copyright © 2000 - 2016 TIPS Inc. Unauthorized reproduction of this manual is prohibited. This manual is being used during review sessions conducted by PharmacyPrep.

www.pharmacyprep.com

Adverse Drug Reactions

C. Canadian Institute of Health Information D. Canadian Pharmacist Association ans. A 35. Health professionals, Adverse reactions detected through cases should report to? A. Canada vigilance program B.ISMP C. CIHI D. C.PhA 36. Which of the following is NOT extra pyramidal symptoms? A. Tardive dyskinesia B. Rigidity C. Tremor

D. Dystonia

E. Vivid dreams

37. Which of the following is NOT require to monitor chest X-ray ?

A. Methotrexate Ans. E

b. Amiodarone

C. Bleomycin D. Bromocriptine E. Clozapine

38. What is used for treating constipation in children a. Mineral oil B. Sennkot C. Psyllium D. Polyethylene glycol E. sucralfate Ans.D

ADVERSE DRUG REACTIONS ANSWERS: 1. Ans: C 65-6 Copyright © 2000 - 2016 TIPS Inc. Unauthorized reproduction of this manual is prohibited. This manual is being used during review sessions conducted by PharmacyPrep.

www.pharmacyprep.com

Adverse Drug Reactions

2. Ans: A 3. Ans: B 4. Ans: C 5. Ans: E 6. Ans: B 7. Ans: E 8. Ans: E 9. Ans: B 10. Ans: D 11. Ans: A 12. Ans: C 13. Ans: B 14. Ans: B 15. Ans: C 16. Ans: A 17. Ans: C 18. Ans: D 19. Ans: A 20. Ans: D 21. Ans: A 22 Ans:. E 23. Ans: D 65-7 Copyright © 2000 - 2016 TIPS Inc. Unauthorized reproduction of this manual is prohibited. This manual is being used during review sessions conducted by PharmacyPrep.

www.pharmacyprep.com

Adverse Drug Reactions

24. Ans: C 25. Ans: E Tips: all anticholinergic drug gives constipation side effect. However donepezil is anticholinesterase (agonist) drug. 26. Ans: B 27. Ans: C Tips: low dose levothyroxine is used in seniors and cardiovascular disease patient. However no interactions with atenolol, ASA and clopidogrel 28. Ans: E Tips: do not use triptans if no relief with first dose. 29. Ans: B 30. Ans: C 31. Ans: C

65-8 Copyright © 2000 - 2016 TIPS Inc. Unauthorized reproduction of this manual is prohibited. This manual is being used during review sessions conducted by PharmacyPrep.

www.pharmacyprep.com

Adverse Drug Reactions

65-9 Copyright © 2000 - 2016 TIPS Inc. Unauthorized reproduction of this manual is prohibited. This manual is being used during review sessions conducted by PharmacyPrep.

PharmacyPrep.Com

Drug Interactions PHARMACY PREP DRUG & DRUG INTERACTIONS

1. A 55 year old male patient receiving warfarin for deep vein thrombosis, he get the prescription of carbamazepine; which of the following drug interaction possible: I-Carbamazepine decrease metabolism of warfarin II-Carbamazepine is has not interaction with warfarin III-Carbamazepine increase metabolism of warfarin A. I only B. III only C. I and II only D. II and III only E. All of the above 2. A 20 year old female patient currently on fluoxetine for depressive disorder, she get the prescription of Tylenol # 3 for sever headache. Which of the following drug interaction are possible: A. Fluoxetine metabolism increases B. Codeine metabolism increases C. Fluoxetine metabolism decrease D. Codeine metabolism decrease E. No change and there is no drug interactions 3. A 40-year old female is regular customer of your pharmacy, her current medical history include lovastatin. She brings prescription for her recent condition, Which of the following medication is safe for her medical profile: A. Erythromycin B. Ketoconazole C. Grapefruit juice D. Niacin E. Hydrochlorothiazide 4. Enzyme complex not working as a drug-metabolizing enzyme may include: A. Cytocrome P-450 B. Cytocrome P450 3A4 C. Alcohol Dehidrogenase D. UDP Glucoronyl Transferase E. Cytocrome oxidase. 5. What is incorrect about tetracycline antibiotics: I-Take with water and milk II-Take with food III- Must take empty stomach A. I only B. III only C. I and II only D. II and III only

E. All of the above

6. Tetracyclin binds with? Copyright © 2000-2016 TIPS Inc. Unauthorized reproduction of this manual is prohibited. This manual is being used during review sessions conducted by PharmacyPrep.

66-1

PharmacyPrep.Com

Drug Interactions

A. Mono valent ions B. Bi valent ions C. Tri valent ions D. B and C only E. All of the above 7. Ciprofloxacin should be taken with: A. Avoid taking with full glass of milk B. Take with or after food C. Take empty stomach D. Take with or without food E. A and D 8) A patient using calcium supplements 500 mg tid. Now gets the prescription of ciprofloxacin 500 mg bid for 3 days. What is appropriate method of administration? A) Take Ca supplements and ciprofloxacin together morning and evening. B) Take Ca supplements before 2 hour and after 4 hours of ciprofloxacin. C) Stop taking Ca supplements for 3 days and continues ciprofloxacin D) Stop taking ciprofloxacin and continue Ca supplement daily. E) Call the doctor, change to levofloxacin. 9. Which of the following statement is incorrect about alcohol and drug interactions? A. Alcohol and Metronidazole can give disulfiram reactions B. Alcohol and metformin can give lactic acidosis C. alcohol and chlorpropamide an antidiabetic drug can give lactic acidosis D. alcohol and benzodiazepine can give sedation 10. A patient using Sildenafil 50mg should not take nitrates, because? A. It can cause hypotension B. It can cause hypertension C. It can cause angina D. It can cause congestive heart failure E. None of the above 11. All of the following agent can cause urine discoloration, EXCEPT; A. Metronidazole B. Rifampin C. Sulfasalazine D. pyrivinium pamoate E. tetracycline 12. Iron supplement should be separated 2 to 4 hrs from all, except? A. Thyroxin B. Tetracycline C. Cholestyramine D. Atorvastatin E. None 13. What type of mechanism for drug interactions of warfarin and antibiotics? Copyright © 2000-2016 TIPS Inc. Unauthorized reproduction of this manual is prohibited. This manual is being used during review sessions conducted by PharmacyPrep.

66-2

PharmacyPrep.Com

Drug Interactions

A. Altering GI flora B. altering prothrombin time C. increasing bleeding time D. Increasing warfarin metabolism 14. A patient has migraine 3 to 4 times a months. Currently also diagnosed with benign prostatic hyperplasia. Doctor has prescribed prazosin. Medical profile also includes constipation. What drug is recommendation are suitable for migraine prophylaxis? A. Amitriptyline B.Verapamil C. Propranolol D. Sumatriptan E. Nortriptyline 15. What over the counter drugs should be avoided with MAOIs? A. Pseudoephedrine B. Xylometazoline C. Dextromethorphan D. Food containing Tyramine E. All of the above 16. Calcium supplement should separated 2 to 4 hrs from all, except? A. Ciprofloxacin B. Tetracycline C. Alendronate D. Thyroxin E. Acetyl salicylic acid 17. All of the following are pharmacodynamic reactions, except? A. Release of 5-ASA from sulfasalazine B. Diuretic and insulin actions C. Sedative with alcohol gives sedation D. None of the above 18. A 60-year-old women-taking calcium supplement 500 mg tid. Recently doctor prescribed ciprofloxacin 500 mg tid for 3 days. What is appropriate recommendation? A. Take calcium supplement with 2 hours apart from ciprofloxacin B. Skip calcium supplement during ciprofloxacin therapy C. Skip ciprofloxacin and just take calcium supplements D. Recommend other antibiotics such as ofloxacin E. Calcium and ciprofloxacin can be combined 19. Tetracycline binds with? Copyright © 2000-2016 TIPS Inc. Unauthorized reproduction of this manual is prohibited. This manual is being used during review sessions conducted by PharmacyPrep.

66-3

PharmacyPrep.Com I-antacids II-bi valent ions III-tri valent ions A. I only B. III only

Drug Interactions

C. I and II only D. II and III only

E. All of the above

20. A patient using atorvastatin 80 mg daily, also wants to use daily 1 glass of grapefruit juice, what is pharmacist concern? A. grapefruit juice induce CYP3A4 B. grapefruit juice inhibit CYP3A4 C. grapefruit juice is substrate for CYP3A4 D. grapefruit juice is essential for statin metabolism E. grapefruit juice may decrease cholesterol 21. A patient using sildenafil 50 mg, should be cautious taking high dose of take the following antihypertensive drugs A. Amlodipine B. Propranolol C. Hydrochlorothiazide D. Prazosin E. Ramipril 22. Which of the following is incorrect? A. Insulin plus physical exercise increase risk of hypoglycemia B. Insulin plus alcohol intake increase risk of hypoglycemia C. Insulin plus glyburide increase risk of hypoglycemia D. Insulin plus alcohol increase risk of lactic acidosis E. Insulin overdose can cause sweating, confusion and palpitation 23. A 60 year old women taking calcium supplement bid. Recently doctor prescribed alendronate 70 mg/wk. What is appropriate recommendation? A. Take together calcium supplements and alendronate 70 mg once weekly B. Do not take calcium supplements during the therapy of alendronate C. Separate calcium supplements 2 to 4 hours from the dose of alendronate D. Do not use alendronate while taking calcium supplement therapy 24. A 56 yo female using levothyroxine 25 mcg daily and metformin 500 bid. She comes to buy Calcium supplements to prevent osteoporosis. What is true statement? A. levothyroxine, calcium supplement should be separated for 4 hours so take levothyroxine empty stomach first thing in the morning. B. avoid taking calcium supplements because she is using levothyroxine C. Separate Ca supplement 4 hour from levothyroxine D. take Ca supplement in morning and take levothyroxine in evening E. Never take levothyroxine, metformin and calcium supplement together 25. A 56 yo female using metformin 500 bid, and ciprofloxacin 500 mg tid for 3days. She comes to buy calcium supplements 1500 mg tid to prevent osteoporosis. What is true statement? Copyright © 2000-2016 TIPS Inc. Unauthorized reproduction of this manual is prohibited. This manual is being used during review sessions conducted by PharmacyPrep.

66-4

PharmacyPrep.Com

Drug Interactions

A. recommend to separate Ca supplement 4 hour after ciprofloxacin B. recommend to take ciprofloxacin tid, metformin bid, and Ca supplement tid as doctor prescribe C. Recommend to start Ca supplement after she finish taking Ciprofloxacin D. Never combine ciprofloxacin and Ca supplements E. All of the above 26. A customer of your pharmacy enquiring that can she use herbal therapy like grapefruit juice to treat her borderline high cholesterol, She using atorvastatin for cholesterol therapy. what is appropriate to do? A. talk to your doctor B. definitely she can use grapefruit juice C. She should start taking statins and avoid grapefruit juice D. She should never take grapefruit juice for high cholesterol E. Give her some information to read about high cholesterol therapy 27. A customer of your pharmacy using allopurinol for the past 10 yrs for hyperurecimia and currently diagnosed for cancer. A doctor want to initiate azathioprine chemotherapy. What is appropriate to do? A. decrease dose of azathioprine B. decrease dose of allopurinol C. avoid allopurinol during chemotherapy D. avoid azathioprine chemotherapy E. do not change, any medications 28. A customer of your pharmacy profile shows nitroglycerin SL spray prn. He brings prescription of sildenafil 50 mg prn. What to do? A. Do not dispense sildenafil 50 mg because it can cause severe hypotension with nitroglycerin SL B. Dispense low dose sildenafil 25 mg C. Dispense sildenafil 50 mg and counsel customer not to combine with nitroglycerin D. dispense sildenafil 50 mg and counsel not to use nitroglycerin SL in case of chest pain E. Talk to doctor and inform doctor about patient profile nitroglycerin SL 29. A customer of your pharmacy wants to use St. Johns wart to improve mood. Currently he is on paroxetine for major depression. What is your opinion? A. ok to use B. do not use St. Johns wart during paroxetine therapy C. refer to doctor D. give him some literature of general health E. none of the above 30. Which of the following is the least drug interaction with levothyroxine? A. Iron supplements B. Calcium supplements C. Magnesium supplements D. Aluminum supplements Copyright © 2000-2016 TIPS Inc. Unauthorized reproduction of this manual is prohibited. This manual is being used during review sessions conducted by PharmacyPrep.

66-5

PharmacyPrep.Com

Drug Interactions

E. Vitamin D supplements 31. A regular customer, takes a glass of grapefruit juice daily. If medication profile have amiodarone, atorvastatin, hydrochlorothiazide, amlodipine and ASA 81 mg daily. Which of the following drug is the least concern? A. atorvastatin B. amlodipine C. hydrochlorothiazide D. amiodarone 32. Warfarin and carbamazepine drug interactions are categorized as? A. metabolism inhibitors B. Pharmacodynamic interaction C. enzyme inducer D. Pharmacokinetic interaction E. Receptor & receptor interactions 33. A regular customer of your pharmacy. Brings a new prescription of sildenafil 50 mg. Which of the following drugs in patient profile is the least concern? A. Nitroglycerin SL spray B. Nitroglycerin SL tablets C. Nitroglycerin transdermal patch D. Sodium nitroprusside E. Warfarin 5 mg tablets 34. If the above patient profile have nitrates, what is the pharmacist concern? A. priapism B. double vision C. headache D. hypotension E. mood changes 35. A patient medical profile include, metformin, gliclazide, rosuvastatin, and ASA 81 mg. Occasionally patient uses omega 3, 6,9 and multivitamin supplements. Patient is chronic alcoholic, renal disease, and liver diseases. Recently patient discharged from hospital after congestive heart failure. What is the least concern of pharmacist with respect to metformin? A. chronic alcoholism B. renal disease C. disulfuram like reactions D. congestive heart failure E. lactic acidosis

Copyright © 2000-2016 TIPS Inc. Unauthorized reproduction of this manual is prohibited. This manual is being used during review sessions conducted by PharmacyPrep.

66-6

PharmacyPrep.Com

Drug Interactions

36. A patient lifestyle include high proteins diet. Which of the following is the most concern for the pharmacist? A. theophylline B. levodopa C. renal disease D. orlistat E. liver diseases 37. All of the following drugs decrease efficacy of oral contraceptives, except? A. Rifampin B. Amoxicillin C. Phenytoin D. Gabapentin E. Carbamazepine Ans. d Tips: antibiotics and antiepileptic drugs likely decrease efficacy of oral contraceptive pills. 38.All of the following drug can be taken on an empty stomach, except a. levothyroxine B. Alendronate C. Omeprazole D. Metformin E. Zafirlukast Ans. D

Copyright © 2000-2016 TIPS Inc. Unauthorized reproduction of this manual is prohibited. This manual is being used during review sessions conducted by PharmacyPrep.

66-7

PharmacyPrep.Com

Drug Interactions

DRUG & DRUG INTERACTIONS ANSWERS: 1.Ans: B Tips: Carbamazepine is an inducer of CYP2C9, and the same enzyme is substrate of warfarin. Therefore Carbamazepine increase metabolism of warfarin 2. Ans: D Tips: Fluoxetine inhibits CYP 2D6 and this same enzyme is a substrate for codeine and therefore decrease codeine metabolism 3. Ans: E 4. Ans: C 5. Ans: C 6. Ans: D Tips: Tetracycline's bind with bi and tri valent ions such as calcium and aluminum ions. 7. Ans: E 8. Ans: C 9. Ans: C 10. Ans: A 11. Ans: E 12. Ans: D 13. Ans: A 14. Ans: C 15. Ans: E 16. Ans: E 17. Ans: A Copyright © 2000-2016 TIPS Inc. Unauthorized reproduction of this manual is prohibited. This manual is being used during review sessions conducted by PharmacyPrep.

66-8

PharmacyPrep.Com

Drug Interactions

18. Ans: B 19. Ans: E 20. Ans: B Tips: Grape fruit juice is potent inhibitor of the most common drug metabolizing enzyme CYP3A4 21. Ans: D 22. Ans: D 23. Ans: C 24. Ans: A 25. Ans: C 26. Ans: C 27. Ans: A Tips: azathioprine is metabolized by xanthine oxidase, thus reduce the dose of azathioprine to 2/3 or 3/4 28. Ans: E 29. Ans: B 30. Ans: E 31. Ans: C 32. Ans: C Tips: carbamazepine induce CYP 2C9 and 1A2, these enzymes are substrate of warfarin 33. Ans: E 34. Ans: D 35. Ans: C 36. Ans: C

Copyright © 2000-2016 TIPS Inc. Unauthorized reproduction of this manual is prohibited. This manual is being used during review sessions conducted by PharmacyPrep.

66-9

PharmacyPrep.Com

Drug Interactions

Copyright © 2000-2016 TIPS Inc. Unauthorized reproduction of this manual is prohibited. This manual is being used during review sessions conducted by PharmacyPrep.

66-10

www.Pharmacyprep.com

Clinical Biochemistry

PHARMACY PREP CLINICAL BIOCHEMISTRY 1. Thyroid Replacement therapy for hypothyroidism should be monitored by: A. Free T 4 B. Sensitive TSH assay C. Free thyroxine assay D. Total thyroxin E. Serum TSH assay 2. Serum TSH is elevated in? A. Hyperthyroidism B. Hypothyroidism C. Hyperparathyroidism D. Hypoparathyroidism E. Pheochromacytoma 3. A. B. C. D. E.

Which of the following plasma proteins has the GREATEST ability to bind acidic drugs Albumin Fibrinogen Hemoglobin Alpha glycoproteins Beta lipoprotein

4. Which of the following is not the correct match: A. Hypoparathyroidism hypocalcimia B. Hypoparathyroidism hyperphosphetemia C. Hyperparathyroidism hypercalcemia D. Hyperparathyroidism causes deficiency phosphates E. Hyperparathyroidism causes deficiency of calcium 5. Cockgraft-gault equation is used to estimate? A. Creatinine clearance. B. Blood Urea Nitrogen C. Transminases D. Urine analysis E. Blood coagulation 6. Which of the following is NOT decreased in elderly? A. Creatinine clearance B. Serum creatinine C. Blood urea nitrogen D. Albumin Copyright © 2000-2016 TIPS Inc. Unauthorized reproduction of this manual is prohibited. This manual is being used during review sessions conducted by PharmacyPrep 67-1

www.Pharmacyprep.com

Clinical Biochemistry

E. Adipose tissue 7. Which of the following is NOT decreased in elderly? A.fat content B. lean body mass C. kidney function D. Glomerular filtration E. Serum creatinine 8. What are the following lab test increase indicate myopathies? A. Creatinin kinase (CK-MM) B. Creatinin kinase (CK-BB) C. Creatinin kinase (CK-MB) D. ADH, AST, ALT & ALP E. ALP & ALT 9. A patient using warfarin should be monitored for? A. Creatinine kinase B. Prothrombin time C. Partial thromboplastin time (aPTT) D-Liver Function Test (LFT) E. Renal Function Test (RFT) 10) Which of the following laboratory investigations help to determine, myocardial infarction? A) Troponin-I and CK-MB B) CK-MB, Troponin C and ECG C) ECG, Troponin I, CK-MM D) Troponin I, CK-MB, ECG E) Troponin C, CK-MB, ECG 11. What are the following lab test increase indicate myocardial infarction? A. Creatinine kinase B. Troponin T C. Troponin I D. Troponin C E. ALP 12. A Patient with DVT was prescribed dalteparin. Which of the following lab test need to monitor? A. INR B. aPTT C. PT D. Vitamin K levels E. None of the above 13. Which of the following is used test calcium levels? A. Parathyroid hormone B. Calcitonin C. Thyroid hormone D. Sun exposure 14. A patient creatinine clearance is 300 ml/hr, it is a? A. chronic renal disease B. normal renal function Copyright © 2000-2016 TIPS Inc. Unauthorized reproduction of this manual is prohibited. This manual is being used during review sessions conducted by PharmacyPrep 67-2

www.Pharmacyprep.com

Clinical Biochemistry

C. excessive renal function D. severe renal disease E. Acute renal disease 15. What is the best urine sample for the urine analysis? A. end of the sample B. sample from midstream C. sample from end of stream D. Sample form beginning of stream E. any one of them is ok 16) A patient creatinine clearance 300 ml/hour. This patient is categorized as? A) normal CrCl B) Chronic renal disease C) Severe renal disease D) hepatic disease E. None of the above 17. Anticancer drugs in chemotherapy often cause the following side effects? A. Neutropenia B. Thrombocytopenia C. Leukopenia D. decrease in basophils E. Increased monocyte 18. Which of the following test is the most sensitive to liver cirrhosis? A. bilirubin B. AST C. ALT D. ALP E. CK 19. Which of the following is the most sensitive test to detect cholestasis jaundice? A. bilirubin B. AST C. ALT D. ALP E. CK 20. Which of the following is the most sensitive to detect acetaminophen hepatotoxicity? A. bilirubin B. AST C. ALT D. ALP E. CK 21. An osteoarthritis patient using NSAIDS to treat joint pain. Approaches to your pharmacy ask if any pain reliever change dark brown stool color or "coffee ground". What is appropriate? A) some NSAIDS can change stool color to dark brown B) No, NSAIDS change stool color however if you notice dark brown of coffee ground stool color you should contact doctor immediately C) all NSAIDS give dark stool color D) color change is quite normal and not a concern. E) Tell patient to avoid taking coffee Copyright © 2000-2016 TIPS Inc. Unauthorized reproduction of this manual is prohibited. This manual is being used during review sessions conducted by PharmacyPrep 67-3

www.Pharmacyprep.com

Clinical Biochemistry

22) Which of the following is the major ketone increase in blood diabetic ketoacidosis? A) Acetone B) Beta hydroxy butyric acid C) Ethyl and methyl ketone D) Long change ketones E) Acetyl acetic acid 23. All of the following drugs can increase CK-MM, except? A. Rosuvastatin B. Benzfibrate C. Atorvastatin plus niacin D. Simvastatin E) Ramipril 24. A laboratory test indicated increased number of neutrophil. This may be due to? A. Viral infections B. Inflammation C. Bacterial infection in sinusitis D) Seasonal flu E. Parasites infections 25. Decreased CD 4 count indicates? A. Viral infections B. Inflammation C. Bacterial infection D. COPD E. Parasites infections 25. A patient on vancomycin IV to treat MRSA. Which of the following lab test should be monitored? A. Liver function test B. Complete blood count C. Renal function test D. Creatinine kinase E. Chest X-ray 26) A patient is diagnosed with chronic bronchitis and pneumonia in COPD. Which of the following is increased? A. Basophills B. Neutrophil C. Lymphocytes D. WBCs E.RBCs 27. A patient laboratory shows increase in Eosinophills. This can be indicator of? A. COPD B. Infection C. Asthma D. pneumonia E. ulcer 28. A 55 yo patient using warfarin to treat deep vein thrombosis. Which of the following factors increase risk of bleeding? A. Using vitamin K supplements B. Using ASA 650 mg for pain C. Using dark green vegetables such as broccoli, avacado D. Using supplements containing vitamin K E. Using iron supplements 29. A patient serum thyroid function test (Serum TSH) shows, 10 mU/L. Doctor should initiate therapy of? Copyright © 2000-2016 TIPS Inc. Unauthorized reproduction of this manual is prohibited. This manual is being used during review sessions conducted by PharmacyPrep 67-4

www.Pharmacyprep.com

Clinical Biochemistry

A. methimazole B. Propylthiouracil C. Levothyroxine D. Radioactive iodide E. Lugol's solution 30) Which of the following test are used determine anemia? A) MCV B) Hematocrit C) Serum ferritin D) MCHC E) TIBC 31. What is lab test are related to diagnosis of lupus like syndrome? A. Increase ESR B. Increase C-reactive proteins C. Antinuclear antibody test D. Associated with myalgia, arthralgia and fever E. All of the above 32. Which of the following is NOT a lab investigation for rheumatoid arthritis? A. Erythrocyte sedimentation rate (ESR) B. C-reactive protein (cRP) C. Rheumatic factor D. Arthralgia E. Bone Mineral Density 33. KP is a 50 year old man using atorvastatin 40 mg daily. Get muscle pain. Which of the following lab test increase? A) ALT B) AST C) CK-MB D) CK-MM E) Troponin I 34. KG is a 50 yo patient on rosuvastatin 20 mg daily. Recently complain muscle pain. What is correct? A) Pharmacist refers patient to doctor to check CK-mm B) pharmacist refers patient to emergency C) pharmacist tell patient stop rosuvastatin and refer to dr D) pharmacist tell patient to reduce the dose of rosuvastatin E) Pharmacist should change rosuvstatin to simvastatin 35. MP is 49 year old male a type 2 diabetic patient. Currently patient has uncontrolled blood glucose. Which of the following are NOT related to diabetes or and glucometry tests? A. Glucose tolerance test B. Fasting blood glucose test Copyright © 2000-2016 TIPS Inc. Unauthorized reproduction of this manual is prohibited. This manual is being used during review sessions conducted by PharmacyPrep 67-5

www.Pharmacyprep.com

Clinical Biochemistry

C. HbA1c test D. Urine ketone test E. Serum Ferritin 36. A 70 year old patient is using heparin to prevent stroke. Doctor suspect patient has hemorrhage. Doctor should consider monitoring? A) INR B) PT C) aPTT D) heparin assay E) no lab test 37. Acetaminophen overdose associated with hepatotoxicity. This can increase the liver transaminases. Which of the following are liver transaminases? A. ALP, AST and ALT B. ALP and AST C. AST and ALT D. ALT and ALP E. Bilirubin 38. Which of the following liver enzyme are indicator or liver cirrhosis? A. AST/ALT B. Serum albumin C. Prothrombin time D. Bilirubin E. Alkaline. Phosphate 39. A patient presents with dark urine and muscle pain. Which of the following drugs may be patient is on? A) metronidazole B) Rifampicin C) bismuth D) lithium E) Atorvastatin 40.What lab test is used to determine liver cirrhosis? A. AST/ALT B. serum albumin C. prothrombin time D.Bilirubin E. Globulin Ans. D Tips. direct bilirubin, and urine bilirubin significant increased and AST/ALT minimally increased in hepatocelluar jaundice, obstructive jaundice and liver cirrhosis. 41. what lab test is for viral hepatitis patterns? A. AST/ALT B. ALP Ans. A

42. ESR is nonspecific test that is often used as screening tests for patients with unexplained fever, arthritis and muscle symptoms. The ESR elevation indicates, all of the following, except? A. RA Copyright © 2000-2016 TIPS Inc. Unauthorized reproduction of this manual is prohibited. This manual is being used during review sessions conducted by PharmacyPrep 67-6

www.Pharmacyprep.com

Clinical Biochemistry

B. Lupus like syndrome C. polymyalgia rheumatic D. giant cell arthritis E. osteoporosis Ans. 43. A Patient with DVT was prescribed dalteparin. Which of the following lab test need to monitor? A. INR B. aPTT C. PT D. Vitamin K levels E. None of the above

ans.e

Copyright © 2000-2016 TIPS Inc. Unauthorized reproduction of this manual is prohibited. This manual is being used during review sessions conducted by PharmacyPrep 67-7

www.Pharmacyprep.com

Clinical Biochemistry

CLINICAL BIOCHEMISTRY ANSWERS: 1. Ans: E 2. Ans: B 3.Ans: A 4. Ans: E 5. Ans: A Tips: Cockgraft-gault equation is used to estimate creatinine clearance. 6. Ans: E 7. Ans: A 8. Ans: A Tips: CK-MM indicator of skeletal muscle, CK-BB is for brain and CK-MB for cardio. 9.Ans: B 10) Ans. D 11. Ans: C

12. Ans: E 13. Ans: A 14. Ans: D 15.Ans: B 16) Ans. C Tips: CrCl 1.5 mEq/L B) < 1.5 mEq/L C) > 2.0 mEq/L D) > 3.0 mEq/L E) > 4.0 mEq/L 21) A 45 yo man recently diagnosed with serum TSH 10 mU/L. However normal range of serum TSH is 0.5 to 5 mU/L. This patient should be initiated? A) hyperthyroidism therapy B) hypothyroidisms therapy C) Methimazole D) Propylthiouracil E) Radioactive therapy 22) A patient is using furosemide to treat congestive heart failure. Which of the following electrolytes loss is NOT monitored? A) Potassium B) Calcium C) Magnesium D) Sodium E) Aluminum 23) Which of the following does NOT required monitoring, serum TSH? A) amiodarone B) Prednisone C) Levothyroxine D) methyl prednisone E) infliximab 24) A 70 yo patient is using hydrochlorothiazide 50 mg daily to treat hypertension. Which of the following is NOT monitored? A) uric acid B) blood glucose C) cholesterol (lipids) D) hypocalcemia E) hypokalemia 25)A 29 yo patient using lithium carbonate to treat manic depression. What is NOT monitored? A) serum TSH B) CBC C) blood glucose levels D) serum level of lithium E) eye exam

Copyright © 2000-2016 TIPS Inc. Unauthorized reproduction of this manual is prohibited. This manual is being used during review sessions conducted by PharmacyPrep.

68-4

PharmacyPrep.Com

Therapeutic Drug Monitoring

THERAPEUTIC DRUG MONITORING ANSWERS: 1) Ans: B Tips: Clozapine associated with side effect of agranulocytosis, thus it should be monitored for WBC or CBC. 2) Ans: A 3) Ans: A 4) Ans: A 5) Ans: A 6) Ans: A 7) Ans: D 8 Ans:) B 9) Ans: C 10) Ans: E Tips: Amiodarone, hydroxychloroquine and quinidien may conreal deposits. 11) Ans: D Tips: TSH are less than 0.4 mU/L is indicator of overdose of Synthroid, thus decrease dose. 12) Ans: C 13) Ans: E 14) Ans: B 15) Ans: B 16) Ans: C Tips: hydrochlorothiazide and furosemide can cause hypokalemia, this can lead to digitalis toxicity. 17) Ans: D Copyright © 2000-2016 TIPS Inc. Unauthorized reproduction of this manual is prohibited. This manual is being used during review sessions conducted by PharmacyPrep.

68-5

PharmacyPrep.Com

Therapeutic Drug Monitoring

18) Ans: C Tips: furosemide can cause hypocalcemia (loop loose calcium) 19) Ans: A 20) Ans: C 21) Ans: B 22) Ans: E 23) Ans: E 24) Ans: D Tips: thiazides cause hypercalcemia or hyperGLUC. Whereas loop diuretics cause hypocalcemia 25) Ans: E

Copyright © 2000-2016 TIPS Inc. Unauthorized reproduction of this manual is prohibited. This manual is being used during review sessions conducted by PharmacyPrep.

68-6

www.pharmacyprep.com

OTC medications dermatological conditions

PHARMACY PREP OTC DRUGS FOR DERMATOLOGICAL CONDITIONS 1-Incorrect statement about poison ivy treatment? A-Hydrocortisone 0.5% B-Calamine lotion C-Antihistamine D-Antibiotics E-Oatmeal cream 2-Incorrect pharmacotherapy for psoriasis treatment? A-Hydrocortisone 1% B-Coal tar C-Tazarotene D- Antibiotics E-UVA light + coal tar 3) What is incorrect about degree of burns classification? A) 1st degree is superficial burns, that may cause by sunburn B) 2nd degree is superficial and partial thickness burns that may cause by hot water, oil spills and splashes C) 3rd degree is the severe burns that can give lathery skin. D) If it burn put burn part in cold water E) If it burn put burn part in warm water 4-Incorrect statement about sunburn? A-peak time of sunburn is 10:00 am to 4:00pm B-High altitude has low sunburn C-cloud do not protect sunburn D-beach area may have more sunburn E-Apply sunscreen before get into sun 5-What is incorrect about sunburn treatment? A-Use cool baths or wet compresses with tap water or saline B-Take plenty of fluids C-Topical vitamin E applied two minutes after UV exposure may decrease erythema and edema D-Acetaminophen can relieve pain E-Avoid NSAIDs for sunburn pain 6) What is incorrect about head lice treatment? A-Permethrin 1% cream rinse apply to hair and scalp (1/2 to 1 bottle) and leave it on for 10 min then rinse. B-Permethrin may repeat after 7 days if live head lice is observed. Copyright © 2000-2016 TIPS Inc. Unauthorized reproduction of this manual is prohibited. This manual is being used during review sessions conducted by PharmacyPrep.

69-1

www.pharmacyprep.com

OTC medications dermatological conditions

C-May temporarily exacerbate itching and pruritus in head. D-May gives burning/stinging, tingling numbness in scalp E-After permethrin if it gives itching and pruritus refer to doctor 7) What is incorrect about head lice? A-All close contact need not to treated if one person has head lice B-Head lice can fly and transmit to other family member C-Long hair or short hair both can have head lice D-Hygiene is a not criteria for head lice E-Nits can hatch head lice in 7 to 10 days 8-Contact dermatitis and itching is managed by: I-oatmeal cream II-calamine lotion or zinc oxide III-local anesthetics (procaine) A-I only B-III only C-I and II only D-II and III only E-All of the above 9) What is the drug of choice to treat complicated diaper rash (diaper rash more than 3 days): A. barrier bases (zinc oxide, petrolatum) B. antifungal agents clotrimazole 1% cream, miconazole and Nystatin C. Anti-inflammatory agents (0.5% hydrocortisone) D. Alcohol wipes E. Changing diaper 10-What is incorrect in treatment of diaper rash? A-Keep diaper area dry B-clean diaper area with alcohol swabs C-Apply petrolatum on diaper area D-Apply 0.5% hydrocortisone 11-What does sun protection factor (SPF) 15 mean? A-if it takes 10 minutes to get sunburn, in SPF 15, it would take 15x10 min B-If it takes 10 minutes to get sunburn in SPF 15, it would take 15x100 min C- If it takes 10 minutes to get sunburn in SPF 15, it would take 150x100 min D- If it takes 10 minutes to get sunburn in SPF 15, it would take 150x1000 min E-If it takes 10 minutes to get sunburn in SPF 15, it would take 1500x100 min 12) All of the following are used for dandruff, except? A-Ketoconazole B-Coal tar Copyright © 2000-2016 TIPS Inc. Unauthorized reproduction of this manual is prohibited. This manual is being used during review sessions conducted by PharmacyPrep.

69-2

www.pharmacyprep.com

OTC medications dermatological conditions

C-Zinc pyrethione D-Hydrocortisone E-Selenium sulfide 13-Which of the following is NOT a pharmacotherapy for acne? A-Oral contraceptives B-Corticosteroids C-Tetracyclin D-Retinoids E-A and B 14-Which of the following is pharmacotherapy for acne in pregnancy? A-Oral contraceptives B-Erythromycin C-Tetracycline D-Retinoids E-Minocycline 15) A 20 year old women treated with topical treatment for acne, however, it not treated, which of the following is recommended? A-Benzoyl peroxide B-Tretinoin C-Isotretinoin D-Salicylic acid E-None 16) Which of the following skin condition have symptoms of red scaly patches? A) Psoriasis B) Dermatitis C) Acne D) Rosacea E) Urticaria 17) Characteristic of Rosacea symptoms is/are? A. Pustules B. Nodules C. Pimples D. Erythema E. Red scaly patches 18) Which of the following used to treat acne vulgaris? I) retinol II) cis/trans retinoic acid III) Vitamin E A) I only B) III only C) I and II only D) II and III E)all 19) What is NOT a symptoms of acne? A) White head (comedon) B) Black head (comedon) C) Pimple D) Postule E) Red scaly patches Copyright © 2000-2016 TIPS Inc. Unauthorized reproduction of this manual is prohibited. This manual is being used during review sessions conducted by PharmacyPrep.

69-3

www.pharmacyprep.com

OTC medications dermatological conditions

20. What is appropriate socks for athletes foot and diabetic foot problems? A. 100% cotton socks B. Nylon socks C. Synthetic fiber D. Synthetic socks E. Cotton (60% less) blend with synthetic fiber, nylon, acrylic or rayon

21. What is the drug of choice for head lice in child with seizures? A. permethrin 1% B. Lindane 1% C. Sulfur 5% D. Isopropyl mersteate E. permethrin 5% Ans. a 22. What is the drug of choice to treat mild acne with inflammation? A. benzoyl peroxide B. topical retinoids C. Tetracycline D. Clindamycin topical E. Erythromycin topical Ans. b 23.What is not related to acne? A. popping white heads with hands cause pimples, infection, scarring. B. small dark spots on skin are blackheads or open comedone C. Pink or silvery scaly patches D. acne forms popules, pustules E. acne forms cysts and nodules Ans. c Tips. white head, pimples, popules, pustules, cysts and nodules are forms of acne. Pink and silvery patches are psoriasis. 24. What is incorrect related to rosacea acne? A. Erythema is present rosacea acne B. Papulopustular lesions and eye involvement with or without C. Drug of choice is metronidazole cream D. Telangiectasia or facial edema or rhinophyma E. Caused by Corynebacterium parvum Ans. e

Copyright © 2000-2016 TIPS Inc. Unauthorized reproduction of this manual is prohibited. This manual is being used during review sessions conducted by PharmacyPrep.

69-4

www.pharmacyprep.com

OTC medications dermatological conditions

25. What is appropriate socks for athletes foot and diabetic foot are recommended? A. 100% cotton socks B. Nylon socks C. Synthetic fiber D. Synthetic socks E. Cotton (60% less) blend with synthetic fiber, nylon, acrylic or rayon Ans. e

Copyright © 2000-2016 TIPS Inc. Unauthorized reproduction of this manual is prohibited. This manual is being used during review sessions conducted by PharmacyPrep.

69-5

www.pharmacyprep.com

OTC medications dermatological conditions

OTC DRUGS FOR DERMATOLOGICAL CONDITIONS ANSWERS: 1- Ans: D 2- Ans: D

3) Ans: E st

1 degree superficial burns, that may cause by sunburn

BURNS CLASSIFICATION 2 degree superficial and partial thickness burns that may cause by hot water, oil spills and splashes nd

3rd degree severe burns that can give lathery skin.

4- Ans: B 5- Ans: E Head lice 6- Ans: E 7) Ans: B 8- Ans: C Tips: Astringents such as calamine lotion, aluminum acetate, zinc oxide 9. Ans: C Tips: The drug of choice to treat complicated diaper rash is clotrimazole 1% cream. Uncomplicated diaper rash Complicated diaper rash Limited rash Confluent tomato red plaques, white scaly border. ZnO, petrolatum, hydrocortisone Clotrimazole, miconazole, nystatin and ZnO, hydrocortisone 10- Ans: B 11- Ans: A Tips: snow, high altitude, and beach, mountains can give more sun burn. 12) Ans: D 13) Ans: B

Copyright © 2000-2016 TIPS Inc. Unauthorized reproduction of this manual is prohibited. This manual is being used during review sessions conducted by PharmacyPrep.

69-6

www.pharmacyprep.com

OTC medications dermatological conditions

14- Ans: B 15) Ans: C 16) Ans: A 17) Ans: D 18) Ans: C 19) Ans: E Tips: Red scaly patches are symptoms of psoriasis. 20. Ans. E

Copyright © 2000-2016 TIPS Inc. Unauthorized reproduction of this manual is prohibited. This manual is being used during review sessions conducted by PharmacyPrep.

69-7

PHARMACY PREP OTC AND PRESCRIPTION DRUGS FOR OPHTHALMIC, EAR AND MOUTH DISORDERS 1. What is an optimal range of pH for ophthalmic products A. 2-3 B. 4-5 C. 5-6 D. 6-8 E. 8-10 2. Opthalmic agents contraindicated in glaucoma patients include which of the following substances: A. antioxidants B. decongestants C. emollients D. antipruritics E. All of the above 3. Which of the following lenses can be worn continuously for 6 days: A. Soft lenses B. Extended wear soft lenses C. Frequent replacement soft lenses D. Hard lenses E. Soft and hard lenses 4. Image formed by lens received by: A. iris B. retina C. sclera

D. conjunctiva

E. pupil

5. What is correct statement about treatment in eruption cysts in infants A. In general cysts rupture spontaneously B. Use teeters to rupture cysts C. Manually cysts can be removed D. None of the above 6. Which of the following treatment is used to treat cold sores? I-Antihistamine II-Zinc oxide (ZnO) III-Benzocaine 7.5% gel A. I only B. III only C. I and II only D. II and III only E. All of the above 7. What is treatment of cold sores in chicken pox? I-Calamine lotion II-Oatmeal bath (micellar colloidal bath) III-antibiotics A. I only B. III only C. I and II only D. II and III only

E. All of the above

8. Which of the following is the incorrect action of sympathomimetics? I- Increase of IOP II- Increase the drainage of H 2 O humor from the eye III- Decrease the H 2 O humor in the eye A. I only B. III only C. I and II only D. II and III only E. All of the above Copyright © 2000-2016 TIPS Inc. Unauthorized reproduction of this manual is prohibited. This manual is being used during review sessions conducted by PharmacyPrep.

70-1

9. Which of the following ophthalmic conditions required referral to doctor? A. Red eye B. Pain in eye C. Blurred vision D. Dry eye E. B and C 10) Which of the following ophthalmic conditions require referral to doctor? except A) Red eye or pink eye B) Pain in eyes C) Blurred vision D) Dry eye in diabetic patient E) Blepharitis Ans. A 11) A customer of your pharmacy using metformin 500 mg bid and glyburide qd. Complains to have gritty and sandy feeling in eye. What is appropriate to do? A) Refer to doctor because of diabetic condition B) Refer to doctor because sandy and gritty feeling in eye is conjunctivitis C) Refer to doctor because glyburide side effect is dry eye D) Refer to doctor because metformin side effect is dry eye E) Refer to doctor because these are side effects of anticholinergic drugs 12. Which of the following can be used for earwax removal? A) carbamide peroxide B) soft mineral oil C) Burrow's solution (aluminum acetate) D) All of the above Ans. D 13. Otitis externa (swimmers ear), is treated by? A. amoxicillin high dose B. amoxicillin + clavulanate C. Aluminum acetate (Burrow's solution) D. Cephalosporin's E. All of the above 14. A 60 yo customer of your pharmacy medical profile include; metformin, glyburide, amiodarone, rosuvastatin, ASA 81 mg. Recently doctor prescribed methrotrexate to treat psoriasis. Patient reports sandy and gritty eyes. Pharmacist direct patient to see doctor. Which of the following drug concern prompted pharmacist refer to doctor? A. methotrexate B. metformin C. Glyburide D. amiodarone E. rosuvastatin

Copyright © 2000-2016 TIPS Inc. Unauthorized reproduction of this manual is prohibited. This manual is being used during review sessions conducted by PharmacyPrep.

70-2

15. Which of the following eye condition can be treated by non prescription therapy? A. Eye pain B. Vision changes C. Blepharitis D. Red eye E. Foreign object in eye Ans. D 16. What is incorrect about ophthalmic lens? A. Soft lens are made of Hydroxyethyl methacrylate (HEMA) B. Surfactant in lens solution prevent infection C. Protein cleaners in lens solution remove protein deposits D. Wetting agents in lens solution produce lubricant effect E. Hard lens can wear for more than 24 hours Ans. E

17. Which of the following condition require referral to doctor? A. Ear pain especially on movement B. Too much ear wax C. Fullness in ear D. Significant debris and fluid discharge from ear canal E. All of the above Ans. e OTC AND PRESCRIPTION DRUGS FOR OPHTHALMIC, EAR AND MOUTH DISORDERS ANSWERS:

1.Ans: D 2. Ans: B 3. Ans: B 4. Ans: B 5. Ans: A 6. Ans: D 7. Ans: C Tips: Calamine lotion (ZnO+FeO) and oatmeal bath and oral antihistamine can be helpful in treatment of cold sores. Copyright © 2000-2016 TIPS Inc. Unauthorized reproduction of this manual is prohibited. This manual is being used during review sessions conducted by PharmacyPrep.

70-3

8. Ans: D Tips: Sympathetic agonist (sympathomimetics) increases IOP. Open angle glaucoma (OAG) is due to increase in IOP and increase in H 2 O humor of eye or could be due decrease in drainage of H 2 O humor in the eye. For the treatment of OAG sympathetic blockers such as Timolol are effectively used to decrease IOP and H 2 O humor. 9. Ans: E 10. Ans: D 11. Ans: A Tips: Diabetic retinopathy symptoms require referral to doctor 12. Ans: C 13. Ans: C 14. Ans: D Tips: amiodarone gives ocular deposits SEs.

Copyright © 2000-2016 TIPS Inc. Unauthorized reproduction of this manual is prohibited. This manual is being used during review sessions conducted by PharmacyPrep.

70-4

Copyright © 2000-2016 TIPS Inc. Unauthorized reproduction of this manual is prohibited. This manual is being used during review sessions conducted by PharmacyPrep.

70-5

PharmacyPrep.Com

OTC Antihistamine's, Decongestants and Antitussives

PHARMACY PREP OTC ANTIHISTAMINE, DECONGESTANTS AND ANTITUSSIVES 1) Which of the following is NOT an environmental measure to prevent allergic rhinitis: A-Avoidance of window or attic fans B-Avoid outdoor activities during pollen season C-Shower or bath following outdoor activity to avoid contamination of bedding D-Use air conditioning E-Open windows over night to get fresh air. 2-Sympathomimetics decongestant acts as? A-Vasoconstrictor B-Vasodilator C-Congestion D-Anti-inflammatory actions E-None of the above 3-Sympathomimetics decongestant can cause, all of the following complications, except? A- Open angle glaucoma B- hypertension C- diabetics D-Can cause rebound congestion in prolong use E-Can cause vasodilatation 4) All of the following drugs gives bronchodilators, except A-short acting beta2 agonist B-long acting beta2 agonist C-Propranolol D-Salbutamol E-Salmeterol 5-Opioids may be used as all the following agents, EXCEPT: a) Antitussive b) Analgesic c) Antidiarrheal d) Anti-inflammatory e) Pre-anesthetic 6-Alpha 1 agonist, phenylephrine has all of the following side effect EXCEPT: A-vasoconstriction B-Headache C) orthostatic hypotension D-Arrhythmias E-Mydriasis 7- Fexofenadine HCI (Allegra 12 Hour and Allegra 24 Hour) is classified as: A-Histamine H 2 -Receptor Antagonist B- Histamine H 1 -Receptor Antagonist C-Proton Pump inhibitor (PPI) D-Serotonin reuptake inhibitor (SSRI) Copyright © 2000-2016 TIPS Inc. Unauthorized reproduction of this manual is prohibited. This manual is being used during review sessions conducted by PharmacyPrep.

71-1

PharmacyPrep.Com

OTC Antihistamine's, Decongestants and Antitussives

E-5-HT 1 Receptor Agonist 8-A mother comes to your pharmacy with concern of her child 2 yr old having fever and pharyngitis, currently using amoxicillin. What is appropriate recommendation? A-recommend ASA B-recommend oral ibuprofen C-Recommend acetaminophen suppositories D-Recommend cold sponges E-do not recommend any medications 9) A customer of your pharmacy travelling to London, Ontario, wants to buy some medications for nausea and vomiting in motion sickness. What is appropriate recommendation? A) Dimenhydrinate B)Dextromethorphan C) Scopolamine D)Ginger root E) Meclazine 10) A customer of your pharmacy using MAO Inhibitors for major depression. Now looking for nasal congestion medication. What is recommended? A) pseudoephedrine B) ephedrine C) codeine D)d extromethorphan E) saline nasal drops 11. A regular customer, purchased dextromethorphan 2 times in last 20 days. Pharmacist talk to patient and checked patient profile and referred to doctor. Which of the following medication in patient profile prompted pharmacist? A) Glyburide B) Metformin C) Rosuvastatin D) Ramipril E) All of the above 12. A 50 year old school bus driver brings prescription of alprazolam and clonazepam to treat anxiety. He is frequent customer of pharmacy to purchase diphenhydramine and dimenhydrinate. Pharmacist informed to doctor patient use of benzodiazepines and antihistamine. Dr reported to employer of patient. What ethical principle were applied? A) Nonmaleficence B) Beneficence C) Autonomy D) Veracity E) Justice 13. Which of the following is the most commonly abused over the counter medication? A. Dextromethorphan B. Pseudoephedrine C. Tylenol #1 D. Diphenhydramine E. Marijuana Ans. a

Copyright © 2000-2016 TIPS Inc. Unauthorized reproduction of this manual is prohibited. This manual is being used during review sessions conducted by PharmacyPrep.

71-2

PharmacyPrep.Com

OTC Antihistamine's, Decongestants and Antitussives

14. A doctor prescribed a sedative antihistamine to a taxi driver. What is appropriate pharmacist action? A. Dispense drug and counsel patient not to drive after taking this medication. B. Call doctor to change to non sedative antihistamine C. Change to non sedative antihistamine and dispense to patient. D. counsel patient to not to take alcohol with sedatives E. Give option to patient to decide to take and not to drive

Copyright © 2000-2016 TIPS Inc. Unauthorized reproduction of this manual is prohibited. This manual is being used during review sessions conducted by PharmacyPrep.

71-3

PharmacyPrep.Com

OTC Antihistamine's, Decongestants and Antitussives

OTC ANTIHISTAMINE, DECONGESTANTS AND ANTITUSSIVES ANSWERS 1) Ans: E 2- Ans: A 3- Ans: E 4) Ans: C 5- Ans: D 6- Ans: C Tips: Hypertension is common side effect phenylephrine. 7- Ans: B 8- Ans: B 9) Ans: A 10) Ans: E Tips: with MAO I for nasal congestion avoid using pseudoephedrine, ephedrine can cause hypertension crisis For cough avoid dextromethorphan cause serotonin syndrome 11. Ans: D Tips: ACE Inhibitors may induce cough. This can be managed by changing therapy to ARBs. 12. Ans: A

Copyright © 2000-2016 TIPS Inc. Unauthorized reproduction of this manual is prohibited. This manual is being used during review sessions conducted by PharmacyPrep.

71-4

PharmacyPrep.Com

OTC Antihistamine's, Decongestants and Antitussives

Copyright © 2000-2016 TIPS Inc. Unauthorized reproduction of this manual is prohibited. This manual is being used during review sessions conducted by PharmacyPrep.

71-5

PharmacyPrep.Com

OTC drugs for Nausea, Vomiting, Constipation..

PHARMACY PREP OTC DRUGS FOR NAUSEA, VOMITING, CONSTIPATION, DIARRHEA, HEMORRHOIDS AND HELMENTHS 1) A 60 yo customer of your pharmacy using oxycontin daily for severe pain after fracture. What is the treatment best option for constipation? I) Sennakot

II) Docusate sodium

a) I only

b) III only

c) I and II only

III) high fiber diet d) II and III only

e) I, II, III

2- Psyllium (Metamucil) is a bulk-forming laxative agent. Patients using should get the following instructions: I- Avoid psyllium (Metamucil) in pregnancy III- It should mix the dose with a glass of cool water or other fluid and drink it down quickly. II-This should be followed with more fluids. a) I only b) III only c) I and II only d) II and III only e) I, II, III 3)Antacids are normally used to equilibrate the stomach pH and protect the stomach from the effect of acids. Which of the following antacids has a cathartic side effect? a) Calcium carbonate b) Dihydroxyaluminum sodium carbonate c) Magnesium hydroxide d) Calcium sulfate e) Aluminum silicate 4- Which antihelmenthic drug is used to treat pin worm? A-Pyremethamine B-Pyridoxin C-Streptomycin D-Pyrivenium pamoate E-Metronidazole 5- Lactulose is an osmotic laxative agent that may be used to treat: I- Constipation II- Portal-systemic encephalopathy III- Renal tubular necrosis a) I only b) III only c) I and II only d) II and III only

e) All are correct

6) A mother of 6 month old child approaches to your pharmacy. Her child has diarrhea. She is also breast feeding her child, what is appropriate recommendation? A) recommend ORS and ask to continue breast feeding B) Recommend ORS and ask to stop breast feeding C) Recommend ORS and refer to doctor Copyright © 2000-2016 TIPS Inc. Unauthorized reproduction of this manual is prohibited. This manual is being used during review sessions conducted by PharmacyPrep.

72-1

PharmacyPrep.Com

OTC drugs for Nausea, Vomiting, Constipation..

D) Refer to emergency E) Do nothing, it will be alright 7) Drug of choice for constipation induced by codeine? A-Psyllium B-Lactulose C-Sodium docusate D-Senna

E-Mineral oil

8) All of the following are self care measure should be recommended in hemorrhoids, except? A. Increase high fibre diet B) Increase fluid intake C) Avoid food on the run D) Regularize toilet habits e)Prolong stay on toilet seat 9) Drug of choice for constipation in pregnancy? A-Psyllium B-Lactulose C-Sodium docusate

D-Senna E-Mineral oil

10) What is incorrect recommendation to prevent traveler’s diarrhea? A-Avoid ice cubes B-Take fresh salad C-Take cooked food D-Used boiled or bottled water E-Avoid seafood 11)A mother comes to pharmacy and has concern about a 6 months baby have diarrhea. She is also breast-feeding child, what is appropriate recommendation? A-recommend ORS and ask to continue breast feeding B-Recommend ORS and ask to stop breast feeding C-Recommend ORS and refer to doctor D-Refer to emergency E-Do nothing, it will be alright 12) A cancer patient experiencing severe nausea and vomiting, should be recommended, all except? A-Avoid large meals B-Avoid fat-oily meals C-Avoid taking small and frequent meals D-Take divided meals E-Avoid heavy meals 13) Drug of choice for constipation induced by codeine? A-Psyllium B-Lactulose C-Sodium docusate

D-Senna

E-Mineral oil

14) All of the following are self care measure should be recommended in hemorrhoids, except? a)Increase high fiber diet b)Increase fluid intake C)Avoid food on the run D)Regularize toilet habits e) Prolong stay on toilet seat 15) Drug of choice for constipation in pregnancy? A-Psyllium B-Lactulose C-Sodium docusate D-Senna E-Mineral oil Copyright © 2000-2016 TIPS Inc. Unauthorized reproduction of this manual is prohibited. This manual is being used during review sessions conducted by PharmacyPrep.

72-2

PharmacyPrep.Com

OTC drugs for Nausea, Vomiting, Constipation..

16) What is incorrect recommendation to prevent travelers diarrhea? A-Avoid ice cubes B-Take fresh salad C-Take cooked food D-Used boiled or bottled water E-Avoid sea food 17) Which of the following is the correct statement about nausea and vomiting in pregnancy A-initial therapy is Diclectin B-initial therapy is dimenhydrinate and self care for nausea and vomiting recommended C)Refer to emergency D)None of the above 18) A customer of your pharmacy, want to buy something for hard stool. What is good recommendation A) Senna B) Bran C) Psyllium D) Docusate E) Enema 19) A pregnant women comes to your pharmacy, and have nausea and vomiting. What is appropriate to do initial therapeutic approach in first 3 days of nausea and vomiting? A-Recommend dimenhydrinate to treat mild N&V and self care B-Refer to doctor for Diclectin C-Recommend self care measure to minimize nausea vomiting D-Refer to emergency E-Recommend ginger root tablets 20) Which of the following drug gives diarrhea? A-Atropine B-Ipratropium C-Scopolamine

D-Pilocarpine E-Trihexphenidyl

21. Bisacodyl mechanism of action (Dulcolax) a. Stimulant laxative b. Lubricant c. Bulk forming d. Stool softener e. Stimulant laxative and stool softener 22) Which of the following microorganism have can cause traveler diarrhea in customer travelling to south east Asia? I) E. coli II) Shigella sp III) Compylobacter jejuni a) I only

b) III only

c) I and II only d) II and III only

e) I, II, III

23) A patient is buying Tylenol # 1, 200 tablets. Which of the following is pharmacist concern? I) Constipation a) I only

II) Liver toxicity b) III only

III) Addiction

c) I and II only

d) II and III only

e) I, II, III

Copyright © 2000-2016 TIPS Inc. Unauthorized reproduction of this manual is prohibited. This manual is being used during review sessions conducted by PharmacyPrep.

72-3

PharmacyPrep.Com

OTC drugs for Nausea, Vomiting, Constipation..

24) A patient with terminal metastatic breast cancer gets the prescription of oxycontin and Psyllium. Later after few days her daughter comes to pick a prescription of hydromorphone and lactulose. What is pharmacist concern? A) addiction potential B) False prescription C) Contraindication D) constipation E) Overdose 25) 1. What suggestions and advice should be given to the patient with pinworm infection? I. Treatment of pin worms should be taken for all members of the family II. Must not share under garments III. Must not share dishes a) I only b) III only c) I and II only d) II and III only e) All of the above 26. What is the drug of choice to treat chemotherapy induced nausea and vomiting? A. Ondansetron B. Dexamethasone C. Lorazepam D. Metoclopramide E. Dimenhydrinate Ans. b 27. What is the drug of choice to treat constipation in children? A. Castol oil B. Senna C. Biscodyl D.Glycerine suppositories Ans. d

Copyright © 2000-2016 TIPS Inc. Unauthorized reproduction of this manual is prohibited. This manual is being used during review sessions conducted by PharmacyPrep.

72-4

PharmacyPrep.Com

OTC drugs for Nausea, Vomiting, Constipation..

OTC DRUGS FOR NAUSEA, VOMITING, CONSTIPATION, DIARRHEA, HEMORRHOIDS AND HELMENTHS ANSWERS 1. Ans: E 2- Ans: D Tips: Psyllium is safe to use in pregnancy 3- Ans: C 4- Ans: D 5- Ans: C Tips: Portal – systemic encephalopathy. Osmotic laxative like lactulose, decrease blood ammonia levels and as result this effect on mental status in encephalopathy patients. 6) Ans: A ( it was D before) 7) Ans: D 8) Ans: E 9) Ans: A 10) Ans: B 11) Ans: A 12) Ans: C 13) Ans: D 14) Ans: E 15) Ans: A

Copyright © 2000-2016 TIPS Inc. Unauthorized reproduction of this manual is prohibited. This manual is being used during review sessions conducted by PharmacyPrep.

72-5

PharmacyPrep.Com

OTC drugs for Nausea, Vomiting, Constipation..

16) Ans: B 17) Ans: B 18) Ans: D 19) Ans: A 20) Ans: D 21. Ans: A 22) Ans: E 23) Ans: C

24) Ans: B 25) Ans: E Tips: spreads by oral-fecal infestation.

Copyright © 2000-2016 TIPS Inc. Unauthorized reproduction of this manual is prohibited. This manual is being used during review sessions conducted by PharmacyPrep.

72-6

PharmacyPrep.Com OTC drugs for headache, sports injuries, pressure ulcers, and back pain PHARMACY PREP OTC DRUGS FOR HEADACHE, SPORTS INJURIES, PRESSURE ULCERS, AND LOW BACK PAIN 1. Correct statement/s about ASA may include I. Low dose are used in heart problem II. Less potent than naproxen as an analgesic III. More potent than acetaminophen as analgesic a) I only b) III only c) I and II only d) II and III only 2-What is antidote of acetyl salicylic acid (ASA)? A-N-acetylcysteine B-Flumazenil C-Naloxone

e) All of the above

D-Naltrexone E-None of the above

3-Anti-inflammatory action of ASA is due to? A-Irreversible inhibition of platelet aggregation B-Prevention of prostaglandin synthesis C-Inhibition of COX-I receptors D-Inhibition of Cox-1 and Cox-II receptors E-Inhibition of Cox-II receptors 4. Dyspnea means a) Painful muscle spasms b) Pain in the heart e) Painful menstruation

c)Pain in extremities d)Painful breathing

5. Over dosage of acetaminophen in patients would affect I) Liver functions ii) Prothrombin time iii) Renal functions a) I only b) III only c) I and II only d) II and III only

e) All of the above

6-Which of the following is not a side effect of an opioids analgesic for example morphine: a) C.V depression b) Constipation c) Sedation d) CNS depression e) Dilatation of eye pupil 7-A sports person has ankle sprain while playing ice hockey, what is inappropriate recommendation? A-Rest to injured part B-Apply ice C-Warm compress D-Elevate injured area E-Recommend OTC NSAIDs

Copyright © 2000-2016 TIPS Inc. Unauthorized reproduction of this manual is prohibited. This manual is being used during review sessions conducted by PharmacyPrep

73-1

PharmacyPrep.Com OTC drugs for headache, sports injuries, pressure ulcers, and back pain 8) Mechanism of action of sumatriptan is? A) 5HT 1b/d agonist B) 5HT 1b/d antagonist C) 5HT 3 antagonist D) 5HT 3 agonist E) 5HT 2 agonist 9) What is treatment for tension headache? A) Triptans B-Ergot alkaloids C-Propranolol E) Acetaminophen

D-Acetyl salicylic acid

10) All of the following can trigger migraine, except? A) Smell B) Food like chocolates C) Light D) Emotional stress E) Physical activity 11) A 45 yo man diagnosed with benign prostatic hyperplasia, and migraine prophylaxis. Which of the following is the appropriate therapy for migraine prophylaxis? A) Propranolol B) Nortriptyline C) Amitriptyline D) Sumatriptan E) Zolmitriptan 12) A customer of your pharmacy using sumatriptan for migraine attacks. What is correct statement? A)after taking sumatriptan migraine headache does not relieve than double the dose of sumatriptan B)after taking sumatriptan migraine headache does not relieve than decrease the dose of sumatriptan C)after taking sumatriptan migraine headache does not relieve than do not use sumatriptan D. If sumatriptan is not effective in relieving headache then try other triptan E. None of the above 13) A 12 month old child swallowed acetaminophen 10 ml syrup. A panic mother contact pharmacy. What is the initial symptoms of acetaminophen overdose? A) Nausea B) Vomiting C) Stomach upset D) Diarrhea E) Hepatotoxicity

14.Nociceptive pain are caused by? A. Psychological factors B. Injury to body tissue Copyright © 2000-2016 TIPS Inc. Unauthorized reproduction of this manual is prohibited. This manual is being used during review sessions conducted by PharmacyPrep

73-2

PharmacyPrep.Com OTC drugs for headache, sports injuries, pressure ulcers, and back pain C. Damage to nerve D. Diabetes E. Post herpes infection Ans. B 15. A patient with renal disease and has low back pain. What is the treatment option? A. Diclofenac topical B. Capsaicin C. Oral NSAIDs D. Methadone E. Codeine Ans. A 16. All of the following can be used to manage low back pain, except? A. Tramadol B. Acetaminophen C. Diclofenac D. Colchicine E. Tramacet Ans. D 17. A 12 yo boy got severe injury in his ankle during a soccer game. All of the following are appropriate, except? A. use acetaminophen for pain B. use ibuprofen pain C. Apply cold compress D. Apply warm compress E. Rest injured area Ans. D

18. Fingolimod is? A. spingosine-1-phosphate receptor agonist B. Adhesion molecule C. TNF alpha inhibitor D. 5-alpha reductase inhibitor E. Anti-CD52 monoclonal antibody ans. A

Copyright © 2000-2016 TIPS Inc. Unauthorized reproduction of this manual is prohibited. This manual is being used during review sessions conducted by PharmacyPrep

73-3

PharmacyPrep.Com OTC drugs for headache, sports injuries, pressure ulcers, and back pain OTC DRUGS FOR HEADACHE, SPORTS INJURIES, PRESSURE ULCERS, AND LOW BACK PAIN ANSWERS: 1. Ans: C 2- Ans: E 3- Ans: B 4. Ans: D 5. Ans: C 6- Ans: E 7- Ans: C Tips: R-I-C-E is recommended approach for sports injuries. R= rest to injured part, I = apply ice, C = cold compress, E = elevated injured part. 8) Ans: A 9) Ans: E 10) Ans: D 11) Ans: A 12) Ans: C 13) Ans: B

Copyright © 2000-2016 TIPS Inc. Unauthorized reproduction of this manual is prohibited. This manual is being used during review sessions conducted by PharmacyPrep

73-4

PharmacyPrep.Com OTC drugs for headache, sports injuries, pressure ulcers, and back pain

Copyright © 2000-2016 TIPS Inc. Unauthorized reproduction of this manual is prohibited. This manual is being used during review sessions conducted by PharmacyPrep

73-5

www.PharmacyPrep.Com

Asthma and COPD

PHARMACY PREP ASTHMA AND COPD 1) Theophylline serum levels are monitored in neonates and infants because A) They are poorly absorbed in systemic circulations B) Drug has narrow therapeutic index C) Slowest rate of clearance D) Fastest rate of clearance E) It is not used in neonates and infants 2) A patient is suffering from COPD and asthma. He is on theophylline, ranitidine and now doctor prescribed ciprofloxacin which of the following is correct? A) ranitidine have drug interaction with theophylline B)Ciprofloxacin have drug interaction with theophylline C)Ranitidine and ciprofloxacin both have drug interactions with theophylline D) None of the above 3) Which of the following is NOT a side effect of theophylline? A-Insomnia B-Sedation C-Appetite suppressant D-None of the above 4) A 3 yr child was newly diagnosed with asthma. All of the following questions are important to ask his mother EXCEPT: A) Any cardiovascular problems? B) Does other family member have asthma? C) How often symptoms come? D) Old asthma medication he used before? E) How many times wake up in night due to symptoms 5) A 45 yo patient with emphysema and pain, and is not controlled, the doctor switched him to morphine and his daughter came to pharmacy, you will call the doctor because: a) You will ask to change the medication b) To tell the dose of medication because its too much c) To ask for written prescription of morphine d) Her concern about constipation 6. What is true about Ipratropium? I. Reversible competitive of anticholinergic II. Synergistic with Theophylline in asthma III. Quaternary ammonium compound a. I only b. III only Copyright © 2000-2016 TIPS Inc. Unauthorized reproduction of this manual is strictly prohibited and 74-1 it is illegal to reproduce without permission. This manual is being used during review sessions conducted by PharmacyPrep.

www.PharmacyPrep.Com

Asthma and COPD

c. I and III only d. II and III only e. I, II, and III only 7) Which of the following is incorrect statement of leukotriene receptors inhibitors? A) used to control the dose of steroids B) Indicated to treat congestion in children over 2 yr C) Treatment of choice for beta blockers and Aspirin induced asthma D) Patient used leukotriene receptors inhibitors require increase requirement of steroids. E) Montelukast is used in children over 2 yr age 8) A 40 yr old asthma patient brings a new prescription of montelukast bid. Currently medication profile include salbutamol prn, Symbicort (budesonide and formoterol) 1 puff bid. What is the action to do? A) ask patient to stop using Symbicort B) ask patient to stop using salbutamol prn C) dispense montelukast and counsel how to use it D) Contact doctor to change Symbicort to Advair E) Contact doctor to change to Zafirlukast 9) A COPD patient brings a prescription of doxycycline daily to treat pneumonia. Which of the following is the correct statement? A) ask patient if he is taken flu vaccine B) ask patient if his close contacts have taken flu vaccine C) ask patient if his close contacts have taken flu and pneumonia vaccine D) ask patient if he had taken flu and pneumonia vaccine E) ask patient if he has taken pneumonia vaccine only 10. What is inappropriate for acute asthma? A) beta2 agonist inhaler B) iv hydrocortisone C) iv epinephrine D) inhaler anticholinergic 11. A 55 yo male admitted in emergence with asthma exacerbations. What is the preferred drug of choice? A) corticosteroids B) salbutamol nebulizer C) antihistamine D) theophylline 12. Which of the following asthma device at home helps patient to determine when to go emergence? A) Peak expiratory flow rate (PEFR) B) Forced expiratory volume in one second (FEV1) C) Forced vital capacity (FVC) D) Total lung capacity Copyright © 2000-2016 TIPS Inc. Unauthorized reproduction of this manual is strictly prohibited and 74-2 it is illegal to reproduce without permission. This manual is being used during review sessions conducted by PharmacyPrep.

www.PharmacyPrep.Com

Asthma and COPD

13. Which of the following is NOT a side effect of oral prednisone? A) weight gain B) acute pancreatitis C) Hypertension D) viral infections E) Steven Johnson syndrome 14) The following medications are used in asthma emergencies: A. Salbutamol B. Ipratropium C. Salmeterol D. All of the above 15) Asthma acute exacerbation can be triggered by all of the following EXCEPT: A. warm and dry weather B. stressful emotional time C. bacterial and viral pneumonia D. withdrawal of asthma medications 16) Emphysema is characterized by: A. Permanent enlargement of small sacs in lungs B. Symptoms are scanty sputum C. Shortness of breath. D. All of the above 17) Which of the following inhaler require rinsing mouth after use. A. Servent (salmetrol) B. Ventolin (albuterol) C. Atrovent (ipratropium) D. Pulmicort (budesonide) 18) In emergency treatment of asthma, the preferred first line treatment is: A. theophylline B. salbutamol C. cromolyn sodium D. Zafirlukast 19. Nosocomial infection is A. Nasal infection B. Mixed bacterial infection C. Mixed viral infection D. Infections acquired in a hospital or other health care settings

Copyright © 2000-2016 TIPS Inc. Unauthorized reproduction of this manual is strictly prohibited and 74-3 it is illegal to reproduce without permission. This manual is being used during review sessions conducted by PharmacyPrep.

www.PharmacyPrep.Com

Asthma and COPD

20. Drugs that Acts as long acting B 2 adrenergic agonist: A. Isoetharine B. Metaproterenol C. Terbutaline D. Salmeterol 21) Which drug is not use in asthma? A. B2 adrenergic antagonist B. Methylxanthine C. Leukotriene antagonist D. Corticosteroid 22) All of the following are side effects of Salbutamol except: A. Tremors B. Palpitation C. Restlessness D) Oral thrush 23) Therapeutic use of Pulmicort: A. Prevent Nocturnal asthma B. Prevent exercise induced asthma (EIA) C. Indicated in COPD and Emphysema D. All of the above 24) Which drug decreases effect of Theophylline? A. Ciprofloxacin B. Allopurinol C. Pulmonary edema D. Carbamazepine 25) Drug of choice for Aspirin induced asthma: A. Salbutamol B. Zafirlukast C. Montelukast D. Fluticasone 26) Which anticholinergic drugs is used in asthma? A. Ipratropium bromide B. Scopolamine C. Pilocarpine D. Atropine

Copyright © 2000-2016 TIPS Inc. Unauthorized reproduction of this manual is strictly prohibited and 74-4 it is illegal to reproduce without permission. This manual is being used during review sessions conducted by PharmacyPrep.

www.PharmacyPrep.Com

Asthma and COPD

27) Fluticasone drugs are use to: A. Control inflammation in asthma and allergic rhinitis. B. Control asthma only C. Treat Aspirin induced asthma D. Relaxes the muscles surrounding the bronchioles. 28. Combivent available dosage forms is/are: A. Nebulizers B. MDI C. A & B D. Capsule 29. Risk factors that causes COPD except: A. Second hand smoke B. Family history C. Asthma D. Age 30. High risk factor for COPD A. Asthma B. Smoking C. Second hand smoke D. Air pollution 31. Function as mast cell inhibitor: A. Cromolyn sodium B. Budesonide C. Zafirlukast D. Beclomethasone 32) Onset of action for short acting drugs is? A. 10-15 minutes B. 10-20 seconds C. 10-15 seconds D. 10-20 minutes 33) What to do to reduce corticosteroids side effects? A. Stop corticosteroid B. Rinse mouth and gargle after using the drug C. Chew a gum D. Brush your teeth every after using the drug

Copyright © 2000-2016 TIPS Inc. Unauthorized reproduction of this manual is strictly prohibited and 74-5 it is illegal to reproduce without permission. This manual is being used during review sessions conducted by PharmacyPrep.

www.PharmacyPrep.Com

Asthma and COPD

34) Drugs are not affected by catecholamine O-methyl transferase (COMT) enzymes except: A. Salmeterol B. Salbutamol C. Metoproterenol D. Terbutaline 35) Function as mast cell stabilizers: A. Zafirlukast B. Ipratropium C. Prednisone D. Nedocromil 36) Rx: Beclomethasone (Vanceril) inhaler: dispense # 1 Sig: ii puffs inhaled bid. In filling the prescription, the technician should: A. Inform the patient that this is a corticosteroid that has many serious side effects. B. Include the following specific directions on the label: “Inhale 2 puffs into each nostril 3x a day” C. Provide the patient the package insert included with the inhaler D. Tell the patient to use this inhaler only when he really needs it.

37) All of the following drugs can be indicated for the treatment of asthma EXCEPT: A. Ipratropium B. Metoprolol C. Albuterol D. Zafirlukast 38) A 45 yo patient with emphysema and pain, and is not controlled, the doctor switched him to morphine and his daughter came to pharmacy, you will call the doctor because: a) You will ask to change the medication b) To tell the dose of medication because its too much c) To ask for written prescription of morphine d) Her concern about constipation 39) What is true about Ipratropium? I. Reversible competitive of anticholinergic II. Synergistic with Theophylline in asthma III. Quaternary ammonium compound a) I only b) III only c) I and III only d) II and III only e) I, II, III Copyright © 2000-2016 TIPS Inc. Unauthorized reproduction of this manual is strictly prohibited and 74-6 it is illegal to reproduce without permission. This manual is being used during review sessions conducted by PharmacyPrep.

www.PharmacyPrep.Com

Asthma and COPD

40) Which of the following is incorrect statement of leukotriene receptors inhibitors? A) used to control the dose of steroids B) Indicated to treat congestion in children over 2 yr C) Treatment of choice for beta blockers and Aspirin induced asthma D) Patient used leukotriene receptors inhibitors require increase requirement of steroids. E) Montelukast is used in children over 2 yr age 41) A 40 yr old asthma patient brings a new prescription of montelukast bid. Currently medication profile include salbutamol prn, Symbicort (budesonide and formoterol) 1 puff bid. What is the action to do? A) ask patient to stop using Symbicort B) ask patient to stop using salbutamol prn C) dispense montelukast and counsel how to use it D) Contact doctor to change Symbicort to Advair E) Contact doctor to change to Zafirlukast 42) A COPD patient brings a prescription of doxycycline daily to treat pneumonia. Which of the following is the correct statement? A) ask patient if he is taken flu vaccine B) ask patient if his close contacts have taken flu vaccine C) ask patient if his close contacts have taken flu and pneumonia vaccine D) ask patient if he had taken flu and pneumonia vaccine E) ask patient if he has taken pneumonia vaccine only 43) What is inappropriate for acute asthma? A) beta2 agonist inhaler B) iv hydrocortisone C) iv epinephrine D) inhaler anticholinergic 44) A 55 yo male admitted in emergence with asthma exacerbations. What is the preferred drug of choice? A) corticosteroids B) salbutamol nebulizer C) antihistamine D) theophylline 45) Which of the following asthma device at home helps patient to determine when to go emergence? A) Peak expiratory flow rate (PEFR) B) Forced expiratory volume in one second (FEV1) C) Forced vital capacity (FVC) D) Total lung capacity 46) Which of the following is NOT a side effect of oral prednisone? A) weight gain B) acute pancreatitis C) Hypertension D) viral infections E) Steven Johnson syndrome Copyright © 2000-2016 TIPS Inc. Unauthorized reproduction of this manual is strictly prohibited and 74-7 it is illegal to reproduce without permission. This manual is being used during review sessions conducted by PharmacyPrep.

www.PharmacyPrep.Com

Asthma and COPD

47) What is true about ipratropium? I) It is reversible competitive inhibitor of acetylcholine II) this give synergistic effect to theophylline in asthma III) Quaternary amine salts a) I only b) III only c) I and III only d) II and III only e) I, II, III 48) The volume of air that passes in and holds and out of the lungs in normal breathing is called? A. Total lung capacity B. Tidal volume C. Breathing capacity D. Peak flow E. Vital capacity 49) Wheezing results from? A. obstruction of airways due to narrowing B. irreversible obstruction of airways due to permanent enlargment of alveoli C. obstruction of airways due to particles D. onstruction of airways due to infection 50) Orthopnea is? A. Shortness of breath when a person lie down but relieved when sit. B. Shortness of breath when a person lie sit but relieved when lie down C. Shortness of breath when a person standing and relieved when sit D. Shortness of breath when a person sit and relieved when stand 51.Which of the following is diagnostic test NOT related asthma? A. spirometer B. bronchoprovocation challenge test C. peak flow meter D. alpha1 antiprotease levels E. methanacholine or histamine test

52. What is the drug of choice for excercise induced asthma? A. Salbutamol B. ICS C. Fluticasone D. Salmeterol Ans. A 53.What drug is used if asthma is not controlled by salbutamol use alone? A. Prednisone Copyright © 2000-2016 TIPS Inc. Unauthorized reproduction of this manual is strictly prohibited and 74-8 it is illegal to reproduce without permission. This manual is being used during review sessions conducted by PharmacyPrep.

www.PharmacyPrep.Com

Asthma and COPD

B. Fluticasone inhaler C. hydrocortisone injection D. Long acting beta agonist E. Tiotropium Ans. b 54. What is the drug of choice for asthma in hospital emergency? A. fluticasone B. oral prednisone C. hydrocortisone inj D. topical steroid E. oxygen Ans. c 55. Which of the following is used for asthma attacks or exacerbations? I. Salbutamol II. Ipratropium III. Theophylline A. I only B. III only C. I and II D. II and III E. I,II,III Ans.C 56. Dysphonia is side effect of? A. ipratropium B. Prednisone oral C. Hydrocortisone injection D. Formeterol E. Fluticasone inhaler ans. e 57. When do you use omalizumab? A. Allergens induced asthma B. excercise induced asthma C. Drug induced asthma D. Asthma maintenance therapy E. All of the above Ans. A 58. In COPD which of the following is least likely used? A. Tiotropium B. Salmeterol C. Formeterol D. Fluticasone inhaler E.oral prednisone Copyright © 2000-2016 TIPS Inc. Unauthorized reproduction of this manual is strictly prohibited and 74-9 it is illegal to reproduce without permission. This manual is being used during review sessions conducted by PharmacyPrep.

www.PharmacyPrep.Com

Asthma and COPD

Ans. d 59. A copd patient prescribed theopylline. Which of the following least likely toxic symptoms if theophylline serum levels are over 10-20 mcg/ml? A.vomiting B. tachycardia C. sweating D. anxiety & restlness E.sedation Ans. E 60. Which of the following least likely used in COPD or emphysema? A. antibiotics B. salbutamol C. ipratropium D. theophylline E. ICS Ans. E

Copyright © 2000-2016 TIPS Inc. Unauthorized reproduction of this manual is strictly prohibited and 74-10 it is illegal to reproduce without permission. This manual is being used during review sessions conducted by PharmacyPrep.

www.PharmacyPrep.Com

Asthma and COPD

ASTHMA AND COPD ANSWERS: 1)Ans: C Tips: neonates and infants have slowest rate of clearance 2) Ans: C Tips: Theophylline is substrate of CYP1A2, and ciprofloxacin is the inhibitor of CYP1A2 3) Ans: B 4) Ans: A 5) Ans: D 6. Ans: E Tips: Theophylline is added id doses not response to anticholinergic) 7) Ans: D 8) Ans: C 9) Ans: D 10. Ans: C 11. Ans: B Tips: In an emergence situation, the most rapidly acting agent is used first. The choice of route of administration depends on severity of attack. 12. Ans: A 13. Ans: E 14) Ans-A 15) Ans-A 16) Ans: D 17) Ans-D 18) Ans-B Copyright © 2000-2016 TIPS Inc. Unauthorized reproduction of this manual is strictly prohibited and 74-11 it is illegal to reproduce without permission. This manual is being used during review sessions conducted by PharmacyPrep.

www.PharmacyPrep.Com

Asthma and COPD

19. Ans: D 20. Ans: D 21) Ans: A 22)Ans: D 23)Ans: D 24) Ans: D 25) Ans: C ( it was B before) 26) Ans: A 27) Ans: A 28. Ans: C 29. Ans: D 30. Ans: B 31. Ans: A 32) Ans: D 33) Ans: B 34) Ans: A Tips: Short acting is not affected by COMT enzyme because this is activated by non COMT enzyme. 35) Ans: D 36) Ans: C

37) Ans: B Tips: Asthma is characterized by increased responsiveness of trachea and bronchi to various stimuli and narrowing of airways. It is associated with shortness of breath, chest tightness, wheezing and coughing. The Beta-2 receptors stimulation generally dilates the bronchial smooth muscles and helps in controlling asthma. Copyright © 2000-2016 TIPS Inc. Unauthorized reproduction of this manual is strictly prohibited and 74-12 it is illegal to reproduce without permission. This manual is being used during review sessions conducted by PharmacyPrep.

www.PharmacyPrep.Com

Asthma and COPD

38) Ans: D 39) Ans: E Tips: Theophylline is added id doses not response to anticholinergic) 40) Ans: D 41) Ans: C 42) Ans: D 43) Ans: C 44) Ans: B Tips: In an emergence situation, the most rapidly acting agent is used first. The choice of route of administration depends on severity of attack. 45) Ans: A 46) Ans: E 47) Ans: E 48)Ans. B 49) Ans. A 50) Ans. A 51) Ans. D (no answer was given)

Copyright © 2000-2016 TIPS Inc. Unauthorized reproduction of this manual is strictly prohibited and 74-13 it is illegal to reproduce without permission. This manual is being used during review sessions conducted by PharmacyPrep.

PharmacyPrep.Com

Smoking Cessation PHARMACY PREP SMOKING CESSATION

1-What is incorrect statement about smoking cessation? A-Person using nicotine replacement therapy, should stop smoking same time as start NRT B-Person using bupropion can set quit date in 1 to 2 week of after start of treatment C-A person using nicotine patches can smoke but reduce number of cigarettes D-A person with hypertension, body weight less than 45 Kg should start with low dose of NRT i.e. step 2 patch E-Patches can be applied on while taking shower 2) Nicotine patches are stored in? A-Refrigerator B-Room temperature D-Freezer E-Any temperature

C-Cold temperature

3) What is incorrect about nicotine patches? A. Stored in room temperature B. Avoid sun light exposure C. Hygroscopic, thus open from pack when you ready to use D. Does not need to protect from light E. Can be applied on arms and thighs 4) What is incorrect about bupropion? A-Reduce the seizure threshold B- Avoid in bulimia nervosa C-Avoid in anorexia nervosa D-Avoid in sexual dysfunction E-It has low weight gain side effects 5-Which of the following is least likely caused by bupropion? A-anorexia B-weight gain and sexual dysfunction C- se re res D-hypertension E-gingivitis 6) All of the following can be used in smoking cessation, EXCEPT: A) Fluoxetine B. bupropion C. Nicotine transdermal patch D. Nicotine inhaler E. Nicotine gum

Copyright © 2000-2016 TIPS Inc. Unauthorized reproduction of this manual is prohibited. This manual is being used during review sessions conducted by PharmacyPrep

75-1

PharmacyPrep.Com

Smoking Cessation

7. What is the first line therapy for smoking cessation? a. nicotine replacement therapy B. bupropion C. varenicline D. Clonidine E. atomoxetine Ans. A

Copyright © 2000-2016 TIPS Inc. Unauthorized reproduction of this manual is prohibited. This manual is being used during review sessions conducted by PharmacyPrep

75-2

PharmacyPrep.Com

Smoking Cessation

SMOKING CESSATION ANSWERS: 1) Ans: C 2) Ans: B 3) Ans: D 4) Ans: D Tips: bupropion has least sexual dysfunction among antidepressants. 5) Ans: B 6) Ans: A

Copyright © 2000-2016 TIPS Inc. Unauthorized reproduction of this manual is prohibited. This manual is being used during review sessions conducted by PharmacyPrep

75-3

PharmacyPrep.Com

Sleeping Disorders PHARMACY PREP SLEEPING DISORDERS

1) What is incorrect recommendations for persons complaining insomnia? A-make a regular sleep schedule for 7 days/wk B-Over the counter diphenhydramine for 3 days may be helpful C-It is essential to exercise before bedtime D-Avoid heavy meals before bedtime E-Avoid long day nap can give good sleep in night 2-What is incorrect about zopiclone? A-A non benzodiazepine act on benzodiazepine receptor B-Associated with sever hangover effect C-It indicated for short time insomnia D-It is short acting E-It has low dependency 3-All of the following are long acting benzodiazepines except? A-Diazepam B-Clonazepam C-Flurazepam D-Temazepam E-Chlorozepate Tips: TEM are short acting Temazepam Estazolam, and Midazolam 4) Zopiclone is? A-Non-benzodiazepine act on BZ 1 receptors B-Benzodiazepine act on BZ 1 receptors C-Non-benzodiazepine act on BZ 2 receptors D-Non-benzodiazepine act on BZ 1 and BZ 2 receptors E-Benzodiazepine act on BZ 2 receptors 5) A 75 yo senior is receiving treatment of anxiety, paroxetine daily, clonazepam 5 mg bid, oxazepam 10 mg tid. Which of the following is the important to monitor? A) Insomnia B) Anxiety C) Disorientation and falls D) Confusion E) Depression 6) Which of the following drug has NOT taste change side effects? A) zopiclone B) captopril C) metformin D) tetracycline E)metronidazole

Copyright © 2000-2016 TIPS Inc. Unauthorized reproduction of this manual is prohibited. This manual is being used during review sessions conducted by PharmacyPrep

76-1

PharmacyPrep.Com

Sleeping Disorders

7) Which of the following drug has faster rebound insomnia side effects? A) Triazolam B) Diazepam C) Oxazepam D) lorazepam E) Alprazolam 8) What benzodiazepines are preferable to treat the type of insomnia that is associated with difficulty in falling in sleep? A) Short acting B) Intermediate acting C) Long acting D) intermediate and long acting E) short and long acting 9) MK is 50 year patient comes to pick up prescription of lorazepam 0.5 mg tab. Which of the following statement is relevant? A. Taking alcohol concomitant with lorazepam can cause severe sedation thereby avoid. B) Take benzodiazepine with food C) avoid taking with grapefruit juice D) All of the above

Copyright © 2000-2016 TIPS Inc. Unauthorized reproduction of this manual is prohibited. This manual is being used during review sessions conducted by PharmacyPrep

76-2

PharmacyPrep.Com

Sleeping Disorders SLEEPING DISORDERS ANSWERS

1) Ans: C Tips: avoid exercise at least 2 to 3 hours before bedtime 2) Ans: B 3) Ans: D 4) Ans: A 5) Ans: C Tips: Lethargy = fatigue; Hangover = disorientation; Sedation = sleepy; Somnolence = sleepy 6) Ans: D 7) Ans: A 8) Ans: A 9) Ans: A

Copyright © 2000-2016 TIPS Inc. Unauthorized reproduction of this manual is prohibited. This manual is being used during review sessions conducted by PharmacyPrep

76-3

PharmacyPrep.Com

Eating disorders and weight loss treatments

PHARMACY PREP EATING DISORDER AND WEIGHT LOSS TREATMENTS 1. What is incorrect about anorexia nervosa? I-It is characterized by deliberate loss of weight II-refusal to maintain normal body weight, fear of weight gain and amenorrhea III-Binge eating is common in anorexia nervosa A-I only B-III only C-I and II only D-II and III only E-All of the above 2-You notice Ms A’s, a young girl of normal weight, in the pharmacy purchasing laxatives. You take the time to talk to her about her laxative use, and find that she has been binge eating and using laxatives about three times per week for the last six months. Based on this information, which of the following statements is most likely to be correct? A. She may have bulimia nervosa of the non-purging type since she is not self-inducing vomiting B. She may have bulimia nervosa of purging type since she uses laxatives C. She does not have bulimia nervosa, since she is of normal weight D. She does not have bulimia nervosa, since her bingeing episodes are not frequent enough 3-What is overweight body mass index (BMI) A. 18.5 to 24.9 B. 25.0 to 29.9 C. >30 D. 30 D. 30 D- 5 mU/L indicate hypothyroidism. 17- Ans: B Copyright © 2000-2016 TIPS Inc. Unauthorized reproduction of this manual is prohibited. This manual is being used during review sessions conducted by PharmacyPrep.

80-7

Pharmacyprep.com

18- Ans: E 19- Ans: B 20) Ans: E Tips: increase serum TSH and decrease Free T4 and TT3 21) Ans: B

22) Ans: D 23) Ans: E 24) Ans: D Tips: FT4, TT4, TT3 and FTI decrease in hypothyroidism, only serum TSH increase. For hyperthyroidism, exactly opposite changes. 25) Ans: E 26) Ans: B 27) Ans: C 28) Ans: D 29) Ans: C 30) Ans: E 31) Ans: D 32. Ans: B Tips: patient experiencing overdose or hyperthyroidism symptoms, may be due overdose of levothyroxine. So refer to doctor to decrease dose of levothyroxine 33. Ans: D Tips: In hyperthyroidism increase T4 and T3. 34. Ans: A 35. Ans: E

Copyright © 2000-2016 TIPS Inc. Unauthorized reproduction of this manual is prohibited. This manual is being used during review sessions conducted by PharmacyPrep.

80-8

Pharmacyprep.com Tips: Food and formula containing large amount of soybean, fiber, or iron should not be used for administration of levothyroxine. However low fiber food does not interact with thyroxin. 36. Ans: A

Copyright © 2000-2016 TIPS Inc. Unauthorized reproduction of this manual is prohibited. This manual is being used during review sessions conducted by PharmacyPrep.

80-9

Pharmacyprep.com

Copyright © 2000-2016 TIPS Inc. Unauthorized reproduction of this manual is prohibited. This manual is being used during review sessions conducted by PharmacyPrep.

80-10

PHARMACY PEP CONTRACEPTION 1-Contraceptive methods that protect STDs such as, gonorrhea, syphilis, chlamydia, and AIDS? A-Oral contraceptive B-Condoms C-Contraceptive sponges D-Vaginal cream E-Plan B 2-A 18 year old women walks into pharmacy, wants to buy emergency contraception Plan-B. Pharmacist on duty believes do not allow dispensing contraceptive, and refuse to give oral contraceptives. What ethical principle is violated? A-beneficence B-Non maleficence C-Autonomy D-veracity E-Justice 3) What is the most common side effects plan B? A-abdominal pain B-Constipation C-Nausea and vomiting D-vaginal bleeding E-Deep vein thrombosis 4)If one contraceptive pill missed, recommend? A-take two pills next day B-take 3 pills next day C-take one pill next day D-use alternative method of protection for next week E-Refer to doctor 5) A customer of your pharmacy she asking for your opinion to choose a contraceptive method, that she does not want to use daily and convenience and also near future she has plan to start family. What is the good contraceptive you recommend? A)OCP B) Condoms C)Evra path D)Nuvaring E) contraceptive injections 6) Which of the following questions is appropriate to ask a customer who wants to buy plan B? I)when was the last period II) are you pregnant? III)how long ago the unprotected intercourse A) I only B) III only C) I and II only D) II and III only E)I, II, III

Copyright © 2000-2016 TIPS Inc. Unauthorized reproduction of this manual is prohibited. This manual is being used during review sessions conducted by PharmacyPrep.

81-1

7) Which of the following contraceptive methods prevents gonorrhea and chlamydea infections? A) Transdermal contraceptives B) Intrauterine device C) Vaginal cream D) Subdermal progesterone implants E) None of the above 8) Many women who take OCs are poorly informed about the proper use of these medications. Which of the following is the first effective methods of providing information? A) The patient first should be given package insert of OCs B) Auxiliary labels on OCs C) Verbal information describing the way medication works D) Patient should be told that this do not protect STIs E) Patient using these products should have knowledge of this product from internet 9) What is correct about transdermal contraceptive Evra patch? I) The patch should be applied to the abdomen, buttocks at the beginning of menstrual cycle II) The patch should be applied to the upper torso, and upper arm at the beginning of menstrual cycle III) The patch should be applied to the chest, and behind the ear at the beginning of menstrual cycle A) I only B) III only C) I and II D) II and III E) I, II, III 10. What is contraceptive is the drug of choice in breast feeding women? A. COC B. emergency contraceptive C. progestin only D. IUDs ans. C 11.A female client is using combined oral contraceptive pills. She forgot to take one pill on her second week of 21 contraceptive pills. She had Sunday start. What is appropriate recommendation? a. Take two pills next day b. Stop taking COC till Sunday and start a new pack from Sunday c. Take two pills on Sunday D. Stop taking COC till Sunday and take two pills on Sunday E. Stop using contraceptives and see doctor ans. a

Copyright © 2000-2016 TIPS Inc. Unauthorized reproduction of this manual is prohibited. This manual is being used during review sessions conducted by PharmacyPrep.

81-2

12.Which of the following the side effect of emergency contraception? a. breast tenderness B. stomach upset C. Deep vein thrombosis D. menstrual bleeding E. none of the above Ans. b

CONTRACEPTION ANSWERS: 1- Ans: B 2- Ans: C 3) Ans: C 4) Ans: A 5) Ans: D 6) Ans: E Copyright © 2000-2016 TIPS Inc. Unauthorized reproduction of this manual is prohibited. This manual is being used during review sessions conducted by PharmacyPrep.

81-3

7) Ans: E Tips: STI are only prevented by condoms contraceptive methods 8) Ans: A 9) Ans: C ( no answer given before)

Copyright © 2000-2016 TIPS Inc. Unauthorized reproduction of this manual is prohibited. This manual is being used during review sessions conducted by PharmacyPrep.

81-4

Copyright © 2000-2016 TIPS Inc. Unauthorized reproduction of this manual is prohibited. This manual is being used during review sessions conducted by PharmacyPrep.

81-5

Pharmacyprep.com PHARMACY PREP GENITOURINARY CONDITIONS 1) Toxic shock syndrome is caused by: A-E. coli B-S. aureus C-Chlamydia D-Gonorrhea E-Non gonococci 2-Toxic shock syndrome may have risk with the use of: I-Tampons use II-Diaphragm contraceptive III-Condoms as contraceptive A-I only B-III only C-I and II only D-II and III only E.I, II, III 3-Finastride is alpha reductase inhibitor, indicated in: I-Benign prostate hyperplasia II-Alopecia III-Increase sexual function a) I only b) III only c) I and II only d) II and III only e) E.I, II, III 4-All of the following are the benign prostate hyperplasia, (BPH) symptoms, EXCEPT: A-Nocturia B-Frequent urination C-Jet urination D-Urinary retention E-Post void dripping 5-Correct statement about pre menstrual symptoms (PMS) include: I-All women have PMS II-Luteal phase of cycle have PMS III-vitamins, and estrogens deficiency may cause PMS a) I only b) III only Copyright © 2000-2016 TIPS Inc. Unauthorized reproduction of this manual is prohibited. This manual is being used during review sessions conducted by PharmacyPrep.

82-1

Pharmacyprep.com c) I and II only d) II and III only e) I, II, III 9. Urinary incontinence is NOT aggravated by: A-Hydrochlorthiazide B-Furosemide C-Diphenhydramine D-Acetazolamide E-None of the above 10-Patient profile includes sildenafil 50 mg and brings a prescription of erythromycin for 5 days upper respiratory infections, what is appropriate advice? A-call doctor to decrease dose of sildenafil 25 mg while using erythromycin B-Stop using sildenafil while using erythromycin C-Call doctor to change other antibiotic D-all of the above 11) A 50 year old lady will begin to take Hormone Replacement Therapy. All of the following questions will help achieved appropriateness of her medication EXCEPT: A) Did she have any children? B) When did you last monitor your lipids? C) Hysterectomy D) When is the last menstruation period? 12. A patient is being treated with UTI, what is important to treat? I. Dysuria and burning II. Vaginal candidiasis III. Fever and flare ups a. I only b. III only c. I and III only d. II and III only e. I, II, III 13) If a patient is not treated for asymptomatic sexually transmitted infections. What is correct? A) symptoms are obvious after few days B) can cause pelvic pain and dysmenorrhea C) can cause pelvic inflammatory disease and infertility D) can cause endometrial cancer E) can results in cervical cancer

Copyright © 2000-2016 TIPS Inc. Unauthorized reproduction of this manual is prohibited. This manual is being used during review sessions conducted by PharmacyPrep.

82-2

Pharmacyprep.com 14) What is correct about premenstrual symptoms? I) all women have premenstrual symptoms II) pre menstrual symptoms occur during luteal phase III) a natural product prime rose oil is used A) I only B) III only C) I and II only D) II and III only E) I, II III 15) What is incorrect about toxic shock syndrome? A) caused by infections of S. aureus B) can cause by tampon use C) Can cause by condom use D) can cause by candida infections E) Can cause cervical cap contraceptives 16) Which of following condition avoid using sildenafil? A) patients experienced priapism B) patient have reported visual disturbances C) patient using nitrates D) patient have diplopia E) children or women with hypertension 17) A customer of your pharmacy presents with symptoms of vaginal discharge, yellow and fishy odor. A) Recommend over the counter antifungal drugs B) Refer to doctor is appropriate because this bacterial infection C) Recommend self care D) None of the above 18) What is the mechanism action of finasteride? A-Blockade of intestinal lipase B-Blockade the conversion of estrogen to dihydroestrogen C-Blockade of testosterone to dihydrotestosterone D-Blockade of sebaceous glands E-Blockade of hair growth 19) What period of menstrual cycle pre-menstrual symptoms (PMS) occur? A-Follicular phase B-Ovulation phase C-Immediately after ovulation D-Luteal phase E-During menstruation Copyright © 2000-2016 TIPS Inc. Unauthorized reproduction of this manual is prohibited. This manual is being used during review sessions conducted by PharmacyPrep.

82-3

Pharmacyprep.com 20) Which of the following methods of contraception can prevent sexually transmitted diseases A-oral contraceptives B-reservoir type contraceptive C-Intrauterine devices D-condoms E-contraceptive sponges 21. What is not menopause symptom? A. Nigh sweating B. Hot flushes C. Mood changes D. vasomotor symptoms E. Fever ans. E 23.Estrogen vaginal suppositories are used as? A. contraception B. to treat acne C. to treat endometriosis D. to treat vasomotor symptoms E. To treat sexual dysfunction Ans. d Tips: vasomotor symptoms are hot flushes, night sweating, vaginal dryness can be treated estrogen vaginal suppositories. 24. Which of the following is NOT associated with toxic shock syndrome? A. Tampon B. Cervical cap C. contraceptive sponges D. condoms E. IUDs Ans. D 25. Premenstrual symptoms occurs during? A. Ovulation phase B. Follicular phase C. Luteal phase D. menstruation E. post menopause. Ans.C

Copyright © 2000-2016 TIPS Inc. Unauthorized reproduction of this manual is prohibited. This manual is being used during review sessions conducted by PharmacyPrep.

82-4

Pharmacyprep.com 26. What is the drug of choice to treat patient with vaginal pruritus, curdy vaginal discharge, no odor and no color discharge? A. Metronidazole vaginal cream B. Metronidazole oral C. Clotrimazole D. Amoxicillin E. Cotrimoxazole Ans. C

Copyright © 2000-2016 TIPS Inc. Unauthorized reproduction of this manual is prohibited. This manual is being used during review sessions conducted by PharmacyPrep.

82-5

Pharmacyprep.com CORRECT ANSWERS Genitourinary Conditions 1- Ans: B 2- Ans: C 3- Ans: C 4- Ans: C 5- Ans: D 9. Ans: C Tips: diphenhydramine belongs to the ethanolamine class of antihistamines gives anticholinergic side effects such as urinary retention, and induce sedation. Hydrochlorothiazide, furosemide and acetazolamide are diuretics increase urinary frequency. 10- Ans: A 11) Ans: A 12. Ans: E 13) Ans: C 14) Ans: D 15) Ans: C 16) Ans: C 17) Ans: B Tips: Color discharge and fishy odor is indicator of bacterial infection, thus refer to doctor. 18) Ans: C (no answer was given from 18-20) 19) Ans: D 20) Ans: D

Copyright © 2000-2016 TIPS Inc. Unauthorized reproduction of this manual is prohibited. This manual is being used during review sessions conducted by PharmacyPrep.

82-6

Pharmacyprep.com

Copyright © 2000-2016 TIPS Inc. Unauthorized reproduction of this manual is prohibited. This manual is being used during review sessions conducted by PharmacyPrep.

82-7

Pharmacyprep.com

PHARMACY PREP. BONES AND JOINT COMPLICATIONS

1. What is not a screening test for rheumatoid arthritis? A. ESR B. CRP C. Rheumatoid factor D. BMD E. joint pain Ans. d 2. Infliximab is biological response modifier is product of? A. monoclonal antibody B. humanize MAB C. Murine MAB D. Chimeric MAB E. All of the above Ans. d 3. If patient is not treated for hyperuricemia. What happens? A. Tophaceous gout (chronic gout). B. Severe hyperuricemia C. Acute gout attacks D. Osteoarthritis E. Osteoporosis Ans. A 4. Febuxostat mechanism of action? A. antihyperuricemic agent B. Uricosuric agent C. Xanthine oxidase inhibitor D. Uric acid synthesis inhibitor E. Used for acute gout Ans. c

Copyright © 2000-2016 TIPS Inc. Unauthorized reproduction of this manual is prohibited. This manual is being used during review sessions conducted by PharmacyPrep.

83-1

Pharmacyprep.com

Copyright © 2000-2016 TIPS Inc. Unauthorized reproduction of this manual is prohibited. This manual is being used during review sessions conducted by PharmacyPrep.

83-2

PharmacyPrep.Com

Osteoporosis

PHARMACY PREP OSTEOPOROSIS 1) These two components in bone are responsible for the hardness and pliability of bone: a) osteoclasts & collagen b) mineralized salts & osteocytes c) mineralized salts & collagen d) collagen & elastic fibres e) collagen & mesenchyme 2) A fracture in the shaft of a long bone would be a break in the: a) epiphysis b) metaphysis c) diaphysis d) epiphyseal plate e) mesenchyme 3) Yellow marrow consists of: a) osteoprogenitor cells b) blood cell progenitor cells c) hyaline cartilage d) adipose e) spongy bone 4) Chondroblasts produce: a) basement membranes b) bone matrix c) cartilage matrix d) mesothelium e) endothelium 5) These structures are at the center of compact bone lamellae and carry blood vessels along the bone length: a) Haversian canals b) canaliculi c) perforating canals d) osteocytes e) lacunae 6) The cell type that is responsible for maintaining bone matrix once it has formed is: a) osteoclasts b) chondrocytes c) osteocytes Copyright © 2000-2016 TIPS Inc. Unauthorized reproduction of this manual is prohibited. This manual is being used during review sessions conducted by PharmacyPrep.

84-1

PharmacyPrep.Com

Osteoporosis

d) fibroblasts e) osteoblasts 7) Soft connective tissue membranes between the cranial bones at birth are: a) an indication of microcephaly b) frontal sinuses c) epiphyseal plates d) cribriform plates e) fontanelles 8) Endochondral and intramembranous are two mechanisms of: a) bone remodeling b) embryonic skeletal ossification c) controlling blood calcium levels d) cartilage synthesis 9) The two pairs of bones that make up the hard palate are the right and left: a) zygomatic and temporal b) palatine and maxillae c) maxillae and zygomatic d) maxillae and mandible 10) The two bones that make up the posterior nasal septum are the: a) nasal and lachrymal b) inferior nasal choncae and vomer c) vomer and ethmoid d) ethmoid and sphenoid 11) Which of the following is the most serious or life-threatening: a) deviated nasal septum b) sinusitis c) damaged cribriform plate d) damaged or cleft palate e) ruptured bursae 12)All of the following are the risk factors for osteoporosis, except? A-Race B-Family history C-Obesity D-Smoking E-Excessive coffee intake 13- Importance of Vitamin D in osteoporosis: I- Increase the absorption of calcium from the small intestine Copyright © 2000-2016 TIPS Inc. Unauthorized reproduction of this manual is prohibited. This manual is being used during review sessions conducted by PharmacyPrep.

84-2

PharmacyPrep.Com

Osteoporosis

II- The best vitamin D analog to treat osteoporosis is calcitriol III- Antacids containing aluminum and magnesium may interfere with proper activity of vitamin D. a) I only b) III only c) I and II only d) II and III only e) I, II, III 14. All of the following are side effect of a systemic corticosteroids prednisone: EXCEPT: A-Weight gain B-Glucose intolerance C-Osteoporosis D-Cataract E-reduce infection 15) Osteoporosis caused by the deficiency of the following: I. Inadequate intake of vitamin D II. Inadequate intake of calcium III. Estrogen deficiency a) I only b) III only c) I and II only d) II and III only e) I, II, III 16-What calcium supplement has the most elemental calcium? A-Calcium citrate B-natural calcium C-Calcium carbonate D-dairy products E-Calcium gluconate 17) The type of calcium supplement produced from oyster’s shells is? A-Calcium citrate B-natural calcium C-Calcium carbonate D-dairy calcium E-Calcium gluconate 18-What is calcium supplement daily dose in adults? A-1 g B- 500 mg C-800 mg D-100 mg

E- 2 g

19-What is incorrect about alendronate? A-Take first thing in the morning empty stomach with 1 full glass of water B-Contraindicated renal diseases C-70 mg/wk dose have equivalent effect as daily 10 mg dose Copyright © 2000-2016 TIPS Inc. Unauthorized reproduction of this manual is prohibited. This manual is being used during review sessions conducted by PharmacyPrep.

84-3

PharmacyPrep.Com

Osteoporosis

D-Monitor hypercalcemia before initiating treatment E-Do not lie down for 30 min after oral dose 20-A patient using alendronate 70 mg/wk and calcium supplement, what is incorrect? A-Avoid calcium supplement while using alendronate therapy B-It is important to take calcium supplement while using bisphosphonates therapy C-separate alendronate and calcium supplement at least 2 hour before and 4 after. D-It is also essential to take vitamin D supplements E-It is recommended to take daily 800 mg IU vitamin D 21- To monitor alendronate safety and efficacy all are needed EXCEPT: a) Hypercalcemia b) Bone pain and swelling c) Ca and Vitamin D intake d) Bone mineral density e) Esophagitis 22- What is correct about osteoporosis? A) Inadequate Ca and vitamin D can cause osteoporosis B) Alendronate can be used once month C) Zolendronoic acid injection is used daily D) Recommend daily vitamin D 2000 IU in over age 65yo E) Recommend swimming a good exercise to increase bone mineral density 23- Which of the following increase risk of osteoporosis? A) History of oral contraceptive use B) Smoking C) Physical activity like weight bearing exercises like stair climbing, walking, and jogging. D) Increase in dietary soy intake E) Increase intake of broccoli 24- A 35 yo women get the prescription of 50,000/wk unit of vitamin D. What to do? A) talk to doctor and dispense B) this could extremely high dose and may cause toxicity C) reduce dose to 500 units and dispense D) May be prescription error just dispense 500 units E) It is outrageous to get prescription of 50, 000 units 25- What is not risk factor of osteoporosis? a)estrogen deficiency B) androgen deficiency C) race D)age

E)protein diet

26- Which of the following is the least recommended exercises in osteoporosis? A) rope jumping B) basket ball playing C) horse riding D) jogging E) Dancing

Copyright © 2000-2016 TIPS Inc. Unauthorized reproduction of this manual is prohibited. This manual is being used during review sessions conducted by PharmacyPrep.

84-4

PharmacyPrep.Com

Osteoporosis

27- Which of the following is not a cranial suture: a) epiphyseal b) lambdoidal c) coronal d) sagittal e) squamous 28. Which of the following is least likely risk factor for osteoporosis? A) Inadequate vitamin D and Ca supplement intake B) Age C) family history of osteoporosis D) Prednisone therapy for over 3 months E) alcohol intake 29. What is the most likely risk for osteoporosis? A) smoking B) alcohol C) coffee D) tea E) none 30. what type of vertebral fracture in osteoporosis? A. compression fracture (back pain, decrease in height and round shoulder appearance) B. tendonitis C. cartilage break down D. Bone softening E. closed fracture Ans. A 31. What is drug is used to treat vertebral and non vertebral fracture? A. bisphosphonates B. Raloxafine C. Calcium supplement D. Teriperatide E. Calcitonin Ans. A 32. Which of the following drug has once year injection A. bisphosphonates B. Raloxafine C. Zoledronic acid D. Teriperatide E. Calcitonin Ans. C Copyright © 2000-2016 TIPS Inc. Unauthorized reproduction of this manual is prohibited. This manual is being used during review sessions conducted by PharmacyPrep.

84-5

PharmacyPrep.Com

Osteoporosis

33.Denosumab (Prolia) mechanism of action is? A. Human IgG2 monclonal antibody (RANKL) B. Pyrophosphate C. Chimeric antibody D. Hormone replacement therapy E. Parathyroid hormone analog ans. A

OSTEOPOROSIS ANSWERS: 1) Ans: C 2) Ans: C 3) Ans: D 4) Ans: C 5) Ans: A Copyright © 2000-2016 TIPS Inc. Unauthorized reproduction of this manual is prohibited. This manual is being used during review sessions conducted by PharmacyPrep.

84-6

PharmacyPrep.Com

Osteoporosis

6) Ans: C 7) Ans: E 8) Ans: B 9) Ans: B 10) Ans: C 11) Ans: C 12) Ans: C Tips: obesity or overweight is not a risk factor of osteoporosis. Race, family history, smoking, excessive coffee intake, deficiency of estrogen, and sedentary lifestyle are some of the risk factors. 13- Ans: E Tips: Osteoporosis is characterized by the loss of bone mass. Vitamin D supplements have been used in the treatment of osteoporosis because vitamin D increases the absorption of calcium that is essential to reduce bone loss. Antacids interact with vitamin D in many ways, magnesium containing antacids may lead to hypermagnesemia and aluminum containing antacids have its levels increased leading to aluminum bone toxicity when currently used with vitamin D. 14. Ans: E Tips: Corticosteroids increase the risk of infections. 15- Ans: E 16- Ans: C 17- Ans: C 18- Ans: A 19- Ans: D 20- Ans: A 21- Ans: A 22- Ans: A

Copyright © 2000-2016 TIPS Inc. Unauthorized reproduction of this manual is prohibited. This manual is being used during review sessions conducted by PharmacyPrep.

84-7

PharmacyPrep.Com

Osteoporosis

23- Ans: B 24- A 25- Ans: E Tips: protein diet like dietary are plant derived phytoestrogen present in soy proteins 26- Ans: C 27- Ans: A 28. Ans: E 29. Ans: C

Copyright © 2000-2016 TIPS Inc. Unauthorized reproduction of this manual is prohibited. This manual is being used during review sessions conducted by PharmacyPrep.

84-8

PharmacyPrep.Com

Osteoporosis

Copyright © 2000-2016 TIPS Inc. Unauthorized reproduction of this manual is prohibited. This manual is being used during review sessions conducted by PharmacyPrep.

84-9

PharmacyPrep.com

Hypertension PHARMACY PREP HYPERTENSION

1) According to Joint National Committee (JNC-7) standards. In measuring blood pressure of someone whose systolic blood pressure in the past had 150 mm Hg. Now if you like to check blood pressure. You would have to inflate the mercury bulb to? A-180 mm Hg B-150 mm Hg C-200 mm Hg D-120 mm Hg E-210 mm Hg Ans. A 2. What is correct about measuring blood pressure? A. Measure blood pressure twice on one hand after 5 min interval B. Measure blood once on left and then right hand after 5 min interval. C. Measure twice on each hand with 5 min interval D. Measure once on any hand E. Measure twice on left hand ans. B 3.. Which of the following main complication is NOT associated with persistent hypertension? A. Kidney B. Brain C. Retina of eye D. Liver E. diabetes Ans. d 4. Which of the following is the blood pressure goal for clients that 60 yo and above without diabetes or renal diseases? a. less than 120/80 b. less than 130/80 c. less than 140/80 d. less than 140/90 e. less than 150/90 ans. D

Copyright © 2000-2016 TIPS Inc. Unauthorized reproduction of this manual is prohibited. This manual is being used during review sessions conducted by PharmacyPrep.

85-1

PharmacyPrep.com

Hypertension

Copyright © 2000-2016 TIPS Inc. Unauthorized reproduction of this manual is prohibited. This manual is being used during review sessions conducted by PharmacyPrep.

85-2

PharmacyPrep.Com

Coronary Artery Diseases PHARMACY PREP CORONARY ARTERY DISEASES

1) Which one of the following factors is most directly attributed to angina attacks? A. Emotional Stress B. Heavy smoking C. Myocardial ischemia D. Obesity E. None of the above 2) Purple toe syndrome side effect is produced by: A-Amiodarone B-Reynaud phenomenon C-Warfarin D-Rifampin E-Verapamil 3. Which medication CAN NOT BE used for prinzmetal angina (vasospastic angina)? A. Amlodipine B. Metoprolol C. Acetyl salicylic acid D. Verapamil E. Diltiazem 4. Which effect of nitroglycerin is most likely responsible for its therapeutic action in angina pectoris? a. Dilation of coronary artery b. Relaxation of peripheral vascular smooth muscle c. Decrease myocardial after load d. A and B e. A and C 5. a) b) c) d) e)

Nitroglycerin dilates the coronary arteries in angina pectoris by Decreasing the heart rate reflex Increasing the metabolic work of the myocardium Direct action on smooth muscle in the vessel walls Increasing the effective refractory period in the atrium Blocking beta-adrenergic receptors

6. Which one of the following conditions is not associated with verapamil? A. Inhibition of calcium ion influx B. Negative inotropic effect C. Coronary vasodilatation effect D. Peripheral vasodilatation effect Copyright © 2000-2016 TIPS Inc. Unauthorized reproduction of this manual is prohibited. This manual is being used during review sessions conducted by PharmacyPrep.

86-1

PharmacyPrep.Com

Coronary Artery Diseases

E. None of the above 7-Which of the following anticholesterol medication may increase his blood sugar levels: A-Atorvastatin B-Niacin C-Simvastatin D-Fluvastatin E-All of the above 8-Administration of which of the following drugs is most likely to PROLONG clotting time? A. Morphine B. Vitamin K C. A barbiturate D. Acetaminophen E. Acetyl salicylic acid 9. a) b) c) d) e)

Which of the following drugs acts by inhibiting renal reabsorption of sodium? Urea Chlorothiazide Theophylline Digitalis glycosides Procainamide

10) All of the following are low molecular weight heparins (LMWH), except? A-Enoxaparin B-Dalteparin C-Tinzaparin D- Nadroparin E-Heparin 11) What is not true about low molecular weight heparins (LMWH)? A-It act by inhibiting factor Xa B-It is long acting than heparin C-It does not require monitoring D-It is contraindicated in pregnancy E-It is used as once daily SC injection 12) Unstable/NSTEMI drug of choice? A-Anticoagulants B-LMWH C-Heparin D-Alteplase E-Propranolol Copyright © 2000-2016 TIPS Inc. Unauthorized reproduction of this manual is prohibited. This manual is being used during review sessions conducted by PharmacyPrep.

86-2

PharmacyPrep.Com

Coronary Artery Diseases

13) What is the drug of choice for ST segment elevated myocardial infarction (STEMI)? A-Anticoagulants B-LMWH C-Heparin D-Alteplase E-Propranolol 14) What is the drug of choice for stable angina? A-Anticoagulants B-LMWH C-Heparin D-Alteplase E-beta blockers 15)Which of the following drug may cause bradycardia? A-Nifedipine B-Nitroglycerine C-Amlodipine D-Verapamil E-Clonidine 16) Which of the following drug should be taken with food? A-Fluvastatin B-Lovastatin C-Simvastatin D-Atorvastatin E-Rosuvastatin 17) What statin can be taken anytime of the day? A-Fluvastatin B-Lovastatin C-Simvastatin

D-Atorvastatin E-Rosuvastatin

18) What statin do not require dosage adjustment in renal disease patient? A-Fluvastatin B-Lovastatin C-Simvastatin D-Atorvastatin E-Rosuvastatin 19) Which of the following is NOT used for antiplatelets action? A-ASA B-Ibuprofen C-Naproxen D-Pyroxicam E-Acetaminophen 20) What antiplatelets drugs is used for prophylactic for angina pectoris? A-dipyridamol b-metoprolol c-nifedipine D-all of the above 21. Which of the following is NOT associated with plaque formation? A) Coronary artery disease B) Angina C) Stroke D) Reynaud's phenomenon E) intermittent claudication 22. Which of the following is recommended to minimize the warfarin dispensing error? I) dose of warfarin should be indicated in milligrams II) check the color of tablets Copyright © 2000-2016 TIPS Inc. Unauthorized reproduction of this manual is prohibited. This manual is being used during review sessions conducted by PharmacyPrep.

86-3

PharmacyPrep.Com

Coronary Artery Diseases

III) Recommend lifestyle modification 23. Exercise induced angina, which is relieved by rest nitroglycerine or both referred as? A) Prinzmetal angina B) stable angina C) unstable angina D) Myocardial infarction 24. Myocardial oxygen demand is increased by all the following except? A)exercise B) Beta blockers C) cold air D) smoking 25. A patient with angina receiving metoprolol and diltiazem should be monitored for? I) increased heart rate II) decreased cardiac output III) decreased heart rate A-I only B-III only C-I and II only D-II and III only E) I, II, III 26) A 35 year old craftsman is on nitroglycerin pills, which of the following statements should discuss with him: A) It causes an elevation of intracellular cGMP B) It can cause postural hypotension, take medication while sitting. C) Store medication away from light, and keep in tightly metal capped bottle. D. B&C Ans-D Tips: Nitroglycerin increases synthesis of intracellular cGMP. However, mechanism of action may not be discussed in counselling. 27) Which of the following is/are the ischemic heart diseases? A. Myocardial infarction B. Angina C. Acute coronary syndrome D. unstable angina E. All of the above Ans. E 28) Aspirin should be used cautiously in patient receiving heparin because aspirin: A. inhibit vitamin k absorption B. has antithrombin activity C. inhibit metabolism of heparin D. inhibit platelet aggregation Ans-D Tips: a drug that increases metabolism of the anticoagulant will lower the steady state plasma concentration both free and bound form, where as one that displaces anticoagulant will increase plasma level of free form only, until elimination of drug as again lower it to steady state levels.

Copyright © 2000-2016 TIPS Inc. Unauthorized reproduction of this manual is prohibited. This manual is being used during review sessions conducted by PharmacyPrep.

86-4

PharmacyPrep.Com

Coronary Artery Diseases

29) Nitroglycerin dilates the coronary arteries in angina pectoris by A. decreasing the heart rate reflex B. increasing the metabolic work of the myocardium C. direct action on smooth muscle in the vessel walls D. increasing the effective refractory period in the atrium Answer: C 30) In angina pectoris, the MOST rapid and dependable relief will be provided by A. oral glyceryl trinitrate B. oral isosorbide dinitrate C. oral erythrityl tetranitrate D. sublingual glyceryl trinitrate Ans: D 31) Nitroglycerin when taken together with Sildenafil will cause: A. Hypertension B. Hypotension C. Headache D. Hyperglycemia Ans: B 32) Aspirin acts as: A. irreversible antiplatelet drug B. reversible antiplatelet drug C. antiarrhythmic D. anticoagulant Ans: A 33) What are correct statement about Storage of Nitroglycerin: A. Amber color glass, hygroscopic B. Amber glass C. Plastic container D. Any container can be use Ans: A 34) What drug shouldn’t be given to post MI? A. ASA B. Ace inhibitors C. CCB and beta blockers D. All of the above Ans: D 35) Which drug causes bradycardia? A. Diltiazem Copyright © 2000-2016 TIPS Inc. Unauthorized reproduction of this manual is prohibited. This manual is being used during review sessions conducted by PharmacyPrep.

86-5

PharmacyPrep.Com

Coronary Artery Diseases

B. Nifedipine C. Nicardipine D. Felodipine E. Amlodipine Ans: A Tips: Diltiazem and Verapamil side effect is bradycardia 36) This drugs cause reflex tachycardia except: A. Amlodipine B. Nifedipine C. Nicardipine D. Felodipine Ans: A Tips: All dihydropyridine calcium channel blockers cause reflex tachycardia except amlodipine 37) Action of nitrates can be described as? A. vasodilation B. vasoconstriction C. bronchoconstriction D. Bronchodilatation Ans: A 38) An imbalance between oxygen supply and oxygen demand in cardiac muscle may produce a condition known as: A. CHF B. Heartburn C. MI D. Angina pectoris Ans: D 39) Drugs may be used in acute myocardial infarction: except A) Beta blocker B) ACE inhibitor C) Calcium channel blockers D) Nitrates E) Digoxin Ans: E 40) What dose does Aspirin acts as antiplatelet? A. 80 mg B. 365 mg C. 500 mg D. 1 g Ans: A Copyright © 2000-2016 TIPS Inc. Unauthorized reproduction of this manual is prohibited. This manual is being used during review sessions conducted by PharmacyPrep.

86-6

PharmacyPrep.Com

Coronary Artery Diseases

41) Drugs used in angina pectoris except: A. Nitrates B. CCB C. Cardiac glycoside D. Beta blockers Ans: C 42) What is the most common form of Ischemic heart disease: A. Myocardial infarction B. Atrial fibrillation C. Angina pectoris D. Ventricular arrhythmia Ans: C

43. An ambulatory patient, the most fastest acting nitroglycerine dosage form? A. SL spray B. SL tab C. wafers D. liquid E. suspension ans. a 44. What is the drug of choice to treat hypertension in angina? A. beta blockers B. CCBs D. ACEi E. ARBs Ans. A 45. What is true about echocardiogram (ultrasound)? A. Determines site and severity of thrombus or wall motion abnormalities. B. Measure heart rhythm C. Measure angina severity D. Measure biological marker E. Measure exercise tolerance Ans. A

Copyright © 2000-2016 TIPS Inc. Unauthorized reproduction of this manual is prohibited. This manual is being used during review sessions conducted by PharmacyPrep.

86-7

PharmacyPrep.Com

Coronary Artery Diseases

ANSWERS Coronary artery diseases 1. C 2- C 3. B Tips: TC page 275, avoid beta blockers in prinzmetal angina, beta blockers cause vasoconstriction (vasospasm) 4. D 5. C 6. E Tips: Verapamil act as coronary vasodilator by inhibiting calcium ion influx in coronary smooth muscles, and it does negative inotropic effects. 7- B 8- E 9. B 10) E(was D before) 11) D

Copyright © 2000-2016 TIPS Inc. Unauthorized reproduction of this manual is prohibited. This manual is being used during review sessions conducted by PharmacyPrep.

86-8

PharmacyPrep.Com

Coronary Artery Diseases

12. A 13) D 14) E 15) D 16) B Tips: Lovastatin should be always administered with food to increase Bioavailability, otherwise it can decrease 30% Bioavailability. 17) D Tips: Atorvastatin can be taken anytime of the day, because has long half life. 18) D Tips: The best agent for renal disease patient is atorvastatin, because it has minimal renal elimination, thus do not require dose adjustments. Simvastatin and lovastatin require dose adjustment in severe renal diseases however it can be used in minor and moderate renal diseases 19) E 20) A 21. D 22. C 23. B 24. B 25. B 26) Ans-D Tips: Nitroglycerin increases synthesis of intracellular cGMP. However, mechanism of action may not be discussed in counselling. 27) Ans-E 28) Ans-D Tips: a drug that increases metabolism of the anticoagulant will lower the steady state plasma concentration both free and bound form, where as one that displaces anticoagulant will Copyright © 2000-2016 TIPS Inc. Unauthorized reproduction of this manual is prohibited. This manual is being used during review sessions conducted by PharmacyPrep.

86-9

PharmacyPrep.Com

Coronary Artery Diseases

increase plasma level of free form only, until elimination of drug as again lower it to steady state levels. 29)Ans: C 30) Ans: D 31) Ans: B 32) Ans: A 33) Ans: A 34) Ans: D 35) Ans: A Tips: Diltiazem and Verapamil side effect is bradycardia 36)Ans: A Tips: All dihydropyridine calcium channel blockers cause reflex tachycardia except amlodipine 37) Ans: A 38) Ans: D 39) Ans: E 40) Ans: A 41) Ans: C 42) Ans: C

Copyright © 2000-2016 TIPS Inc. Unauthorized reproduction of this manual is prohibited. This manual is 86-10 being used during review sessions conducted by PharmacyPrep.

www.pharmacyprep.com PHARMACY PREP STROKE 1. The artery that supply blood to brain from heart? A. Internal carotid artery B. Coronary artery C. Suclavian artery D. Brachial artery E. Pulmonary artery Ans. A 1.A client presents with following symptoms, Which of the following symptoms is the most pharmacist concern, which require referral to emergency? A. patient face changes to uneven B. patient arm drifting down C. patient speech changes to strange D. severe unprovoked headache E. All of the above Ans. E 2) All of the following are initial symptoms of stroke include, Except? A-headache B-blurred vision C-paralysis D-seizure Ans. E

E-Chest pain

3) What is the drug of choice to treat transient ischemic attack (TIA) A. Acetyl salicylic acid B. Ticlopidine C. Warfarin D. Heparin E.Ramipril Ans. A 4) Which of the following factor that does NOT predispose stroke? A) Age B) Diabetes C) Ethnicity D) Hypertension E) chronic liver diseases Ans. E

87-1 Copyright © 2000-2016 TIPS Inc. Unauthorized reproduction of this manual is strictly prohibited and it is illegal to reproduce without permission. This manual is being used during review sessions conducted by PharmacyPrep.

PharmacyPrep.com

Congestive Heart Failure (CHF) PHARMACY PREP CONGESTIVE HEART FAILURE

1. What is the classical symptoms of CHF? A. Swollen legs B. Dyspnea, fatigue and weight gain C. Weakness, nausea and vomiting D. Chest pain radiating to left arm E. Weight loss Ans. b 2. What is the drug of choice to treat congestive heart failure? A. ACEi B. CCBs C. Diuretics D. Digoxin E. Nitroglycerine ans. A 3. Which of the following is least likely used to manage CHF? A. Beta blockers B. ACEi C. Losartan D. verapamil E. Carvedilol ans. d 4. What is the drug of choice for mild hypertension in patient with congestive heart failure? A. Ramipril B. Hydralazine C. Losartan D. Hydrochlorothiazide E. Furosemide ans. A

Copyright © 2000-2016 TIPS Inc. Unauthorized reproduction of this manual is prohibited. This manual is being used during review sessions conducted by PharmacyPrep.

88-1

PharmacyPrep.Com

Arrhythmias PHARMACY PREP DYSARRHYTHMIAS

1. Embolus (clot) enters into common carotid artery and travels toward the brain due to atrial fibrillation can increase risk?. A. angina B. Cardiogenic stroke C. myocardial infarction D. DVT E. CHF Ans.b 2. A distinct p-waves is seen in? A. Atrial fibrillation B. Atril flutter C. Ventricular tachycardia D. Sinus tachycardia E. all of the above Ans. A 3. Patient with atrial fibrillation. Which of the following is used to prevent risk of cardiogenic stroke. A. ASA B. Warfarin C. ACEi D. Beta blockers E. Calcium channel blockers Ans. B 4. What is not monitored in patient using amiodarone? A. Serum TSH B. Chest x-ray C. Eye exam D. Renal function test E. Liver function test Ans. d 5.What is the most important to monitor in patient using amiodarone? A. Serum TSH B. Chest x-ray C. Eye exam D. Renal fucntion test E. Liver function test Ans.A 6. What type arrhythmias digoxin is contraindicated? A. Atrial fibrillation Copyright © 2000-2016 TIPS Inc. Unauthorized reproduction of this manual is prohibited. This manual is being used during review sessions conducted by PharmacyPrep.

89-1

PharmacyPrep.Com

Arrhythmias

B.Atril flutter C. Ventricular tachycardia D. Sinus tachycardia E. all of the above Ans. C

8)Atrial fibrillation. Which of following is the characteristic wave? A. P wave B. QRS wave C. QT wave D. T wave E. U wave Ans. A

2 . Which of the following dysrrhythmia associated with sudden high fever? A. Atrialfibrillation B. Proxysmal C. Ventricular D. Atrial E. Premature Ans. B

Copyright © 2000-2016 TIPS Inc. Unauthorized reproduction of this manual is prohibited. This manual is being used during review sessions conducted by PharmacyPrep.

89-2

PharmacyPrep.Com

Arrhythmias

Copyright © 2000-2016 TIPS Inc. Unauthorized reproduction of this manual is prohibited. This manual is being used during review sessions conducted by PharmacyPrep.

89-3

PharmacyPrep.Com

Peripheral Vascular Diseases PHARMACY PREP PERIPHERAL VASCULAR DISEASES

1) Aneurysm is ? A) bulging of arteries B) a blood clot c) a moving blot clot d)permanent enlargement of alveoli E) None of the above 2) Thrombus is? A) bulging of arteries B) a blood clot c) a moving blot clot d)permanent enlargement of alveoli E) None of the above 3) Embolus is ? A) bulging of arteries B) a blood clot c) a moving blot clot d)permanent enlargement of alveoli E) None of the above 4) Symptoms of Reynaud phenomenon? a) fingers spasm b) extremities turn to pale, or blue C)often in elderly D)all of the above 5) Symptoms of Intermittent claudication? a) cold feet, B)pulses are absent, c) exercise induced cramping pain d)all of the above 6) Symptoms of Stroke? A)headache B)confusion C)dizziness D)blurred vision E)all of the above 7) What are the symptoms of deep vein thrombosis? A) Leg pain B) swollen legs C) tenderness

D) Calf pain

E) all of the above

8) What is treatment of deep vein thrombosis? A) warfarin, b) heparin, c) LMWH D) thrombolytic E)All of the above 9) Drug of choice against the prophylaxis of transient ischemic attack? A)ASA 81 mg chewable B)ASA 81 mg enteric coated C)Alteplase injection D)Warfarin E)heparin 10. Which of the following monitoring parameter is essential for patient who currently taking Low molecular weight heparin: A-International normalized ratio (INR) B-Prothrombin time C-Heparin assay D-Sensitive TSH E-provide predictable response, thus no monitoring required

Copyright © 2000-2016 TIPS Inc. Unauthorized reproduction of this manual is prohibited. This manual is being used during review sessions conducted by PharmacyPrep.

90-1

PharmacyPrep.Com

Peripheral Vascular Diseases

11. Which of the following are the monitoring parameters of warfarin therapy? I Complete blood count (CBC) II PT (prothrombin time) III INR (international normalization ratio) a. I only b. III only c. I and II d. II and III e. I, II, III 12-Which of the following drugs increase international normalized ratio (INR)? A-Clotrimazole B-Clindamycin C-Azithromycin D-Penicillin's 13) Which of the of the following can be used in a pregnant women with deep vein thrombosis ? A-Captopril B-Warfarin C-Atorvastatin D-Heparin E-Rosuvastatin 14) Which of the following is NOT a low molecular weight heparins (LMWH)? A-Dalteparin B-Heparin C-Nadoparin D-Enoxaparin E-Tinazeparin 15) What is incorrect about Reynaud's phenomenon? A- Reynaud's phenomenon in which arteries of fingers become spastic (vasospastic). B-Reynaud's phenomenon may result from atherosclerosis, connective tissue disease, ingestion of ergot alkaloids, or frequent use of vibrating tools. C- Reynaud's phenomenon may result from peripheral artriel plaques formation D) Reynaud's phenomenon symptoms fingers spasm, extremities turn to pale, or blue. E- Reynaud's phenomenon non-pharmacological is minimizing cold exposure, and use warm gloves 16)Which of the following the most important drug is used to prevent peripheral vascular diseases? A)Beta blocker B)ACEI C) warfarin D)digoxin E)ASA 17) Which of the following the most important drug is used to treat peripheral vascular diseases? A)Beta blocker B)ACEI C) warfarin D)digoxin E)ASA 18) Which of the following drug is least likely used to treat peripheral vascular diseases? A)Beta blocker B)ACEi C) warfarin D)digoxin E) ASA 19) Which of the following is incorrect statement? A)Reynaud phenomenon symptoms occurs in limb extremities B)Calcium channel blockers are the drug of choice C)Arterial plaques are the risk factor for Raynaud phenomenon Copyright © 2000-2016 TIPS Inc. Unauthorized reproduction of this manual is prohibited. This manual is being used during review sessions conducted by PharmacyPrep.

90-2

PharmacyPrep.Com

Peripheral Vascular Diseases

D)Reynaud phenomenon can be trigger cold exposure E)Reynaud phenomenon is may occur in old age 20) Which of the following drugs increase risk of blood clots in peripheral vascular system? A) Enoxaparin B) Alteplase C) combined oral contraceptive pills D. Warfarin E. NSAIDs 21) Which of the following drugs least likely used in DVT patients? A) warfarin B) Dabigatran C) LMWH D) heparin E) cardio selective beta blockers 22) All are risk factors of deep vein thrombosis (DVT), except? A) pregnancy B) cancer and chemotherapy C) Progestin only oral contraceptive pills D) diabetes E) mobility 23) A 64 yo man weighing 115 kg, and tall 60 inches, presents to his physician after long international flight complaining of pain, swelling of his right lower leg. The patient had knee surgery a month before he travelled. His medication profile include, CHF (ejection fraction < 15%), remission from lymphoma, MI. His father, mother, and sister are all diseased from stroke, pulmonary embolism. Given his profile. Patient most likely suffering from? A) deep vein thrombosis B) coronary artery disease C) congestive heart failure D) diabetes 24) Which of the following is NOT risk factors of deep vein thrombosis (DVT)? A) Knee surgery B) over weight C) long flight D) family history E) gender 25. What is the drug of choice to treat DVT in pregnancy? A. Dalteparin B. Heparin C. warfarin D. Dabigatran E. apixaban Ans. A 26. All of the following conditions are associated with atherosclerosis plaque, except? A. DVT B. Stable angina C. MI D. Stroke E. Raynaud's phenomenon Ans. E Copyright © 2000-2016 TIPS Inc. Unauthorized reproduction of this manual is prohibited. This manual is being used during review sessions conducted by PharmacyPrep.

90-3

PharmacyPrep.Com

Peripheral Vascular Diseases CORRECT ANSWERS Peripheral Vascular Diseases

1)Ans: A 2) Ans: B 3) Ans: C 4) Ans: D 5) Ans: D 6) Ans: E 7) Ans: D 8) Ans: E 9) Ans: A 10. Ans: E Tips: INR and PT is monitored for warfarin, aPTT for heparin, where as for LMWH these tests are not effective 11. Ans: D 12- Ans: A 13) Ans: D 14) Ans: B 15) Ans: C 16) Ans: C Tips: Anticoagulants like warfarin is commonly used to treat peripheral embolic disorders. 17) Ans: C 18) Ans: A Tips: beta blocker cause vascular constriction thus it is contraindicated in peripheral vascular diseases

Copyright © 2000-2016 TIPS Inc. Unauthorized reproduction of this manual is prohibited. This manual is being used during review sessions conducted by PharmacyPrep.

90-4

PharmacyPrep.Com

Peripheral Vascular Diseases

19) C Ans: 20) Ans: C Tips. Combined oral contraceptive pills are combination of estrogen and progestin increase platelet aggregation and thus increase risk of blood clot formation. 21) Ans: E 22) Ans: E Tips: mobility or physical activity lower the risk of deep vein thrombosis 23) Ans: A 24) Ans: E Tips: RF for above patient, except gender. other RFs include age > 40 yr, cancer, CHF, MI, and obesity.

Copyright © 2000-2016 TIPS Inc. Unauthorized reproduction of this manual is prohibited. This manual is being used during review sessions conducted by PharmacyPrep.

90-5

PharmacyPrep.Com

Peripheral Vascular Diseases

Copyright © 2000-2016 TIPS Inc. Unauthorized reproduction of this manual is prohibited. This manual is being used during review sessions conducted by PharmacyPrep.

90-6

Pharmacy Prep ANTICOAGULANTS 1-Which of the following clotting factor is NOT inhibited by warfarin? A-2 B-7 C-9 D-10 E-12 2. A drug is said to reach "steady state" when 5 half-lives have elapsed. In the case of warfarin (a blood thinner), however, the steady state is dependent not upon the drug's half-life, but upon the half-life of the clotting factors it inhibits. The clotting factors inhibited by warfarin have the following half-lives: Factor II - 50 hours Factor VII - 6 hours Factor IX - 24 hours Factor X - 36 hours Based on this information, how long will it take a patient to reach maximal anticoagulation? A- 5 days B-10 days C- 15 days D- 20 days E-48 hours 3-The hematocrit (HCT) measures the A. Total number of blood cells per volume of blood B. Weight of red blood cells per volume of blood C. Number of red blood cells per volume of blood D. Weight of hemoglobin per volume of blood E. Percentage of red blood cells per volume of blood 4) A patient is using injections of Dalteparin to treat deep vein thrombosis what is monitored? A) aPTT B) antifactor Xa C) Prothrombin time D) international normalized ratio E) Heparin assay 5) Which of the following oral anticoagulant approved to treat atrial fibrillation? A) heparin B) dalteparin C)Tinzaparin D)Dabigatran E)all of the above 6) What is incorrect statement? A) LMWH therapy are effective in preventing DVT B) Heparin therapy require close monitoring than LMWH C) Heparin is weight based intravenous therapy is used in acute DVT D) Warfarin produce rapid anticoagulation than LMWH E) Long term heparin therapy can cause osteoporosis side effect 7) What is incorrect statement? A) LMWH have longer duration of action than heparin B) LMWH once daily SC administration C) LMWH do not require routine monitoring D) Heparin has rapid anticoagulant effect E) aPTT reflects changes in extrinsic pathway of the coagulation cascade Copyright © 2000-2016 TIPS Inc. Unauthorized reproduction of this manual is prohibited. This manual is being used during review sessions conducted by PharmacyPrep.

91-1

8) Patient is hospitalized. Doctors initiated heparin iv infusion. What is incorrect about heparin A) heparin dose is based on body weight B) loading dose is 80 units/kg, and maintenance dose is 15 to 25 units/kg/hr. C) Heparin is slower onset than LMWH D) LMWH have longer half life E) LMWH have predictable response thus not monitored 9) Over dose of heparin is treated by? I) Stop heparin infusion II) Protamine sulfate III) oral vitamin K A. I only B. III only C. I and II D. II and III E. I, II, III 10) Patient is receiving warfarin after discharged from hospital. He is receiving cotrimoxazole to treat UTI. One may expect to see the INR (2 - 3)? I) Increase II) No change III) Decrease A. I only B. III only C. I and II D. II and III E. I, II, III 11. What anticoagulant is used in emergency as iv dosage form? A) Heparin B) Low molecular weight heparins (LMWH) C) Warfarin D) Dabigatran E. Apixaban 12. The mechanism of action of Acetylsalicylic acid that results in greater effect as an antiplatelet drug is because of: A. Selective COX I inhibitor B. Selective COX II inhibitor C. COX I and COX II inhibitor D. Acetylate cyclooxygenase 13) Which of the following anticoagulant available as iv and sc injections A. Heparin B. LMWH C. Warfarin Copyright © 2000-2016 TIPS Inc. Unauthorized reproduction of this manual is prohibited. This manual is being used during review sessions conducted by PharmacyPrep.

91-2

D. Apixaban E. Dabigatran 14) What should be monitored in heparin treatment? A. INR (International Normalised Ratio) B. Vitamin K C. aPTT D. Serum levels of Heparin E. Prothrombin Time 15) Which of the following tests are used for antithrombotic therapy: A. INR, Prothrombin time and aPTT B. aPTT and Prothrombin Time C. Heparin assay and a PTT D.INR only E. a PTT only 16) Which of the following is the drug of choice to treat deep vein thrombosis (DVT): A) Warfarin B. streptokinase C. Heparin D. LMWH E. Rivaroxaban 17) What anticoagulants is safe in renal disease patients? A. warfarin B. dalteparin C. Heparin D. dabigatran E. apixaban 18) In order to measure effectiveness of warfarin therapy, the following monitoring tests are used: A. INR, must be 2-3 B. Prothrombin test (PT) C. aPTT D. A&B 19) At what dose Aspirin has anticoagulant property A. 80 mg B. 325mg C. 500 mg D. 800 mg

Copyright © 2000-2016 TIPS Inc. Unauthorized reproduction of this manual is prohibited. This manual is being used during review sessions conducted by PharmacyPrep.

91-3

20) Effects of warfarin include all except: A. Decreasing the formation of blood clot B. Nose bleeds C. Red-orange urine D. Ringing in the ears 21) All drugs will decrease INR of warfarin except: A. Rifampin B. Barbiturate C. Green leafy vegetable D. Metronidazole 22) Warfarin site of action is: A. In vivo B. In vitro C. In vivo and in vitro D. Neither 23) Which drug is not a low molecular weight heparin (LMWH)? A. Enoxaparin B. Nadroparin C. Heparin D. Dalteparin 24) What is the safest anticoagulant in pregnancy? A. Dalteparin B. Warfarin C. Heparinoids D. Heparin E. Rivaroxaban 25) Antidote for heparin toxicity: A. Vitamin K B. Protamine sulphate C. Physostigmine D. Atropine 26) All of the following increases INR except: A. ASA and NSAID B. Chronic alcohol C. Cotrimoxazole D. Acute alcohol

Copyright © 2000-2016 TIPS Inc. Unauthorized reproduction of this manual is prohibited. This manual is being used during review sessions conducted by PharmacyPrep.

91-4

27) Which of the following vitamins is needed for proper blood clotting? A. Vitamin A B. Vitamin D C. Vitamin E D. Vitamin K 28) Which of the following OTC drugs is contraindicated for a patient taking warfarin? A. Diphenhydramine B. Loratadine C. Psyllium D. ASA 29) All of the following drugs should be carefully prescribed with aspirin EXCEPT: A. Enoxaparin B. Coumadin C. Heparin D. Metoclopramide 30) An overdose of warfarin can be treated by administering: A. Vitamin K B. Acetylcysteine C. MESNA D. Protamine sulfate E. Sodium bicarbonate 31. What anticoagulants is safe in renal disease patients? A. warfarin B. dalteparin C. Heparin D. dabigatran E. apixaban Ans. A 32. What is rat poison? A. Coumarin B. warfarin C. Heparin D. LMWH E. Dabigatran Ans. A 33. What is incorrect about anticoagulants? A. warfarin is preferable because antidote is readily available B. warfarin is less preferable after surgeries because of increase risk of bleeding C. warfarin require frequent monitoring Copyright © 2000-2016 TIPS Inc. Unauthorized reproduction of this manual is prohibited. This manual is being used during review sessions conducted by PharmacyPrep.

91-5

D. warfarin is not available in liquid or injections E. warfarin require dose adjustment in renal disease patients. Ans. e 34. what anticoagulant is safe in pregnancy with DVT? A. warfarin B. dalteparin C. Fondaparinux D. dabigatran E. apixaban Ans. B

Copyright © 2000-2016 TIPS Inc. Unauthorized reproduction of this manual is prohibited. This manual is being used during review sessions conducted by PharmacyPrep.

91-6

ANTICOAGULANTS ANSWERS: 1- Ans: E 2. Ans: B Tips: “maximal” anticoagulation will be reached when all clotting factors are inhibited. This will take approximately 10 days, since 50hrs (the half-life of the clotting factor taking the longest time to reach steady) x 5 = 250 hours, which is 10.4 days. 3- Ans: E Tips: Whole blood treated with anticoagulant is centrifuged in a calibrated hematocrit tube. The volume ratio of the packed red blood cells to total blood volume is determined. The hematocrit is normally 39 to 49 for men and 33 to 43 for women. The hematocrit value provides some indication of both the number and size of the red blood cells present in an individual 4) Ans: B 5) Ans: D Tips: Dabigatran an oral anticoagulants recently approved to treat atrial fibrillations. It is factor Xa inhibitor. (competitive direct thrombin inhibitor). 6) Ans: D 7) Ans: E 8) Ans: C Tips: Heparin iv infusion is faster onset than LMWH. 9) Ans: E 10) Ans: A Tips: antibiotics potentiate anticoagulant effect : cotrimoxazole, erythromycin, metronidazole, (other drugs: omeprazole, fluconazole, ketoconazole, cimetidine, amiodarone, quinidine, vitamin E (large doses), acute phenytoin, thyroxin, acute alcohol also potentiate. Reduce the clearance of warfarin, displacement from warfarin protein binding, decrease intestinal flora Drugs that inhibit anticoagulant effect: carbamazepine, penicillin, rifampin, cholestyramine, barbiturates, high vitamin K, chronic alcohol use 11. Ans-A

Copyright © 2000-2016 TIPS Inc. Unauthorized reproduction of this manual is prohibited. This manual is being used during review sessions conducted by PharmacyPrep.

91-7

12. Ans: D Tips: Aspirin is the only drug that irreversibly inhibits the enzyme. This action permits a longer duration of action causing platelet aggregation and acetylation of cyclooxygenase. 13) Ans: A 14) Ans-C 15) Ans-A 16) Ans-D Tips: LMWH is the drug of choice to treat DVT. Alternatively heparin is in emergency. The oral anticoagulants are used for maintenance.

17) Ans. A 18) Ans-D 19) Ans-A 20) Ans: D Tips: Ringing in the ear is associated with ASA, salicylates, quinine, and quinidine, but not warfarin. 21) Ans: D Tips: Metronidazole increases INR 22) Ans: A Tips: Warfarin site of action is in vivo and heparin is either in vivo or in vitro 23) Ans: C 24) Ans: D 25) Ans: B 26) Ans: B Tips: Chronic alcohol decreases INR while acute alcohol increases INR. 27) Ans: D 28) Ans: D

Copyright © 2000-2016 TIPS Inc. Unauthorized reproduction of this manual is prohibited. This manual is being used during review sessions conducted by PharmacyPrep.

91-8

29) Ans: D Tips: Aspirin is a blood thinning agent indicated for prevention of heart stroke. It should be carefully prescribed with other blood thinning agents because of risks of bleeding. Lovenox (Enoxaparin), Coumadin (Warfarin), Heparin, Plavix (Clopidrogel), Ticlid (Ticlopidine), Depakene (Valproic acid), Persantine (Dipyridamole), Mandol (Cefamandole), Cefotan (Cefotetan), Cefobid (Cefoperazone) and Moxam (Moxalactam) are agents that increase bleeding tendency in patients and should be carefully prescribed with other blood thinning agents. 30) Ans: A Tips: The overdose of Coumadin can be treated by administration of Vitamin K1 (Mephyton).

Copyright © 2000-2016 TIPS Inc. Unauthorized reproduction of this manual is prohibited. This manual is being used during review sessions conducted by PharmacyPrep.

91-9

PharmacyPrep.Com

Anxiety Disorders PHARMACY PREP ANXIETY DISORDERS

1. What is the drug of choice for panic attacks? A. amitriptyline B. propranolol C. SSRI D. benzodiazepine Ans. C Tips: benzodiazepine for short term use only if needed, then the drug therapy is SSRI and alternate TCA with cognitive behavior therapy.

Copyright © 2000-2016 TIPS Inc. Unauthorized reproduction of this manual is prohibited. This manual is being used during review sessions conducted by PharmacyPrep.

92-1

PharmacyPrep.com

Depression PHARMACY PREP DEPRESSION

1. Duration of therapy for first episode of major depression is? A. 3 mo B. 6 wks C. 1 year D. 2 to 3 yrs E. Lifelong Ans. c 2. What antidepressants have sweating side effects? a. SSRIs B. SNRIs C. TCA D. MAOi Ans. A

E. NDRIs

Tips:

Antidepressants Mirtazapine Bupropion Trazadone Fluvoxamine Sertraline Fluoxetine Moclobamide

Highs Side effects Weight gain

Lows side effects Nausea & vomiting Sexual dys. & weight gain Insomnia

Sedation Drug interaction

Drug interaction. Weight gain Sexual dysfunction

Wash out period DIs

3. What is the onset of SSRIs? A. 1 to 4 days B. 1 to 4 wks Ans. B Antidepressants Onset SSRI 1 to 4 wks safe TCA s MAOi s SNRI 1 to 4 wks safe Lithium s a

C. 1 to 4 months

D. 1 to 4 years

Tips

Early toxic: N&V, D Late: cerebellar ataxis, mental confusion

4 A patient with major depression, doctor initiated fluoxetine. However patient did not tolerated and there was no improvement. Doctor initiated phenelzine, a week later. After patient started therapy for phenelzine, 4 days later the patient presented with fever, high blood pressure, diarrhea, and confusion. What is the most likely cause? A) Neuroleptic malignant syndrome B) Hypertension crisis C) Serotonin syndrome D) Serotonin withdrawal syndrome Ans: C Copyright © 2000-2016 TIPS Inc. Unauthorized reproduction of this manual is prohibited. This manual is being used during review sessions conducted by PharmacyPrep.

93-1

PharmacyPrep.com

Depression

Tips: Serotonin symptoms Autonomic Diarrhea, Fever, Shivering, change in BP, and & N, V Neuromuscular Tremors, seizure, myoclonus, Dysfunction hyper reflexes Cognitive Agitation, Confusion, hypomania

Withdrawal symptoms Flu like fever, chills, dizziness, light headed, N &V, sleep disturbances Myalgia, lethargy, sensory disturbances Not present

5. What serum level lithium give toxicity symptoms? A) Lithium serum levels > 2 mEq/L B) Lithium serum levels < 1.5 mEq/L C) Lithium serum levels < 1 mEq/L D) Lithium serum levels 2mEq/L can give lithium toxicity. Initial Onset SEs N&V, GI upset, Dry mouth, Fine hand tremor, sedation Muscle weakness Polydypsia Polyuria Nocturia

Lithium Long term SEs Fine hand tremors Weight gain Hypothyroidism acne, psoriasis rash, alopecia Decreased libido Arrhythmia: Non specific T-wave changes, Nephrotoxicity Nephrogenic Monitor: Li, Na, Serum TSH, CBC, weight, CrCl (No LFT)

Lithium Toxicity Coarse hand tremor Muscle twitching Vomiting, severe drowsiness, confusion, nystagmus, seizure, Ataxia, cogwheel rigidity, coma, death

Copyright © 2000-2016 TIPS Inc. Unauthorized reproduction of this manual is prohibited. This manual is being used during review sessions conducted by PharmacyPrep.

93-2

PharmacyPrep.Com

Antipsychotics PHARMACY PREP ANTIPSYCHOTICS

1. Drug of choice to treat negative and positive symptoms? A. haloperidol B. Olanzapine C. Fluoxetine D. Metoclopramide E. Chlopromazine Ans. B 1. A patient receiving haloperidol to treat schizophrenia. Presents with hyperthermia (fever), sweating, tachycardia, muscle rigidity. Which of the following most likely cause? A) Neuroleptic malignant syndrome B) Serotonin syndrome C) withdrawal symptoms D) Extra pyramidal symptoms Ans: A Tips: NMS develops quickly and symptoms stay for 5 to 10 days. Require referral to emergence. Extrapyramidal symptoms (EPS) Parkinson's (TRAP) Akathisia Dystonia Tardive dyskinesia

Neuroleptic malignant symptoms (NMS) Fever Rigidity tachycardia Fluctuation BP WBC, CK

Copyright © 2000-2016 TIPS Inc. Unauthorized reproduction of this manual is strictly prohibited and it is illegal to reproduce without permission. This manual is being used during review sessions conducted by PharmacyPrep.

1

PharmacyPrep,Com

Dementia PHARMACY PREP DEMENTIA

1) Alzheimer is a progressive disease characterized by relentless and loss of mental function due to lack of: A. Thiamine B. Dopamine C. Acetylcholine D. Norepinephrine E. Epinephrine 2. All of the following are symptoms of Alzheimer’s dementia, EXCEPT: A. Slurred speech B. Reduced cognitive functions C. Fatigue D. Reduced sexual function E. Incoherent speech 3) All of the following are symptoms of Alzheimer's dementia, except? A) loosing items B) impaired perception of time C) Hallucinations and dilutions D) Depression E) able to execute functions 4. Risk factors of Alzheimer’s dementia A. High cholesterol B. Smoking C. Use of alcohol D. Obesity E. All of the above 5) All of the following are risk factors of Alzheimer's dementia, except? A) Age B) Family history C) race D) coronary artery diseases E) sedentary life style 6) What is incorrect about Alzheimer’s dementia? A)A e er s e en a cc rs e c B)the drug of choice against Alzheimer's dementia is donepezil C)Delirium is acute agitation, lack of judgement and thinking. D)Amnesia is short loss of memory E) Old age is NOT a risk factors for Alzheimer's dementia

Copyright © 2000-2016 TIPS Inc. Unauthorized reproduction of this manual is prohibited. This manual is being used during review sessions conducted by PharmacyPrep.

95-1

PharmacyPrep,Com

Dementia

7. What is the drug of choice for the treatment of Alzheimer's dementia ? A. Galantamine B. Donepezil C. Rivastigmine D. Neostigmine E. Memantine 8. Which of the following is classified as non selective reversible acetyl cholinesterase inhibitor? A. Galantamine B. Donepezil C. Rivastigmine D. Memantine E. Neostigmine 9) What is/are the common side effects of Donepezil, Rivastigmine, and Galantamine? A. Confusion B. Sweating C. Diarrhea D. Cholinergic agonist effects E. All of the above 10) Drug that increase effectiveness of Donepezil: A. Carbamazepine B. Erythromycin C. Phenytoin D. All of the above 11) What drugs is/are not a treatment for Alzheimer's? A. Rivastigmine B. Donepezil C. Memantine D. Gingko biloba E. Galantamine 12) Which herbal product is used to treat Alzheimer and dementia? A. Gingko biloba B. Vitamin D C. St. John Wort D. Omega 3 E. Licorice 13)What is incorrect statement? A) donepazil is selective acetylcholinesterase inhibitor Copyright © 2000-2016 TIPS Inc. Unauthorized reproduction of this manual is prohibited. This manual is being used during review sessions conducted by PharmacyPrep.

95-2

PharmacyPrep,Com

Dementia

B) Rivastigmine is a acetylcholinesterase and butyryl cholinesterase inhibitor C) Galantamine is acetylcholinesterase inhibitor and also effect on acetyl choline by binding with nicotinic receptors D) Memantine is NMDA receptor antagonist E) None of the above 14.What is the drug of choice to treat levy body dementia? A. Rivastigmine B. Donepezil C. Memantine D. Gingko biloba E. Galantamine Ans. A

Copyright © 2000-2016 TIPS Inc. Unauthorized reproduction of this manual is prohibited. This manual is being used during review sessions conducted by PharmacyPrep.

95-3

PharmacyPrep,Com

Dementia

CORRECT ANSWERS DEMENTIA

1) Ans: C Tips: deficiency of acetylcholine and damage of brain cells in older age causes dementia. 2) C 3) E 4) E Tips: Lifestyle factors like alcohol intake have no evidence worsening Alzheimer's. 5) C 6) E Tips: old age is a risk factor for Alzheimer's dementia 7) Ans: B 8) Ans: A 9) Ans: E Tips.Dementia is treated by acetylcholinesterase inhibitors i.e. cholinergic agonist, thus produce cholinergic agonist side effects " DUMBLESS"

10) Ans: A Tips: Drugs that increases Donepezil are inhibitors of CYP 2D6 or CYP 3A4. Drugs that decreases Donepezil are inducers of CYP 2D6 or CYP 3A4. 11) Ans: C 12) Ans: A 13) Ans: E

Copyright © 2000-2016 TIPS Inc. Unauthorized reproduction of this manual is prohibited. This manual is being used during review sessions conducted by PharmacyPrep.

95-4

PharmacyPrep.com

Seizure and antiepileptic drugs PHARMACY PREP SEIZURE & ANTIEPILEPTICS

1) What type of seizure symptoms in-patient is NOT unconscious? A) Simple partial B) Complex partial C) Generalized seizures D) Petit mal seizures E) Absence seizures 2) Phenytoin is indicated the treatment of all of the following types of seizure EXCEPT: A-Grand mal B-Complex partial C-Simple partial D-Tonic clonic E) Absence seizure (petit mal) 3)Which of the following drug is NOT used in absence seizure patient? A-Phenytoin B-Valproic acid C-Diazepam D-Ethosuximide E-All of the above 4) Which of the following drug least likely to effect on oral contraceptives: A-Tetracycline B-Phenytoin C-Carbamazepine D-Gabapentin E-Phenobarbital 5) Which of the following is the drug of choice in status epilepticus? A-phenytoin iv B-Carbamazepine iv C-Diazepam iv D-Infliximab E-Valproic acid 6) Which of the following anticonvulsive drug treatment most likely cause kidney stone formation. A-Carbamazepine Copyright © 2000-2016 TIPS Inc. Unauthorized reproduction of this manual is strictly prohibited and it is illegal to reproduce without permission. This manual is being used during review sessions conducted by PharmacyPrep.

96-1

PharmacyPrep.com

Seizure and antiepileptic drugs

B-Phenytoin C-Tiagabine D-Topiramate E-Phenobarbital 7)The most common side effects of anti seizure drugs: I-Appetite and body weight changes II-GI symptoms III-Head ache and dizziness A-I only B-III only C-I and II D-II and III

E-I, II, III

8) Which of the following antiepileptic drugs has weight loss side effects: A-Valproic acid B-valproate C-Phenytoin D-Topiramate E-Carbamazepine 9) A 25-year-old female patient taking gabapentin for neuropathic pain, which of the following statement is/are true: I-She should avoid taking oral contraceptive because, this drug decrease effectiveness of gabapentin II-Gabapentin chemical structure is not similar to GABA III-Gabapentin chemical structure contains GABA structure. A-I only B-III only C-I and II only D-II and III only E-All of the above 10) Which of the antiseizure drug has weight loss side effects. A-Topiramate B-Carbamazepine C-Phenytoin D-Valproic acid E-Gabapentin 11) A 25-year-old female patient taking gabapentin for neuropathic pain, which of the following statement is not true: A-She should avoid taking oral contraceptive because, this drug decrease effectiveness of gabapentin B-Gabapentin have advantage of the least drug interactions Copyright © 2000-2016 TIPS Inc. Unauthorized reproduction of this manual is strictly prohibited and it is illegal to reproduce without permission. This manual is being used during review sessions conducted by PharmacyPrep.

96-2

PharmacyPrep.com

Seizure and antiepileptic drugs

C-Gabapentin chemical structure contains GABA structure D-Gabapentin can be used seizures and neuralgia E-Gabapentin is GABA analog 12) What is incorrect about phenytoin? A-Phenytoin available as suspension, iv, chewable tablets and capsule. B-Always recommend chlorhexidine to treat deficiency of folic acid C-Phenytoin blood levels should be monitored to 10 to 20 mcg/ml D-Phenytoin have saturated kinetics, thus overdose can give toxicity E-Chronic use of phenytoin can cause megaloblastic anemia 13) Symptoms of Steven Johnson’s Syndrome, include all except A-Rash B-Sores on mucus membrane, C-Blistering mucus membrane, typically mouth, eyes and vagina. D- Patchy areas rashes. E-Butterfly rash 14-What is drug of choice in petit-mal seizure? A-Diazepam B-Phenytoin C-Carbamazepine D-Valproic acid and ethosuximide E-Gabapentin 15-Valproic acid is used in treatment of a) Petit-mal b) Grand-mal c) Febrile seizure in children d) All of the above e) A and C only 16-Anticonvulsive agent that cause weight loss? A-Topiramate B-Phenytoin C-Carbamazepine

D-Valproic acid E-Gabapentin

17-Phenytoin causes elevation of blood glucose levels by? A-Increasing insulin degradation B-Decreasing insulin release C-Increasing glucose absorption D-increasing glycogen release 18-Side effects of phenytoin are? Copyright © 2000-2016 TIPS Inc. Unauthorized reproduction of this manual is strictly prohibited and it is illegal to reproduce without permission. This manual is being used during review sessions conducted by PharmacyPrep.

96-3

PharmacyPrep.com

Seizure and antiepileptic drugs

A-Gingival hyperplasia B-Steven Johnson syndrome C-Hirsutism D-All of the above E-A and C 19-Epilepsy cause due to deficiency of? A-Sodium ion B-GABA C-Serotonin

D-Dopamine E-Acetylcholine

20-Early symptoms of epilepsy are? A-Part of brain is involved B-Sensory seizure or focal motor seizure C-Patient is conscious D-All of the above 21-Which one of the following CNS receptors is directly coupled to an ion channels, and this do not involve secondary messenger system? A-Alpha (NE) B-dopamine D 2 (DA) C-Acetyl choline (N) D- serotonin (5HT) E- None of the above 22-What are antimuscarinic drugs that are used in treatment of antipsychotic induced Parkinson’s disease, A-Benztropine B-Trihexphenidyl C-Bromocriptine D-Diphenhydramine E-Procyclidine 23-What type of local anesthetic is the most effective? A-The most hydrophilic B-The most lipophylic D-The least lipophylic

C-The least hydrophilic

24-Metabolism of local anesthetic takes place in? A-Skin B-Tissue C-Liver D-Gut E-Local area 25) All of the following are GABA analogs except? A-Gabapentin B-Pregabalin C-Vigabatrin D-Beclofen

E-Phenytoin

26) Tachycardia is the side effect of? Copyright © 2000-2016 TIPS Inc. Unauthorized reproduction of this manual is strictly prohibited and it is illegal to reproduce without permission. This manual is being used during review sessions conducted by PharmacyPrep.

96-4

PharmacyPrep.com

Seizure and antiepileptic drugs

A-Anticholinergic B-Sympathomimetics C-Sympathetic blockers D-Non dihydropyridine type of CCBs E-All of the above 27) Treatment of epilepsy for? A-Inhibiting GABA neurotransmitter B-Stimulating NMDA neurotransmitter (N-methyl D-Aspartate) C-Decrease firing threshold impulse D-Stimulating sodium ion channels E-Stimulating potassium ion channels 28) Valproic acid is used to treat? I-Petit-mal (absent of seizure) II-Grand-mal (general seizure) III-Febrile seizure in children A-I only B-III only C-I and II only D-II and III only E-I, II,III 29. When changing from phenytoin syrup to phenytoin capsule, at same dose, change in effect is due to I. Change in salt form II. Change in dose III. Change in dose elimination A-I only B-III only C-I and II only D-II and III only E-I, II,III 30) What is drug of choice for petit mal (absence) seizure? A-Diazepam iv B-Diazepam oral C. Ethosuximide and valproic acid D-Phenytoin E-Carbamazepine 31) Side effects of phenytoin include? I-Gingival hyperplasia, ataxia, nystagmus II-Steven Johnsons syndrome III-Hirsutism and acne A-I only B-III only C-I and II only

D-II and III only

E-I, II,III

32) Epilepsy is resulted from the deficiency of?--> GABA action; Early symptoms of partial seizures are? I-Part of the brain II-Sensory seizure or focal motor seizure III-Patient is conscious Copyright © 2000-2016 TIPS Inc. Unauthorized reproduction of this manual is strictly prohibited and it is illegal to reproduce without permission. This manual is being used during review sessions conducted by PharmacyPrep.

96-5

PharmacyPrep.com A-I only

B-III only

Seizure and antiepileptic drugs C-I and II only

D-II and III only

E-I, II,III

33)What are the common SEs of antiepileptic drugs? A) Stevens Johnson Syndrome B) Kidney stone formation C)Head ache, dizziness, GI, appetite, weight gain, D) Weight loss, euphoria 34) A 25 yo women with one 2 yr old child. She has been using antiepileptic drugs for the past 1 year. Which drug least likely decrease efficacy of antiepileptic drugs? a) Phenytoin B) Carbamazepine C) Valproic acid D) Lamotrigine 35) MP is a 23 year old women. In her medication profile she is valproic acid daily for seizure. She calls to your pharmacy and ask if she should stop taking her medication because for the past few weeks she has no seizures? What is the common reason for medication non compliance in patient using antiepileptic medications? A) because of drugs taste bad B) because recently had no seizure C) because expensive medications D) because complicated storage conditions 36) Ethosuximide is drug of choice for the following seizures: A. Absence seizures B. Tonic clonic C. status epilepticus D. A&B 37) Oral contraceptives interact with: A. Antibiotics B. Anticonvulsant C. Antifungals D. All of the above 38) What drugs should be avoided in absence seizure? A. Phenytoin and valproic acid B. Valproic acid and carbamazepine C. Gabapentin and Phenytoin D. Topiramate and Ethosuximide E. Phenytoin and Carbamazepine

39) Drugs that has least likely have drug interaction with oral contraceptives except: Copyright © 2000-2016 TIPS Inc. Unauthorized reproduction of this manual is strictly prohibited and it is illegal to reproduce without permission. This manual is being used during review sessions conducted by PharmacyPrep.

96-6

PharmacyPrep.com A. B. C. D.

Seizure and antiepileptic drugs

Gabapentin Ethosuximide Valproic acid Lamotrigine

40) One side effect of Phenytoin is gingival hyperplasia, the best way to prevent it: A. Use chlorhexidine wash B. Use dental floss C. Take Vitamin C D. Gargle with water 41) Anti epileptic drug that doesn’t cause weight gain? A. Valproic acid B. Lamotrigine C. Topiramate D. Phenobarbital 42) Phenytoin available in which of the dosage forms? A. Injection B. Capsule C. Chewable tablets D. All of the above 43) Valproic acid is the drug of choice for A. Absence seizure B. Myoclonus C. Tonic D. all of the above 44) Drug use as “add on” for patients nearly seizure free: A. Gabapentin B. Clobazam C. Valproic acid D. Phenytoin 45) Phenytoin metabolism is not inhibited by: A. Sulfonamide B. Isoniazid C. Carbamazepine D. Chloramphenicol 46) Divalproex is a mixture of: Copyright © 2000-2016 TIPS Inc. Unauthorized reproduction of this manual is strictly prohibited and it is illegal to reproduce without permission. This manual is being used during review sessions conducted by PharmacyPrep.

96-7

PharmacyPrep.com A. B. C. D.

Seizure and antiepileptic drugs

Gabapentin Valproic acid Sodium valproate A and C

47) What drug inhibits glutamate and aspartate release and blocks sodium channels and prevents repetitive firing: A. Topiramate B. Lamotrigine C. Clobazam D. Gabapentin 48) What antiepileptic is found in hospital emergence carts? A. Diazepam iv B. Chlorpromazine C. Carbamazepine D. Lithium E) Phenytoin IV

SEIZURE & ANTIEPILEPTICS Copyright © 2000-2016 TIPS Inc. Unauthorized reproduction of this manual is strictly prohibited and it is illegal to reproduce without permission. This manual is being used during review sessions conducted by PharmacyPrep.

96-8

PharmacyPrep.com

Seizure and antiepileptic drugs ANSWERS:

1) Ans: A 2. Ans: E Tips: Grand mal Valproic acid, Complex partial Carbamazepine, Simple partial Carbamazepine, Tonic-clonic Valproic acid, Absence seizure Ethosuximide, and Status epilepticus Diazepam 3) Ans: A 4) Ans: D 5) Ans: C 6) Ans: D Tips: topiramate is a derivative of fructose. Indicated as anti seizure drug—partial seizure, tonic clonic seizures. Has high kidney stone formation side effect. Fibrates also causes gall bladder stone formation 7) Ans: E Tips: valproic acid and valproate increase appetite and may results in weight gain, carbamazepine may cause dizziness and confusion. 8) Ans: D 9) Ans: B 10) Ans: A 11) Ans: A Tips: Gabapentin do not decrease efficacy of oral contraceptives and can be used along with oral contraceptives. 12) Ans: B 13) Ans: E Tips: butterfly rash symptoms occurs in lupus erythromatus 14- Ans: D 15- Ans: D Copyright © 2000-2016 TIPS Inc. Unauthorized reproduction of this manual is strictly prohibited and it is illegal to reproduce without permission. This manual is being used during review sessions conducted by PharmacyPrep.

96-9

PharmacyPrep.com

Seizure and antiepileptic drugs

16- Ans: A 17- Ans: B 18- Ans: D 19- Ans: B 20- Ans: D 21- Ans: C Tips; Ach receptors, in the CNS are present on less than 5% of the neuronal population. 22- Ans: A and B 23- Ans: B 24- Ans: C 25) Ans: E 26) Ans: A 27) Ans: A 28) Ans: C Tips: valproic acid is the drug of choice for generalized seizures and also can be used for petit mal seizures. 29. Ans: A Tips: Phenytoin syrup 100mg- more potent 8% increase in drug content; Phenytoin caps 100mg – free acid; Sodium salt of Phenytoin – Dilantin caps. 30) Ans: C 31) Ans: C 32) Ans: A 33) Ans: C Copyright © 2000-2016 TIPS Inc. Unauthorized reproduction of this manual is strictly prohibited and it is illegal to reproduce without permission. This manual is being used during review sessions conducted by PharmacyPrep.

96-10

PharmacyPrep.com

Seizure and antiepileptic drugs

Drugs that cause SJS: SASPAN: Sulphonamides, antiepileptic drugs (CBZ) 34) Ans: C 35)Ans: B Tips: Evidence shows there are two reasons seizure medication have compliance problems. 1) because of side effects patient may not take regularly. 2) patient recently had no seizure. Reference: Patient medication related non compliance. CE TT 2010 June 36) Ans:A 37)Ans: D

38) Ans: E Tips: Phenytoin and Carbamazepine are not use in absence seizure 39)Ans: B 40)Ans: A 41)Ans: C 42)Ans: D 43)Ans: D 44) Ans: B 45) Ans: C Tips: Carbamazepine stimulates phenytoin metabolism 46) Ans: D 47) Ans: B 48) Ans: A

Copyright © 2000-2016 TIPS Inc. Unauthorized reproduction of this manual is strictly prohibited and it is illegal to reproduce without permission. This manual is being used during review sessions conducted by PharmacyPrep.

96-11

PharmacyPrep.com

Seizure and antiepileptic drugs

Copyright © 2000-2016 TIPS Inc. Unauthorized reproduction of this manual is strictly prohibited and it is illegal to reproduce without permission. This manual is being used during review sessions conducted by PharmacyPrep.

96-12

PharmacyPrep.Com

Parkinson’s Disease PHARMACY PREP PARKINSON’S DISEASE

1) Parkinson patients are characterized by; A. Increased ratio of dopaminergic/cholinergic activity in neostratum B. most commonly occurs in elderly over 40 years of age patient C. Decreased ratio of dopaminergic/cholinergic activity in neostratum. D. All of the above 2-Involuntary movements typically involves mouth, face, limbs and trunk is characterized as: A-Trigeminal neuralgia B-Tardive dyskinesia C-Neuroleptic malignant syndrome D-Extrapyramidal symptoms E-Akathisia 3-Which of the following Catecholamine Methyl Transferase (COMT) inhibitor has fatal liver toxicity: A-Selegiline B-Tolcapone C-Entacapone D-Pergolide E-Amantadine 4-A 60-year-old adult who has been treated with haloperidol for 3 weeks. He presents with muscle stiffness, tremors. This is the most likely what kind of side effect: A-Acute dystonia B-Akathisia C-Tardive dyskinesia D-Parkinsonism disease E-Pseudo parkinsonism 5-Which of the following is NOT used in treatment of Extrapyramidal side effects (EPS)? A-Benztropine B-Diphenhydramine C-Trihexyphenidyl D-Donepezil E-All of the above 6-Selegiline is: A-COMT inhibitor B-MAO-B selective inhibitor C-Non-selective MAO inhibitor Copyright © 2000-2016 TIPS Inc. Unauthorized reproduction of this manual is prohibited. This manual is being used during review sessions conducted by PharmacyPrep.

97-1

PharmacyPrep.Com

Parkinson’s Disease

D-Levodopa preparations E-MAO-A selective inhibitor 7) Which of the following is a COMT inhibitor? A. Amantadine, B. Bromocriptine, C. Levodopa, D. Selegiline E. Entacapone 8) Why COMT inhibitors (Tolcapone, entacapone) always should used with levodopa? A-COMT inhibitor decrease metabolism of levodopa to 3-O-methyldopa B-COMT inhibitors have synergistic action with levodopa C-COMT inhibitors have additive action with levodopa D-COMT inhibitors should not combine with levodopa E-None of the above 9)All of the following statement is correct, except? A-Deficiency of dopamine gives Parkinson’s disease B-Tardive dyskinesia is NOT a symptom of Parkinson’s disease C-Selegiline is a selective MAO- type B inhibitor D-Tolcapone and entacapone are COMT inhibitor E-Ropinarole and pramipexole are non-selective dopamine agonist 10-Tardive Dyskinesia is? A-No movement or difficulty in movement B-Involuntary movement of limbs, trunk, mouth and face C-Slow movement D-Rigidity E-Postural instability 11)Which of the following antiviral drug indicated in Parkinson disease treatment: A-Pergolide B-Amantadine C-Rimantadine D-Selegiline E-Entacapone 12) A 75 yo Parkinson's patient is on levodopa/carbidopa (Sinemet) therapy taking morning and night time. However, recently patient experiencing rigidity and involuntary movement in the evening before taking night time dose? It is characterized as? A) fluctuation or on-off effect B) Dyskinesia C) Tardive dyskinesia D) Dystonia E) wearing off effect Copyright © 2000-2016 TIPS Inc. Unauthorized reproduction of this manual is prohibited. This manual is being used during review sessions conducted by PharmacyPrep.

97-2

PharmacyPrep.Com

Parkinson’s Disease

13) A doctor has entacapone therapy to a Parkinson's patient. Already is using levodopa/carbidopa (Sinemet SR). A) to treat wearing off effect B) To treat tardive dyskinesia C) to treat side effect of levodopa D) to potentiate action of levodopa therapy E) none of the above 14) A patient is on levodopa/carbidopa therapy for the past 3 yrs. Recently patient is getting periods of poor movements i.e return of tremor, rigidity or slowness. This conditions is referred as? A) Tardive dyskinesia B) Akathisia C) Dystonia D) on-off effect or fluctuations E) Freezing 15) Which of the following drug extends the levodopa effects? A) bromocriptine B) Selegiline C) Ropinirole D) Pramipexole E) Carbidopa 16) Which of the following is NOT a Parkinson's symptoms? A) tremors B) rigidity C) jerks D) gait E) Tardive dyskinesia 17) Which of the following drugs have levodopa sparing effect? A) Bromocriptine B) amantadine C) pramipexole D) Ropinirole E) All of the above 18) Which of the following is NOT side effect of levodopa? A) hypotension B) Arrhythmias C) peptic ulcer D) sudden onset of sleep E) pulmonary disease 19) Levodopa is taken empty stomach, what is correct? A) take 30 min before meals B) take 8 hours after meals C) take 2 hours after meals D) take 4 hours after meals 20) Hypofunction of nigrostriatal pathway resulting in decrease of dopamine levels and increase of acetylcholine levels. The deficiency in dopamine would cause: A. Alzheimer’s disease B. Parkinson’s disease C. Schizophrenia D. Serotoninergic syndrome 21) Which of the following is the COMT inhibitor; Copyright © 2000-2016 TIPS Inc. Unauthorized reproduction of this manual is prohibited. This manual is being used during review sessions conducted by PharmacyPrep.

97-3

PharmacyPrep.Com

Parkinson’s Disease

A. Tolcapone B. Entacapone C. Pergolide D. A&B 22) Treatment of Parkinson’s disease may be treated with all of the following agents, EXCEPT: A. Amantadine B. Selegiline C. Pergolide D. Donezepil E. Pramipexol 23) Pramipexole and Ropinirole are categorized as? A. Selective D 1 agonist B. Non selective D 1 and D 2 agonist C. Non selective D 1 and D 2 antagonist D. Selective D 2 agonist 24) The following antiviral drug can be used in PD treatment: A. Levodopa B. levodopa/carbidopa C. Pergolide D. Amantadine 25) What is the most common side effect of levodopa? A. Tremor B. Dyskinesia C. Hallucination D. Hypotension 26) MP is a 75 year old patient diagnosed with Parkinson's disease. Doctor considering to prescribe medication. What is the drug of choice for Parkinson disease is? A. Levodopa B. Carbidopa C. A and B D. Dopamine E. Selegiline 27) Which of the following drug should be avoided in Parkinson's disease? A. Oral contraceptive pills B. Metochlopramide C. B and D D. Antipsychotic drug

Copyright © 2000-2016 TIPS Inc. Unauthorized reproduction of this manual is prohibited. This manual is being used during review sessions conducted by PharmacyPrep.

97-4

PharmacyPrep.Com

Parkinson’s Disease

28. What is not a Parkinson disease symptom? A. Tardive dyskinesia B. Akathisia C. Tremor D. Bradykinesia 29) In treating parkinsonism, there is an increase of dopamine but on the other hand it will increase also what neurotransmitter? A. GABA B. Acetylcholine C. Norepinephrine D. Epinephrine 30) Parkinson's disease symptoms characterized by difficulty in initiating movements is: A. Akinesia B. Bradykinesia C. Dyskinesia D. Anaesthesia 31) Over treatment of anti-Parkinson's drugs can result to: A. Alzheimer's B. Psychosis C. Seizure D. Respiratory arrest 32) An antinauseating drug that should be avoided in Parkinson patients: A. Metoclopramide B. Diclectin C. A and B D. Ondansetron 33) Which is not true about Parkinson drug? A. Levodopa does penetrate brain B. Carbidopa does not penetrate brain C. Dopamine does not penetrate the brain D. All of the above 34) How do you manage “wearing off”? A. Switch to a different dopamine agonist B. Stop the drug right away C. Decrease dose of dopamine agonist D. None of the above

Copyright © 2000-2016 TIPS Inc. Unauthorized reproduction of this manual is prohibited. This manual is being used during review sessions conducted by PharmacyPrep.

97-5

PharmacyPrep.Com

Parkinson’s Disease

35) Nausea and vomiting are adverse effects of Levodopa, how do you manage this? A. Take levodopa with food B. Take levodopa without food C. Take it on empty stomach D. Do not crash or chew 36) Which of the following is the suitable combination of drugs are initiated after as drug of choice for Parkinson disease is? A. Levodopa and carbidopa B. Carbidopa and Dopamine C. Levodopa and Tolcapone D. Dopamine and Entacapone 37) Which of the following antiviral drug indicated in Parkinson disease treatment: A. Valacyclovir B. Amantadine C. Acyclovir D. selegiline E. Interferon's 38) Parkinson’s disease is caused by deficiency of? A-Dopamine in brain B-Acetylcholine C-GABA in CNS E-Histamines

D-Serotonin in brain

Copyright © 2000-2016 TIPS Inc. Unauthorized reproduction of this manual is prohibited. This manual is being used during review sessions conducted by PharmacyPrep.

97-6

PharmacyPrep.Com

Parkinson’s Disease Parkinson's Disease ANSWERS

1- Ans: A Tips: That is why Dopamine agonist & anticholinergics are effective 2- Ans: B Tips: dyskinesia = slow movement, tardive dyskinesia = involuntary movement of mouth, face, limbs and trunk. Akinesia = difficulty in movement. 3- Ans: B 4- Ans: E Tips: Pseudo parkinsonism (TRAP) may be side effects of antipsychotics. Pseudo parkinsonism may disappear after discontinuation of medication. 5- Ans: D Tips: anticholinergic agents such as benztropine, diphenhydramine, and trihexyphenidyl are use in treatment of EPS. Donepezil is acetyl cholinesterase inhibitor is not indicated in EPS treatment. 6- Ans: B Tips: MAO-B selective inhibitors; treatment for Parkinson’s disease is: BALSAC = Bromocriptine, Amantadine, Levodopa, Selegiline, Antimuscarinic (benztropine), and COMT inhibitors 7) Ans: E 8) Ans: A 9) Ans: E Tips: Ropinirole and pramipexole are selective D 2 agonist 10- Ans: B 11) Ans: B 12) Ans: E 13) Ans: A 14) Ans: D Tips: End-of-dose wearing off gives on-off effect. This phenomenon is related to the increasing loss neuronal storage capability of dopamine as well as short half life of dopa. Copyright © 2000-2016 TIPS Inc. Unauthorized reproduction of this manual is prohibited. This manual is being used during review sessions conducted by PharmacyPrep.

97-7

PharmacyPrep.Com

Parkinson’s Disease

15) Ans: B 16) Ans: D 17) Ans: E 18) D 19)Ans: A 20) Ans:B 21) Ans: D Tips: COMT (Catecholamine O-Methyl transferase), the drugs names ends with “capone”. COMT are indicated in Parkinson's disease 22) Ans: D Tips: Donepezil is a drug for Alzheimers 23)Ans: D 24) Ans: D 25) Ans: B 26) Ans: C 27)Ans: C 28. Ans: A 29) Ans: B 30) Ans: A 31)Ans: B 32)Ans: A 33) Ans: D 34)Ans: A Tips: To manage “wearing off”:Administer levodopa more frequently Copyright © 2000-2016 TIPS Inc. Unauthorized reproduction of this manual is prohibited. This manual is being used during review sessions conducted by PharmacyPrep.

97-8

PharmacyPrep.Com

Parkinson’s Disease

Switch to a different dopamine agonist Add or increase dose of a dopamine agonist Switch to Sinemet CR 35) Ans: A Tips: Take levodopa with food and increase dose of levodopa or dopamine agonist gradually over several weeks 36)Ans: C Tips: Sinemet is the combination of levodopa and carbidopa 37) Ans: B 38) Ans: A

Copyright © 2000-2016 TIPS Inc. Unauthorized reproduction of this manual is prohibited. This manual is being used during review sessions conducted by PharmacyPrep.

97-9

PharmacyPrep.Com

Parkinson’s Disease

Copyright © 2000-2016 TIPS Inc. Unauthorized reproduction of this manual is prohibited. This manual is 97-10 being used during review sessions conducted by PharmacyPrep.

Pharmacyprep.com

Antimicrobial agents PHARMACY PREP ANTI-INFECTIVE AGENTS – ANTIBIOTICS

1. What symptoms distinguish complicated and uncomplicated UTI? A. Blood in urine (turbid urine) B.fever C. pain, fever and chills D. frequency of urination E. urgency of urination Ans.A 2.What is empiric treatment of urinary tract infection, patient has allergy to penicillin? A. cotrimoxazole 3 d B. Fluroquinolone 3d C. Amoxicillin 3 d D. Cephalexin 7 d E. Aminoglycoside 14 d Ans. A

Copyright © 2000-2016 TIPS Inc. Unauthorized reproduction of this manual is prohibited. This manual is being used during review sessions conducted by PharmacyPrep.

98-1

PharmacyPrep.Com

Anticancer drugs and chemotherapy PHARMACY PREP ANTICANCER DRUGS AND CHEMOTHERAPY

1. What is the drug of choice to treat chemo induced nausea and vomiting A. dexamethsone

B. Ondansetron

C. Metoclopramide

D. dimenhydrinate

Ans. a 2) Which of the following chemotherapeutic agent is categorized as low emitogenic? A) Cisplatin B) Cyclophosphamide C) Iphosphomide D) Vincristine Ans. D 3) Hemorrhagic cystitis results from irritation of the lining of the bladder by acrolein, a metabolite of ifosfamide and cyclophosphamide. Acrolein can be inactivated by? A-Mesna B-Doxarazoxane C-Chlropropamide D-Erythropoeitins E-None of the above Ans. A 4) The drug of choice to treat delayed nausea and vomiting? A-Ondansetron B-Metoclopramide C-Dexamethasone D-Dimenhydrinate E-Scopolamine 5) Cardio protective agent is? A-Mesna B-Dexorasoxane Ans. B

C-Acroline

D-None of the above

6. Chemotherapy induced nausea and vomiting is mediated through number of receptors, however which of the following are the most important in CINV? A. LTC4 and LTD4 B. M1 and 5HT3 receptors C. H1 and 5HT3 receptors D. 5HT3 and NK-1 receptors E.5HT1b/1d and 5HT2 Ans. D Tips. The CINV is commonly associated with 5HT3 and neurokinin-1 receptors. Metoclopramide and Prochlorpromazine are dopamine antagonist is used for management of low emitogenic CINV. Copyright © 2000-2016 TIPS Inc. Unauthorized reproduction of this manual is prohibited. This manual is being used during review sessions conducted by PharmacyPrep.

99-1

Pharmacy Prep Natural Products and Pharmacognosy 1) A customer of your pharmacy wants to buy ginkgo biloba to enhance memory. What to do? A) give gingko biloba B) do not recommend gingko C) refer to doctor D) refer to emergency E) say there is no clinical evidence 2) A customer of your pharmacy wants to buy ginkgo biloba to enhance memory. Patient currently on warfarin therapy . What to do? A) give gingko biloba B) do not recommend gingko because, has reports of drug interactions C) refer to doctor D) refer to emergency E) say there is no clinical evidence 3) A customer of your pharmacy wants to buy garlic pills to treat borderline high cholesterol. What to do? A) give garlic pills B) do not recommend garlic pills C) refer to doctor D) refer to emergency E) say there is no clinical evidence 4. Which of the following herbal products can increase blood pressure? A. Garlic B. St. John wart C. Licorice D. Valerian E. Feverfew 5. Health Canada approved Natural or herbal products are identified by? A. Drug identification number B. Product identification number C. Natural Product number D. Natural Health Products E. Prescription number

Copyright © 2000-2016 TIPS Inc. Unauthorized reproduction of this manual is prohibited. This manual is being 100-1 used during review sessions conducted by PharmacyPrep.

ANSWERS Natural Products and Pharmacognosy 1) Ans: A Tips: As long as patient is not medication and/or have no medical conditions that do not interfere with natural products if this helps to alleviate their problems. I will give it to patient. 2) Ans: B 3) Ans: C 4. Ans: C 5. Ans. C

Copyright © 2000-2016 TIPS Inc. Unauthorized reproduction of this manual is prohibited. This manual is being 100-2 used during review sessions conducted by PharmacyPrep.